55
Pontifícia Universidade Católica do Paraná Processo Seletivo de Residência Médica 20 de Janeiro de 2013 ANESTESIOLOGIA CIRURGIA GERAL CLÍNICA MÉDICA MEDICINA DA FAMÍLIA E COMUNIDADE NEUROCIRURGIA NEUROLOGIA OFTALMOLOGIA ORTOPEDIA E TRAUMATOLOGIA RADIOLOGIA INFORMAÇÕES / INSTRUÇÕES: 1. Verifique se a prova está completa: questões de números 1 a 100. 2. A compreensão e a interpretação das questões constituem parte integrante da prova, razão pela qual os fiscais não poderão interferir. 3. Preenchimento do Cartão-Resposta: - Preencher para cada questão apenas uma resposta - Preencher totalmente o espaço correspondente, conforme o modelo: - Usar caneta esferográfica, escrita normal, tinta azul ou preta. - Para qualquer outra forma de preenchimento, a leitora anulará a questão. O CARTÃO-RESPOSTA É PERSONALIZADO. NÃO PODE SER SUBSTITUÍDO, NEM CONTER RASURAS. Duração total da prova: 4 horas - - - - - - - - - - - - - - - - - - - - - - - - - - - - - - - - - - - - - - - - - - - - - - - - - - - - - - - - - - - - - - - - - - - - - - - - Anote o seu gabarito. 1. 2. 3. 4. 5. 6. 7. 8. 9. 10. 11. 12. 13. 14. 15. 16. 17. 18. 19. 20. 21. 22. 23. 24. 25. 26. 27. 28. 29. 30. 31. 32. 33. 34. 35. 36. 37. 38. 39. 40. 41. 42. 43. 44. 45. 46. 47. 48 49. 50. 51. 52. 53. 54. 55. 56. 57. 58. 59. 60. 61. 62. 63. 64. 65. 66. 67. 68. 69. 70. 71. 72. 73. 74. 75. 76. 77. 78. 79. 80. 81. 82. 83. 84. 85. 86. 87. 88. 89. 90. 91. 92. 93. 94. 95. 96. 97. 98. 99. 100.

Pontifícia Universidade Católica do Paraná Processo ...static.medgrupo.com.br/static/concursos/editais/HUC/2013/GABARITOS... · A) Homem de 24 anos após uso inalatório de cocaína

  • Upload
    ngonhan

  • View
    215

  • Download
    0

Embed Size (px)

Citation preview

Page 1: Pontifícia Universidade Católica do Paraná Processo ...static.medgrupo.com.br/static/concursos/editais/HUC/2013/GABARITOS... · A) Homem de 24 anos após uso inalatório de cocaína

Pontifícia Universidade Católica do Paraná Processo Seletivo de Residênc ia Médica

20 de Janeiro de 2013

ANESTESIOLOGIA CIRURGIA GERAL CLÍNICA MÉDICA

MEDICINA DA FAMÍLIA E COMUNIDADE NEUROCIRURGIA

NEUROLOGIA OFTALMOLOGIA

ORTOPEDIA E TRAUMATOLOGIA RADIOLOGIA

INFORMAÇÕES / INSTRUÇÕES: 1. Verifique se a prova está completa: questões de números 1 a 100. 2. A compreensão e a interpretação das questões constituem parte integrante da prova, razão pela qual os

fiscais não poderão interferir. 3. Preenchimento do Cartão-Resposta :

- Preencher para cada questão apenas uma resposta - Preencher totalmente o espaço correspondente, conforme o modelo: - Usar caneta esferográfica, escrita normal, tinta azul ou preta. - Para qualquer outra forma de preenchimento, a leitora anulará a questão.

O CARTÃO-RESPOSTA É PERSONALIZADO. NÃO PODE SER SUBSTITUÍDO, NEM CONTER RASURAS.

Duração total da prova: 4 horas �- - - - - - - - - - - - - - - - - - - - - - - - - - - - - - - - - - - - - - - - - - - - - - - - - - - - - - - - - - - - - - - - - - - - - - - - Anote o seu gabarito.

1.

2.

3.

4.

5.

6.

7.

8.

9.

10.

11.

12.

13.

14.

15.

16.

17.

18.

19.

20.

21.

22.

23.

24.

25.

26.

27.

28.

29.

30.

31.

32.

33.

34.

35.

36.

37.

38.

39.

40.

41.

42.

43.

44.

45.

46.

47.

48

49.

50.

51.

52.

53.

54.

55.

56.

57.

58.

59.

60.

61.

62.

63.

64.

65.

66.

67.

68.

69.

70.

71.

72.

73.

74.

75.

76.

77.

78.

79.

80.

81.

82.

83.

84.

85.

86.

87.

88.

89.

90.

91.

92.

93.

94.

95.

96.

97.

98.

99.

100.

Page 2: Pontifícia Universidade Católica do Paraná Processo ...static.medgrupo.com.br/static/concursos/editais/HUC/2013/GABARITOS... · A) Homem de 24 anos após uso inalatório de cocaína

Pontifícia Universidade Católica do Paraná – Programa de Residência Médica – Janeiro/ 2013 Pág. 1

Page 3: Pontifícia Universidade Católica do Paraná Processo ...static.medgrupo.com.br/static/concursos/editais/HUC/2013/GABARITOS... · A) Homem de 24 anos após uso inalatório de cocaína

Pontifícia Universidade Católica do Paraná – Programa de Residência Médica – Janeiro/ 2013 Pág. 2

O caso clínico a seguir servirá de base para as próximas 3 (três) questões.

Sra. Julia, 72 anos, vem ao consultório para trazer os exames solicitados na primeira consulta e receber a orientação terapêutica. Repetido o exame físico encontram-se: Peso: 48 kg, PA: 170/98 mmHg em pé e sentada; diminuição do pulso tibial posterior à direita; ITB de 0.6; presença de B4. Nos exames complementares apresentava: ECG: sobrecarga ventricular esquerda, colesterol total: 265 mg/dL; HDL: 28 mg/dL; triglicerídeos: 456 mg/dL; glicemia: 109 mg/dL e TTG: 158 mg/dL; ácido úrico: 2.8 mg/dL; creatinina: 1.3 mg/dL.

1. Considerando os dados obtidos, as melhores

evidências científicas e a recomendação da VI Diretrizes Brasileiras de Hipertensão Arterial, o risco estratificado para esta paciente é:

A) Alto risco adicional. B) Moderado risco adicional. C) Baixo risco adicional. D) Risco basal. E) Risco adicional muito alto.

2. Considerando os dados obtidos, as melhores

evidências científicas e a recomendação da VI Diretrizes Brasileiras de Hipertensão Arterial, a taxa de filtração glomerular estimada-TFGE (Cockcroft-Gault) e, uma vez que não se pode obter o valor do LDL-Col pela fórmula de Friedewald, deve-se optar pelo cálculo do “não-HDL” nestes casos. Dado este contexto, os valores obtidos para TFGE e não-HDL, são:

A) TFGE: 29,64 mL/min e não-HDL: 237 mg/dL. B) TFGE: 34,87 mL/min e não-HDL: 237 mg/dL. C) TFGE: 42.34 mL/min e não-HDL: 187 mg/dL. D) TFGE: 29,64 mL/min e não-HDL: 187 mg/dL. E) TFGE: 34,87 mL/min e não HDL: 187 mg/dL.

3. Considerando os dados obtidos, as melhores

evidências científicas e a recomendação da VI Diretrizes Brasileiras de Hipertensão Arterial, a melhor opção terapêutica inicial (Via Oral) para a paciente, seria:

A) Atenolol 100 mg/dia e hidroclorotiazida 25

mg/dia. B) Hidroclorotiazida 25 mg/dia. C) Losartana 100 mg/dia e furosemida 40 mg/dia. D) Ramipril 5 mg/dia e hidroclorotiazida 25

mg/dia. E) Losartana 100 mg/dia.

4. Sr. João Doe, 49 anos, vem à consulta com queixa

de palpitação e tontura. Feito o ECG (a seguir), qual o diagnóstico eletrocardiográfico?

A) Síndrome de Lown–Ganong–Levine. B) Sobrecarga Ventricular Esquerda. C) Síndrome de Wolff–Parkinson–White. D) Bloqueio da Divisão Póstero-Inferior do Ramo

Direito. E) Eletrocardiograma dentro dos padrões da

normalidade.

5. Sr. João Doe, 49 anos (mesmo paciente da questão anterior), após a consulta com cardiologista, que o informou de que estava com fibrilação atrial, cai desacordado ao sair do consultório. Durante o atendido de emergência o traçado do ECG obtido foi este:

Ao revisar a medicação em uso pelo paciente nos últimos seis meses, foi identificado um medicamento que poderia ser a causa deste evento. Dado este contexto, pergunta-se: qual destes medicamentos poderia, provavelmente, ter contribuído para isto?

A) Amiodarona. B) Procainamida. C) Sotalol. D) Digoxina. E) Metoprolol.

6. Sra. Maria Alice, 28 anos, procura o médico com uma

queixa de dois anos de evolução, que iniciou com crises de dor que sucedia à alteração de coloração das extremidades superiores. Em princípio era somente palidez. Passados alguns meses este evento era seguido de cianose e algumas semanas após o surgimento dessa cianose, ao final da crise, manifestava-se com a tríade: palidez, cianose e

Page 4: Pontifícia Universidade Católica do Paraná Processo ...static.medgrupo.com.br/static/concursos/editais/HUC/2013/GABARITOS... · A) Homem de 24 anos após uso inalatório de cocaína

Pontifícia Universidade Católica do Paraná – Programa de Residência Médica – Janeiro/ 2013 Pág. 3

rubor. Há 3 semanas apresentou pontos de necrose em polpas digitais bastante dolorosos. Considerando estas queixas, o diagnóstico mais provável é:

A) Ergotismo. B) Acrocianose. C) Livedo reticular. D) Eritromelalgia. E) Fenômeno de Raynaud.

7. Dr. Alberto estava muito preocupado com a chegada

do Sr. Antônio ao pronto-atendimento exatamente um ano após seu infarto agudo do miocárdio. Embora se considerasse um paciente obediente não havia parado de fumar. Ao chegar apresentava forte dor abdominal difusa, de início súbito há 40 minutos, seguida de náusea e vômito. Ao exame estava pálido, com sudorese importante, PA: 60/30 mmHg. Solicitado uma radiografia simples de abdome que evidenciou: alças intestinais superpostas, imóveis e com espessamento de parede. Logo após a realização do Raio X, apresentou um episódio importante de diarreia sanguinolenta. Dado este contexto, pergunta-se: qual o diagnóstico mais provável com os dados clínicos e laboratoriais apresentados?

A) Oclusão arterial mesentérica aguda. B) Trombose venosa mesentérica aguda. C) Mesenterite retrátil. D) Torção do grande epíploo. E) Pancreatite aguda.

8. Uma professora de português aposentada vem ao

consultório, trazida por familiares, pois subitamente, ao se deparar com uma texto escrito de Carlos Drummond de Andrade, o qual gostava muito, foi capaz de reconhecer as letras, mas incapaz de reconhecer as palavras ou o significado destas. A este achado semiológico denomina-se:

A) Alexia. B) Dislexia. C) Agnosia visual. D) Simultagnosia. E) Ataxia óptica.

9. O Sr. Rodolfo está internado com forte suspeita de pancreatite aguda. Foram solicitados alguns exames laboratoriais, colhidas amostras de lugares diferentes (sangue, urina e derrame cavitário). Dado este contexto, qual deles deveria estar alterado nas três amostras para a confirmação diagnóstica?

A) Lipase. B) Tripsina. C) Cálcio D) Meta-hemalbumina. E) Amilase.

10. Sra. Clara, 81 anos, veio à consulta no CMUM, no dia 02 de janeiro após ter abusado da alimentação no final de ano, principalmente da maionese. Apresentou

vários episódios de vômito e diarreia nas últimas horas. Encontraram-se mucosas secas, pele com turgor e elasticidade diminuídos, FC: 108 bpm e PA 110/70 mmHg (deitada) e 80/40 mmHg (em pé). Foram solicitados vários exames laboratoriais antes de medicá-la. A relação da osmolaridade urinária e plasmática (Osm U/P) esperada neste caso é:

A) = 1 B) < 1 C) > 1 D) ≥ 1 E) ≤ 1

11. Homem de 20 anos procura atendimento médico

devido à dispneia de início súbito. Nega comorbidades e refere apenas tabagismo de 6 maços/ano. Nega trauma, viagens ou alterações em membros inferiores. Em seu exame físico mostrava-se taquidispneico e com murmúrio vesicular abolido em hemitórax direito. Sua radiografia de tórax comprovava a presença de pneumotórax direito. Considerando pneumotórax espontâneo primário (PEP) e pneumotórax espontâneo secundário (PES) leia as assertivas abaixo e assinale a CORRETA.

I. O PEP ocorre caracteristicamente em pacientes

jovens e parece possuir relação com o tabagismo.

II. O PES pode estar associado à doença pulmonar obstrutiva crônica e até mesmo à infecção pulmonar por Pneumocystis jiroveci.

III. A principal causa etiológica associada ao PEP é a presença de pequenas bolhas subpleurais, as quais podem estar presentes em 50 a 80% dos pacientes que desenvolvem a doença.

Está(ão) correta(s) APENAS :

A) II. B) I, II e III. C) I e II. D) I e III. E) II e III.

12. Mulher de 68 anos, hipertensa, diabética e

depressiva, procurou atendimento médico devido à tosse produtiva, dispneia aos médios esforços e febre de 37,9oC iniciados há 3 dias. Negava tabagismo. Seus dados vitais de chegada eram:

• PA: 110/70mmHg • Pulso: 100bpm • FR: 24ipm • Taxilar: 38,0oC • SpO2: 93% (ar ambiente)

Em seu exame físico identificavam-se crepitantes pulmonares em base esquerda. Durante toda a consulta mostrava-se lúcida e cooperativa. Considerando o diagnóstico desta paciente, leia as assertivas a seguir:

Page 5: Pontifícia Universidade Católica do Paraná Processo ...static.medgrupo.com.br/static/concursos/editais/HUC/2013/GABARITOS... · A) Homem de 24 anos após uso inalatório de cocaína

Pontifícia Universidade Católica do Paraná – Programa de Residência Médica – Janeiro/ 2013 Pág. 4

I. Se a dosagem de ureia sérica desta paciente for 60 mg/dL, ela preencherá 2 critérios do escore CURB-65.

II. O seu tratamento pode ser feito com uma fluorquinolona respiratória, como a azitromicina.

III. Streptococcus pneumoniae, Mycoplasma pneumoniae e Haemophilus influenzae são possíveis agentes etiológicos para esta condição.

Está(ão) correta(s) APENAS :

A) III. B) I e III. C) I e II. D) I. E) II e III.

13. Sobre as infecções fúngicas pulmonares, leia as

assertivas abaixo e assinale V para as verdadeiras, F para as falsas. Em seguida marque a alternativa que possui a ordem CORRETA.

( ) A paracoccidioidomicose pode apresentar achado radiográfico de lesões pulmonares bilaterais, em “asa de borboleta”. O Itraconazol pode ser considerado como forma de tratamento. ( ) O aspergiloma ou micetoma pode ser um achado incidental em radiografias de tórax de pacientes que estão em investigação de hemoptise e geralmente acometem os lobos superiores. ( ) Criptoccose é a infecção pulmonar de origem fúngica mais comum e manifesta-se como nódulos ou massas nos imunodeprimidos e com padrão intersticial nos imunocompetentes.

A) F, V, F B) V, F, F C) V, V, V D) F, F, V E) V, V, F

14. Sobre a avaliação da doença pulmonar obstrutiva

crônica, leia as assertivas abaixo e assinale V para as verdadeiras, F para as falsas. Em seguida marque a alternativa que possui a ordem CORRETA.

( ) A Espirometria deve ser realizada preferencialmente na fase estável da doença, sendo que a relação VEF1/CVF menor que 70% após broncodilatador confirma a presença de limitação ao fluxo aéreo. ( ) VEF1 após broncodilatador, quando analisado isoladamente, é útil na determinação do estágio da doença. ( ) A Escala do Medical Research Council (MRC) pode ser utilizada como forma de classificação da dispneia e possui relação com o prognóstico da doença.

A) V, V, V B) V, V, F C) V, F, F D) F, V, F E) F, F, V

15. Qual dos pacientes abaixo apresentaria miose como manifestação de intoxicação exógena aguda?

A) Homem de 24 anos após uso inalatório de

cocaína. B) Mulher de 60 anos após intoxicação exógena

intencional por morfina. C) Homem de 40 anos após intoxicação exógena

intencional por bupropiona. D) Mulher de 32 anos após intoxicação exógena

intencional por mirtazapina. E) Mulher de 18 anos após intoxicação exógena

intencional por amitriptilina.

16. Paciente de 70 anos, portador de hipertensão, dislipidemia e insuficiência renal crônica não dialítica (creatinina basal: 2,4 mg/dL), iniciou há 4 dias com diarreia líquida, várias vezes ao dia, sem sangue, muco ou pus. Admitia vômitos alimentares. Negava febre. Em seu exame físico apresentava-se taquicárdico e desidratado. Em seus exames laboratoriais destacam-se:

• Creatinina: 3,8 mg/dL • Ureia: 200 mg/dL • Potássio: 6,8 mEq/L • Sódio: 150 mEq/L

Dado este quadro clínico, leia as assertivas abaixo:

I. Deve-se realizar um eletrocardiograma. II. O tratamento de sua hipercalcemia deve ser feito

com furosemida e resina de troca intestinal. III. Hidratação e uso de solução polarizante (10UI

insulina regular em 50 g de glicose) podem ser considerados no seu tratamento.

Está(ão) correta(s) APENAS :

A) III. B) II e III. C) I e III. D) I e II. E) I.

17. Paciente de 50 anos internado na enfermaria de

clínica médica evoluiu com parada cardiorrespiratória presenciada em fibrilação ventricular. O paciente recebeu atendimento preconizado pelas condutas do Suporte Avançado de Vida em Cardiologia (American Heart Association) e retornou a ritmo sinusal, com pulso central palpável e pressão arterial efetiva. Durante seu atendimento, além dos choques monofásicos e da massagem torácica, recebeu uma dose de 1mg de adrenalina e uma dose de 300 mg de amiodarona. Dado este contexto, pergunta-se: qual a assertiva abaixo possui as doses de manutenção de amiodarona preconizadas para as próximas 24 horas?

A) 2 mg de amiodarona por minuto nas primeiras

6 horas seguidas por 1 mg de amiodarona por minutos nas próximas 18 horas.

Page 6: Pontifícia Universidade Católica do Paraná Processo ...static.medgrupo.com.br/static/concursos/editais/HUC/2013/GABARITOS... · A) Homem de 24 anos após uso inalatório de cocaína

Pontifícia Universidade Católica do Paraná – Programa de Residência Médica – Janeiro/ 2013 Pág. 5

B) 0,5 mg de amiodarona por minuto nas primeiras 6 horas seguidas por 0,25 mg de amiodarona por minutos nas próximas 18 horas.

C) 10 mg de amiodarona por minuto nas primeiras 6 horas seguidas por 5 mg de amiodarona por minutos nas próximas 18 horas.

D) 1 mg de amiodarona por minuto nas primeiras 6 horas seguidas por 0,5 mg de amiodarona por minuto nas próximas 18 horas.

E) 1 mg de amiodarona por minuto nas primeiras 24 horas.

18. Homem de 60 anos, portador de cirrose hepática por hepatite C crônica, foi trazido para atendimento devido à hematêmese e a melena iniciadas há 8 horas. Os familiares negam síncope, mas admitem que o paciente encontra-se mais sonolento que seu habitual. Traziam endoscopia digestiva alta realizada há 4 meses revelando 2 varizes de esôfago de grosso calibre. O paciente não fazia uso de medicações. Seus dados vitais na chegada eram:

• PA: 100/60 mmHg • Pulso: 112 bpm • FR: 20 ipm • Taxilar: 36,8oC • SpO2: 95% (ar ambiente)

Em seu exame físico encontravam-se ginecomastia e ascite moderada. Dado este quadro clínico, afirma-se:

I. A profilaxia para peritonite bacteriana espontânea

deve ser iniciada nos pacientes que se apresentem com hemorragia digestiva alta varicosa.

II. Nos casos de hemorragia digestiva alta varicosa, o uso de terlipressina é indicado.

III. Deve iniciar imediatamente tratamento com metoprolol 50 mg de 12/12 horas e omeprazol 8 mg/hora em bomba infusora.

Está(ão) correta(s) APENAS :

A) I e III. B) III. C) I e II. D) II e III. E) II.

19. Homem de 63 anos, tabagista, hiperuricêmico e

dislipidêmico, realizou investigação de quadro dispéptico e diagnosticou uma úlcera bulbar ativa com presença de Helicobacter pylori pelo método da urease. O paciente passou por tratamento antibiótico para erradicação desta bactéria e vem ao consultório para reavaliação. Qual das assertivas abaixo possui o teste preconizado como padrão-ouro para documentar a erradicação do Helicobacter pylori após este tratamento?

A) Sorologia para Helicobacter pylori. B) Teste da urease. C) Histologia gástrica. D) Determinação de antígenos fecais de

Helicobacter pylori. E) Teste respiratório da ureia.

20. Mulher de 46 anos procura atendimento devido à

paralisia facial periférica esquerda (Paralisia de Bell) iniciada há 24 horas. Nega febre, trauma ou alterações auditivas. Em seu exame físico comprovava-se apenas a presença da paralisia facial periférica esquerda. Dado este quadro clínico, afirma-se:

I. A lesão periférica do nervo facial leva ao

comprometimento do movimento da hemiface homolateral a esta.

II. O aciclovir possui superioridade à prednisona no tratamento da Paralisia de Bell e deve ser iniciado em todos os casos.

III. Todo paciente com paralisia facial periférica deve ser submetido a exame de imagem do crânio, como tomografia computadorizada e/ou ressonância nuclear magnética.

Está(ão) correta(s) APENAS :

A) I e III. B) I e II. C) I, II e III. D) I. E) II.

21. A cicatrização é um processo dinâmico que envolve

uma sequência de fases mediadas por processos celulares coordenados e fatores hormonais. A primeira fase, conhecida como fase inflamatória, ocorre do início da lesão até 4 a 6 dias. As células envolvidas são os neutrófilos (pico em 24 horas) e macrófagos (48 a 96 horas). As citocinas são os fatores moduladores. Nesta primeira fase (inflamatória), as principais citocinas envolvidas são:

A) IL-1, TNF-α, TFG-β B) VEGF, FGF, TNF C) PDGF,EGF e TNF, IL-1 D) FGF, TNF-α, IL-6 E) TNF-α. IL-6, IL-10

22. Paciente com 23 anos apresenta tumoração redutível

em região inguinal D, após esforço físico. Indicada correção cirúrgica. No transoperatório o cirurgião confirma uma hérnia direta ou anteromedial ou ainda oblíqua interna. De acordo com a Classificação de Nyhus, pode-se afirmar.

A) Hérnia tipo III (A) exteriorizada medialmente

aos vasos epigástricos. B) Hérnia tipo I exteriorizada medialmente aos

vasos epigástricos. C) Hérnia tipo II exteriorizada medialmente aos

vasos epigástricos. D) Hérnia tipo III (B) exteriorizada medialmente

aos vasos epigástricos.

Page 7: Pontifícia Universidade Católica do Paraná Processo ...static.medgrupo.com.br/static/concursos/editais/HUC/2013/GABARITOS... · A) Homem de 24 anos após uso inalatório de cocaína

Pontifícia Universidade Católica do Paraná – Programa de Residência Médica – Janeiro/ 2013 Pág. 6

E) Hérnia tipo IV exteriorizada medialmente aos vasos epigástricos.

23. A combinação de nível elevado de PTH-sérico,

hipercalcemia na ausência de hipocalciuria e hipofosfatemia é quase sempre patognomônica de hiperparatireoidismo.

A) A causa mais comum de hiperparatireoidismo

primário é o carcinoma seguido de hiperplasia. B) O hiperparatireoidismo pode ocorrer na

neoplasia endócrina múltipla tipo I (Síndrome de Werner) e na neoplasia endócrina múltipla tipo IIa (Síndrome de Sipple).

C) As glândulas paratireoides apresentam função intimamente relacionada com o metabolismo de cálcio, através do PTH, que diminui a calcemia pelo bloqueio da mobilização óssea do calcio e aumenta a fosfatemia.

D) A hipocalcemia aguda produz sudorese fria, rubor facial, vômitos em jato e diarreia.

E) Pacientes com diagnóstico de hiperparatireoidismo primário dificilmente têm indicação cirúrgica.

24. J.T.S., pedreiro, foi submetido a um procedimento

cirúrgico convencional de apendicectomia. Após 22 dias deu entrada no serviço de emergência, apresentando febre, inapetência, cefaleia, sinais de desidratação. No local da cirurgia você observa um importante edema com rubor local e flutuação. Encaminhado ao centro cirúrgico para drenagem do abscesso, a infecção envolve músculo oblíquo externo e músculo transverso. Dado este contexto, assinale a alternativa CORRETA:

A) Não se pode afirmar que existe infecção do

sítio cirúrgico, pois infecção ocorre apenas até o 15° dia do pós-operatório.

B) Com estes dados pode-se afirmar que existe infecção do sítio cirúrgico tipo incisional superficial, provável infecção por MRSA.

C) Com estes dados pode-se afirmar que existe infecção do sítio cirúrgico tipo visceral, com provável infecção por Enterococcus sp e E.coli.

D) Não se pode afirmar que existe infecção do sítio cirúrgico, pois estas ocorrem apenas até o 15° dia do pós-operatório. Porém existe uma infecção de padrão superficial.

E) Com estes dados pode-se afirmar que existe infecção do sítio cirúrgico tipo incisional profunda, com provável infecção por Enterococcus sp e E.coli.

25. Paciente com nódulo de tireoide em investigação. Os

achados ultrassonográficos que denotam maior risco de câncer em um nódulo são:

I. Microcalcificações. II. Margens irregulares. III. Hipervascularização. IV. Nódulos hipoecoicos.

V. A PAAF (punção aspirativa com agulha fina) é obrigatória nos nódulos suspeitos.

Marque a alternativa CORRETA:

A) Apenas I e III são verdadeiras e V

complementa. B) Apenas IV é falsa. C) Apenas I, II e III são verdadeiras e V não

complementa. D) Apenas I, II, III, IV são verdadeiras e V

complementa. E) Apenas III é falsa.

26. O Sr. P.T.S., com 38 anos hígido previamente,

apresenta uma volumosa hérnia inguinal esquerda. O cirurgião opta pela técnica cirúrgica de Lichtenstein, para corrigir o defeito herniário. Você é designado(a) para realizar a avaliação pré-operatória. Após um exame clinico normal você deverá solicitar:

A) ECG (eletrocardiograma), glicemia, creatinina,

coagulograma e hemograma completo. B) Coagulograma e glicemia. C) Nenhum exame pré-operatório é necessário

para liberar o ato cirúrgico, pois paciente é hígido com menos de 40 anos.

D) Glicemia, creatinina, coagulograma e hemograma completo.

E) ECG (eletrocardiograma), glicemia, coagulograma e hemograma completo.

27. Desde 2006, no Brasil, a escala de gravidade de

doença hepática proposta por Child-Turcotte-Pugh (CTP), para seleção de pacientes com indicação de transplante hepático, foi substituída pelo chamado “ MELD score” . A mudança dos critérios tem em vista uma alocação de órgãos mais justa e baseada em critérios de severidade da doença.

A) A escala de MELD baseia-se em 5 valores

laboratoriais objetivos: creatinina sérica, bilirrubina total e RNI, proteínas total e gama gt.

B) A escala de MELD baseia-se em 3 valores laboratoriais objetivos: creatinina sérica, bilirrubina total e RNI e 2 critérios clínicos: hematêmese previa e ascite.

C) A escala de MELD baseia-se em 5 valores laboratoriais objetivos: creatinina sérica, bilirrubina total e RNI, proteínas total e gama gt e 2 criterios clínicos: ascite e icterícia.

D) A escala de MELD baseia-se em 3 valores laboratoriais objetivos: creatinina sérica, bilirrubina total e RNI.

E) A escala de MELD baseia-se em 4 critérios clínicos: ascite, icterícia, hematêmese prévia e ascite.

28. Paciente, 43 anos, trabalhador da construção civil,

veio ao serviço ambulatorial, com queixa de dispepsia. Durante a investigação na história clínica, paciente refere piora desta dispepsia há 3 meses, quando também apresentou discreta diminuição de

Page 8: Pontifícia Universidade Católica do Paraná Processo ...static.medgrupo.com.br/static/concursos/editais/HUC/2013/GABARITOS... · A) Homem de 24 anos após uso inalatório de cocaína

Pontifícia Universidade Católica do Paraná – Programa de Residência Médica – Janeiro/ 2013 Pág. 7

peso e inapetência. Você solicita exames complementares: endoscopia digestiva alta, ultrassonografia endoscópica e tomografia abdominal. Recebe, assim, um laudo de anatomia patológica informando Classificação Bormann tipo II ou ulcerado em pequena curvatura, com células em anel de sinete. Dado este quadro, sua conduta será:

A) Confirmação de neoplasia maligna, com

indicação precoce de gastrectomia, porém deve aguardar antes a ultrassonografia endoscópica.

B) Confirmação de neoplasia maligna, com indicação precoce de gastrectomia. A ultrassonografia endoscópica não é necessária neste caso.

C) Desconsideração da Classificação de Bormann, pois ela não se refere ao estadiamento das lesões gástricas.

D) Confirmação de neoplasia benigna, iniciar imediatamente bloqueador H2 ou inibidor de bomba de prótons.

E) Confirmação de neoplasia maligna, câncer gástrico precoce; a ultrassonografia endoscópica não é necessária. Indicar mucosectomia.

29. A dor abdominal no abdômen agudo deve ser

caracterizada pelo tipo, evolução, localização e irradiação e pode ser dividida em 3 tipos. De acordo com as classificações, afirma-se:

I. Dor visceral pura ou verdadeira é a que ocorre no

início do quadro, quando existe distensão do órgão sem a presença de inflamação. Dor de localização mal definida, geralmente ocorre na linha média e em cólica.

II. Peritoneal ou peritônio-pariental é aquela decorrente do comprometimento do peritônio-parietal pela inflamação visceral, traduzida pela defesa muscular ou sinais de irritação peritoneal ao exame físico.

III. As vísceras ocas intraperitoneais apresentam uma inervação sensitiva, sendo os impulsos transmitidos pelo sistema nervoso autônomo. O estímulo que desperta a dor geralmente é a distensão ou contração.

IV. A característica da dor do abdômen perfurativo é a dor tipo cólica, com intervalos variáveis.

V. A dor do abdômen hemorrágica é súbita, fraca, difusa, com intervalos curtos.

Marque a alternativa CORRETA:

A) Todas as afirmativas são verdadeiras. B) Apenas a afirmativa III é falsa. C) Apenas as afirmativas IV e V são verdadeiras. D) Apenas as afirmativas I, II, III e IV são

verdadeiras. E) Apenas a afirmativa IV é falsa.

30. Considera-se pólipo intestinal todo o tumor circunscrito que apresente protrusão desde a parede ate o lúmen intestinal. Os pólipos podem ser únicos ou múltiplos. Quando o número de pólipos é grande são classificados em polipose intestinal. As poliposes podem apresentar características únicas. Polipose adenomatosa familiar é caracterizada pela tríade: pólipos gastrointestinais, tumores de tecidos moles (cistos epidermoides) e osteomas, acompanhada de hipertrofia do epitélio pigmentário da retina. Pacientes com pólipo intestinal são portadores da:

A) Síndrome de Turcot. B) Síndrome de Gardner. C) Síndrome de Peutz-Jeghers. D) Síndrome de Cowden. E) Síndrome de Cronkhite-Canada.

31. Em relação ao trauma abdominal segundo o ATLS

Advanced Trauma Life Suport for Doctors, assinale a alternativa CORRETA:

I. A lavagem peritoneal diagnóstica LPD é um processo diagnóstico invasivo, de rápida execução, que alcança uma sensibilidade de 50% para a detecção de hemorragia intraperitoneal.

II. A lavagem peritoneal diagnóstica pode ser indicada em um doente vítima de traumatismo multissistêmico e hemodinamicamente instável com traumatismo cranioencefálico associado.

III. A tomografia computadorizada de abdome exige a transferência do doente para o setor de radiologia e está indicado apenas nos doentes hemodinamicamente estáveis.

IV. Trauma abdominal contuso com parada cardiorrespiratória na sala de emergência é uma indicação de toracotomia de reanimação na sala de emergência.

V. O lavado peritoneal diagnóstico é considerado positivo quando se detectam bactérias pela coloração pelo Gram, quando há 100.000 ou mais glóbulos vermelhos ou 500 ou mais glóbulos brancos por mm3

.

Está(ão) correta(s) APENAS a(s) afirmativa(s):

A) I e II. B) II, III e IV. C) II, III e V. D) II. E) III.

32. Paciente do sexo feminino, 59 anos, apresenta

quadro de dor abdominal tipo aperto e vômitos há 12 horas. Os sintomas iniciaram após a refeição da noite anterior. Não tem história de uso de álcool ou de medicamentos. Ao exame físico, tem frequência cardíaca de 110 bpm, com dor de moderada intensidade a palpação epigástrica, sem sinais de irritação abdominal. Após exame clínico, laboratorial e de imagem fez-se o diagnóstico de pancreatite aguda. Sobre pancreatite aguda, assinale a alternativa CORRETA:

Page 9: Pontifícia Universidade Católica do Paraná Processo ...static.medgrupo.com.br/static/concursos/editais/HUC/2013/GABARITOS... · A) Homem de 24 anos após uso inalatório de cocaína

Pontifícia Universidade Católica do Paraná – Programa de Residência Médica – Janeiro/ 2013 Pág. 8

A) Todos os pacientes com diagnóstico de pancreatite aguda devem ser submetidos à tomografia computadorizada de abdome sem contraste.

B) Na suspeita de necrose pancreática, deve-se solicitar como primeira opção de exame de imagem uma cintilografia.

C) Critérios de Ranson devem ser avaliados na admissão e após 48h. TGO – tansaminase oxalacética e LDH desigrogenase láctica fazem parte dos critérios de 48 horas.

D) A nutrição parenteral total NPT deve ser indicada em pacientes com pancreatite grave, não existindo outra opção de dieta para estes pacientes.

E) Segundo o Grupo de Estudos para a Classificação da Pancreatite, a pancreatite aguda grave caracteriza-se por apresentar três ou mais critérios do escore de Ranson, oito ou mais pontos na classificação de APACHE II, complicações pancreáticas ou a presença de falência orgânica.

33. Paciente feminina, 50 anos, previamente hígida, deu

entrada, no pronto-atendimento, às 21 horas, com história de dor em hipocôndrio direito de forte intensidade, de início pela manhã do mesmo dia, associado à náusea e a vômitos biliosos. Nega episódios semelhantes anteriores. Nega comorbidades associadas. Ao exame físico, a paciente mostra-se estável hemodinamicamente, anictérica, com dor à palpação abdominal em quadrante superior direito e sinal de Murphy positivo. O restante do exame físico apresenta-se sem alterações. Referente ao provável diagnóstico do enunciado acima, assinale a alternativa CORRETA:

A) A paciente referida apresenta quadro clínico compatível com cólica biliar – dor biliar, podendo ser liberada sem maiores investigações. Deverá apenas ser submetida à analgesia intra-hospitalar e receber alta com orientações dietéticas.

B) A prioridade do tratamento da colecistite aguda é o uso de antibióticos parenterais, sendo o tratamento cirúrgico uma segunda opção.

C) A tomografia de abdome sem contraste é o exame diagnóstico de escolha no caso de colecistite aguda e sempre deverá ser realizado na investigação desse quadro clínico.

D) A cintilografia das vias biliares é um excelente método para o diagnóstico do quadro clínico descrito acima.

E) O achado ecográfico mais comum que sugere inflamação aguda da vesícula biliar é uma vesícula biliar com paredes lisa e sem espessamento.

34. Um paciente masculino de 67 anos foi submetido à

correção de hérnia inguinal bilateral pela técnica de Lichtenstein. Do lado direito identificou-se um defeito herniário indireto, com anel inguinal interno sem

dilatação. Do lado esquerdo observou-se hérnia recidivada direta. Assinale a alternativa que apresenta a descrição cirúrgica deste caso de acordo com a classificação de Nyhus.

A) Lado direito: Nyhus II - Lado esquerdo: Nyhus

IVa. B) Lado esquerdo: Nyhus I - Lado direito: Nyhus

IVc. C) Lado direito: Nyhus IIIb - Lado esquerdo:

Nyhus I. D) Lado direito: Nyhus IIIa - Lado esquerdo:

Nyhus IVa. E) Lado direito: Nyhus I - Lado esquerdo: Nyhus

IIIc. 35. Em relação à síndrome compartimental abdominal,

afirma-se:

I. A definição atual de síndrome compartimental abdominal (SCA) é uma pressão intraabdominal PIA maior ou igual 60 mmHg independente da presença ou ausência de falência orgânica associada.

II. Hipertensão intraabdominal é definida como uma elevação persistente ou repetida da pressão intraabdominal maior ou igual a 12 mmHg.

III. A síndrome compartimental abdominal (SCA) é consequência de um aumento agudo da pressão intra-abdominal (PIA), que promove alterações fisiológicas adversas, devido ao acometimento de vários sistemas, podendo levar à falência orgânica e à morte, caso não ocorra uma descompressão imediata da cavidade abdominal.

IV. A mensuração da PIA é considerada como dado adicional de suporte diagnóstico, podendo ser realizada de forma indireta com a aferição da pressão vesical. Atualmente a pressão vesical tornou-se o sistema padrão de mensuração da PIA, pois a bexiga é capaz de transmitir a PIA sem contribuir com qualquer pressão adicional de sua própria musculatura.

V. A SCA deve ser pensada em todo paciente vítima de trauma abdominal grave, com sangramento intraperitoneal significativo e que foi politransfundido.

Está(ão) correta(s) APENAS a(s) afirmativa(s):

A) III. B) IV e V. C) I. D) II, III, IV e V. E) II e III.

36. Paciente de 42 anos, com diagnóstico prévio de

colecistite crônica litiásica dá entrada no pronto-atendimento com icterícia com 2 dias de evolução. Submetido à colangiorresonância pré-operatória, que evidenciou colédoco de 3,1 cm com 8 cálculos no seu interior. Do ponto de vista cirúrgico, além da colecistectomia, qual a melhor conduta para o tratamento da coledocolitíase associada?

Page 10: Pontifícia Universidade Católica do Paraná Processo ...static.medgrupo.com.br/static/concursos/editais/HUC/2013/GABARITOS... · A) Homem de 24 anos após uso inalatório de cocaína

Pontifícia Universidade Católica do Paraná – Programa de Residência Médica – Janeiro/ 2013 Pág. 9

A) Coledocotomia e coledocorrafia sobre tubo em

“T”, saindo por contraincisão. B) Anastomose biliodigestiva coledocojejunal em

Y de Roux. C) Duodenotomia e papilotomia para retirada dos

cálculos seguida de duodenorrafia. D) Apenas colecistectomia sem manipulação da

via biliar principal. E) Litrotripsia ou uso de medicamentos como por

exemplo o ácido ursodesoxicólico - ursacol associados à papilotomia.

37. Paciente 74 anos, mulher, dá entrada no

pronto-atendimento com história de dor abdominal intensa, difusa, progressiva, sem fatores de melhora, com 1 dia de evolução associada à náusea e vômitos. Nega história prévia de cirurgia abdominal. Refere tratamento inadequado para fibrilação atrial (diagnóstico desta enfermidade feito há 4 anos). Ao exame físico paciente apresenta abdome flácido, distensão leve, com dor a palpação superficial e profunda, sem sinais de irritação peritoneal. Exames laboratoriais: hemograma com 15.000 leucócitos, 11 % de bastões, LDH 953 U/dl, Cálcio sérico de 6. Rotina de abdome agudo com sinais de edema de parede de alça intestinal. Em relação ao quadro acima, assinale o diagnóstico mais provável:

A) Volvo de sigmoide. B) Pancreatite necro-hemorrágica. C) Isquemia mesentérica. D) Obstrução intestinal. E) Úlcera péptica perfurada.

38. Em relação aos tumores de tireoide, afirma-se:

I. O adenocarcinoma papilar responde por 85% dos cânceres da glândula tireoide e se apresenta normalmente como um nódulo solitário no início da vida adulta para então se disseminar principalmente por via hematogênica

II. O adenocarcinoma folicular responde por aproximadamente 10% dos tumores malignos da tireoide. O carcinoma de células de Hurthle é uma variante deste tipo de neoplasia.

III. O carcinoma medular contém amiloide; é um tumor sólido, duro e nodular, que não capta radioiodo e secreta calcitonina.

IV. A linfadenopatia cervical e metástases pulmonares são comuns no carcinoma indiferencido de tireoide.

V. Os depósitos metastáticos de carcinoma folicular e papilar devem ser tratados com 131 I, após tireoidectomia total ou ablação da tireoide com iodo radioativo.

A) Somente I, III e IV são verdadeiras. B) Somente II e IV são verdadeiras. C) Somente V é verdadeira. D) Somente II é verdadeira. E) Somente II, III, IV e V são verdadeiras.

39. Nos casos de tratamento cirúrgico das afecções do baço, é CORRETO afirmar:

A) A maior parte dos abscessos piogênicos do

baço se formam por contiguidade das lesões no próprio baço e apresentam baixa taxa de mortalidade, em torno de 5%.

B) A presença de esplenomegalia no exame físico praticamente confirma o diagnóstico de púrpura trombocitopênica idiopática.

C) O tratamento de escolha para os cistos parasitários equinocócicos do baço é clínico, sendo a esplenectomia raramente realizada nestes pacientes.

D) Streptococcus pneumoniae, Haemophilus influenzae e meningococos são os micro-organismo mais frequentemente envolvidos nos casos de infecção fulminante pós-esplenectomia.

E) Púrpura trombocitopênica idiopática e esferocitose são contraindicações absolutas para realização de esplenectomia videolaparoscópica.

40. Em relação à terapia nutricional, assinale a

alternativa CORRETA:

A) A bioimpedância tem sido utilizada na determinação da composição corporal e baseia-se na aplicação de corrente elétrica pelo corpo conseguindo determinar o percentual de massa magra e massa gorda.

B) Nutrição parenteral deve ser preferida sempre, mesmo naqueles pacientes com o trato gastrointestinal íntegro por apresentar menores taxas de complicações metabólicas

C) Pacientes desnutridos apresentam as mesmas incidências de complicações infecciosas que pacientes nutridos.

D) O uso da infusão da dieta enteral em bolus previne o risco de broncoaspiração quando comparada à infusão contínua.

E) Para terapia nutricional parenteral por veia periférica a dieta deve ser mais hiperosmolar para evitar o risco de flebite.

41. Paciente de s6 anos, com história referida de febre

(até 40ºC) e odinofagia há uma semana. Foi atendida na Unidade Básica nos primeiros dias do quadro e recebeu penicilina benzatina e paracetamol, porém não teve melhora clínica. Na evolução, a mãe da paciente buscou atendimento em um pronto-socorro onde lhe foi prescrito cefalexina, porém após 48 horas de uso da medicação, ainda persiste sintomática. Hoje, ao exame apresenta: bom estado geral, febril, (T= 39,5ºC), hipoativa, porém, colaborativa, com adenomegalia cervical de 1,6 cm de diâmetro, dolorosa à palpação; orofaringe com hiperemia e hipertrofia de amídalas, além de hiperemia conjuntival. O restante do exame físico não apresenta alterações. A melhor conduta a ser tomada é, neste caso:

A) Prescrever amoxicilina com ácido clavulânico +

dipirona.

Page 11: Pontifícia Universidade Católica do Paraná Processo ...static.medgrupo.com.br/static/concursos/editais/HUC/2013/GABARITOS... · A) Homem de 24 anos após uso inalatório de cocaína

Pontifícia Universidade Católica do Paraná – Programa de Residência Médica – Janeiro/ 2013 Pág. 10

B) Orientar sobre a evolução e transmissibilidade das infecções respiratórias e conjuntivites virais.

C) Prescrever dexclorfeniramina + prednisolona (1 mg/Kg/dia) e orientar retorno se sinais de alerta.

D) Prescrever ceftriaxona + ibuprofeno e orientar para uso de poucas roupas e compressas frias.

E) Encaminhá-la para um centro de referência.

42. Escolar de seis anos, trazido pela mãe por apresentar perda de urina durante o sono, pelo menos 3-4 vezes por semana. A mãe refere que a criança nunca controlou a bexiga definitivamente à noite, mas não perde urina durante o dia. Traz um parcial de urina e urocultura normais. Nega história mórbida pregressa ou outras queixas atuais. Tem bom desempenho na escola. A família é bem estruturada, mas os pais são separados. A melhor conduta a ser tomada no momento será:

A) Encaminhar para um serviço de psicologia. B) Solicitar estudo urodinâmico. C) Prescrever oxibutinina. D) Utilizar métodos comportamentais como

alarmes e reforço positivo. E) Orientar restrição de líquidos a partir das 16

horas.

43. Paciente de 9 anos em tratamento com metilfenidato para transtorno de déficit de atenção e hiperatividade vem apresentando boa melhora nas atividades acadêmicas, porém a mãe preocupa-se com as consequências de se fazer uso de uma medicação controlada em idade tão precoce. Ao citar-lhe os riscos, você CORRETAMENTE lhe dirá que o uso do medicamento, mesmo na posologia poderá causar:

A) Dependência, tolerância progressiva e

síndrome de abstinência ao medicamento. B) Desenvolvimento de comportamento agressivo

e transtorno de conduta. C) Desconforto abdominal, disforia, perda de peso

e dificuldade para dormir. D) Propensão a abuso de substâncias na

adolescência e vida adulta. E) Síndrome de Tourette, convulsões e outros

transtornos de ansiedade em alguns pacientes.

44. Das alternativas abaixo, qual a melhor indicação para introduzir suplementação medicamentosa de Ferro elementar em dose profilática (1 mg/Kg/dia) à alimentação de um lactente no primeiro ano de vida?

A) Todos os lactentes com idade superior a 4 m

que estejam recebendo apenas fórmula infantil. B) Todos os lactentes em aleitamento materno

cujas mães têm diagnóstico de anemia ferropriva.

C) Todos os lactentes com idade superior a 6 m que estejam em aleitamento materno.

D) Todos os lactentes que apresentem valores de hemoglobina sérica < 11,0 g/dl.

E) Todos os lactentes de baixo peso que recebam fórmula infantil à base de soja.

45. Com relação à febre é CORRETO dizer:

A) O uso de resfriamento corporal com compressas de álcool pode causar irritação da pele e intoxicação alcoólica em crianças.

B) É consequência de um desequilíbrio na produção e dissipação do calor.

C) A ocorrência de temperaturas superiores a 39°C tornam o diagnóstico de etiologia viral pouco provável.

D) Uma criança de dois anos com febre sem foco há 24 horas e temperatura documentada > 38,6°C só deverá ser submetida à investigação laboratorial se estiver toxêmica.

E) As temperaturas superiores a 40,0 ° C devem ser prontamente tratadas com banho de água fria e antitérmicos devido aos riscos metabólicos que elas representam.

46. Lactente de 40 dias, em aleitamento exclusivo, vem

apresentando desenvolvimento adequado para a idade, porém, a família preocupa-se com episódios intermitentes de choro intenso e ausência de evacuações há 5 dias; a mãe nega doenças pregressas ou complicações puerperais. Ao exame físico apresenta icterícia leve, atingindo apenas a face e o pescoço (está bem melhor – sic) e o restante não apresenta alterações. Dado este quadro, a melhor conduta a ser tomada é:

A) Iniciar dimeticona e lactulona, uma vez ao dia,

até restabelecer o trânsito intestinal normal. B) Introduzir suco de laranja, uma vez ao dia para

melhorar a função intestinal do paciente. C) Prescrever um supositório de glicerina infantil a

cada 4 horas até voltar a evacuar. D) Encaminhar para gastroenterologista pediátrico

para realização de exames de imagem. E) Orientar a mãe que essa é uma condição

fisiológica e que deverá melhorar mesmo sem utilizar medicações.

47. Pré-escolar de 2 anos iniciou quadro de coriza e

congestão nasal que evoluiu em 24 horas para tosse rouca, disfonia e febre. A sua melhor terapêutica no pronto atendimento neste momento será:

A) Alta com dexametasona V.O. e sintomáticos. B) Inalação no pronto-atendimento com

adrenalina (1:1000) diluída em SF 0,9% . C) Inalação no pronto-atendimento com S.F. 0,9%

gelado e sintomáticos. D) Alta com inalação contendo S.F. 0,9%,

fenoterol e brometo de ipratrópio. E) Internamento com suporte respiratório.

48. RN de seis dias é trazido à Unidade de Saúde por apresentar icterícia. Está em aleitamento exclusivo e nasceu de cesárea por rotura prematura de membranas, sem trabalho de parto (sic). Idade Gestacional estimada em 37 semanas, não apresentando intercorrências durante o período em que esteve na maternidade. Peso ao nascer: 2800g. Peso na alta: 2720 g. Peso atual: 2750 g. Não há referência à icterícia na Carteira do Recém-Nascido,

Page 12: Pontifícia Universidade Católica do Paraná Processo ...static.medgrupo.com.br/static/concursos/editais/HUC/2013/GABARITOS... · A) Homem de 24 anos após uso inalatório de cocaína

Pontifícia Universidade Católica do Paraná – Programa de Residência Médica – Janeiro/ 2013 Pág. 11

e a mãe confirma que o “amarelão” surgiu há um dia. As fezes têm aspecto e cor normais. Ao exame apresenta-se: ativo, ictérico até a região das coxas, reflexos presentes e simétricos, sem alterações na avaliação segmentar. Diante deste caso, a melhor conduta será:

A) Orientar a mãe do RN de que se trata de

icterícia fisiológica e que ela irá desaparecer nos próximos dias.

B) Encaminhar para um serviço de referência para avaliação e conduta.

C) Interromper o aleitamento materno por 48-72 horas para confirmar o diagnóstico de icterícia do leite materno.

D) Solicitar, tipagem sanguínea, reticulócitos, bilirrubinas totais e frações e sorologias para toxoplasmose, citomegalovírus, hepatite B e hepatite C e tomar a conduta conforme o resultado dos exames.

E) Solicitar apenas ecografia para descartar malformação de vias biliares.

49. Pré-escolar de 3 anos, previamente hígido,

frequentador de creche, vem apresentando há 1 dia febre, tosse e prostração. A mãe refere que a criança já recebeu vários antibióticos neste ano, sendo o último há 2 meses. Ao exame apresenta-se em bom estado geral, porém com dispneia (FR=40 ipm) e estertoração em base dir. A partir destes dados, é recomendado que você utilize o seguinte esquema terapêutico:

A) Iniciar sulfametoxazol com trimetroprim (8 12

mg/ Kg/dia). B) Iniciar amoxicilina com clavulanato (25-50

mg/Kg/dia). C) Iniciar ceftriaxona (50-100 mg/Kg/dia). D) Iniciar amoxicilina (80-90 mg/Kg/dia). E) Iniciar levofloxacina (25 - 50 mg/Kg/dia).

50. Lactente de 12 meses é trazida à emergência com dor em cotovelo esq. devido a uma queda do sofá. Os pais dizem que haviam saído de casa e deixaram a irmã de 6 anos cuidando dela, que se distraiu assistindo à televisão e não viu que a bebê estava sobre o sofá (sic). Ao exame físico a paciente parece chorosa, irritada e com edema em cotovelo esq. O Rx mostra fratura em braço esq. Observando o seu prontuário, observa-se o registro de mais atendimentos de urgência nos últimos meses, sendo um deles por TCE com fratura e outro por queimadura. Dado este quadro, qual a conduta a ser tomada?

A) Imobilizar o braço da criança, prescrever

analgésicos e encaminhá-la para retornar no dia seguinte no ambulatório de ortopedia.

B) Prescrever analgésico, imobilizar o membro acometido e solicitar cintilografia para identificar possíveis áreas de lesão no resto do corpo.

C) Prescrever analgésico e realizar internamento imediato.

D) Orientar a família sobre prevenção de acidentes domésticos, liberar a paciente com analgésicos e suplementação de cálcio, além de solicitar encaminhamento para ortopedia pela Unidade de Saúde de origem.

E) Descrever detalhadamente suas impressões clínicas no prontuário, prescrever analgésico e encaminhar para geneticista para descartar possibilidade de osteogênese imperfeita.

51. Na abordagem do paciente com doença celíaca pode- se afirmar que:

A) Diante da suspeita clínica deve-se suspender imediatamente a ingestão de glúten até a confirmação do diagnóstico para evitar prejuízo nutricional.

B) Os exames sorológicos: anticorpo IgG antigliadina, anticorpo IgG antiendomísio e anticorpo IgM anti-transglutaminase tecidual são os melhores testes diagnósticos.

C) A biópsia de intestino delgado deve ser necessariamente realizada antes da retirada do glúten da dieta.

D) Os exames sorológicos: anticorpo IgA antiglúten, Anticorpo IgA antiendomísio e RAST para glutenina são úteis, mas o diagnóstico só é confirmado através de teste de HLA DQ2.

E) A susceptibilidade à doença está fortemente ligada à expressão de moléculas HLA DQ1 na superfície das células apresentadoras de antígenos.

52. A tetralogia de Fallot é uma cardiopatia congênita cianótica que faz manifestação clínica na infância. Como se classifica esta anomalia?

A) Cianótica, normofluxo pulmonar com hipertensão venocapilar pulmonar.

B) Acianótica, normofluxo pulmonar com normotensão venocapilar pulmonar.

C) Cianótica, hiperfluxo pulmonar com hipertensão venocapilar pulmonar.

D) Cianótica, hipofluxo pulmonar com normotensão venocapilar pulmonar.

E) Acianótica, hiperfluxo pulmonar com hipertensão venocapilar pulmonar.

53. Qual é a síndrome pediátrica febril caracterizada por

febre periódica, estomatite aftosa, faringite e adenite?

A) Síndrome da Mononucleose Infecciosa. B) Síndrome de Marchall ( PFAPA ). C) Síndrome de Hipergamaglobulinemia D. D) Síndrome da Febre do Mediterrâneo. E) E) Síndrome Neutropênica Cíclica.

54. A anemia ferropriva é a carência nutricional mais

prevalente no mundo, acarretando prejuízos, a curto e a longo prazo, no desenvolvimento neuropsicomotor, na aprendizagem, além de

Page 13: Pontifícia Universidade Católica do Paraná Processo ...static.medgrupo.com.br/static/concursos/editais/HUC/2013/GABARITOS... · A) Homem de 24 anos após uso inalatório de cocaína

Pontifícia Universidade Católica do Paraná – Programa de Residência Médica – Janeiro/ 2013 Pág. 12

comprometimento na resposta do sistema imunológico. Sobre o do Ferro, é CORRETO afirmar:

A) Não é necessária a suplementação de Ferro nos primeiros 6 meses de vida no lactente a termo, adequado para a idade gestacional e recebendo leite materno exclusivo.

B) O Ferro não heme encontrado na carne é mais bem absorvido que o Ferro heme.

C) Apresenta uma boa quantidade de Ferro de alta biodisponibilidade alimentos como a gema de ovo, feijão, lentilha, soja e vegetal verde escuro.

D) Aumenta a absorção do Ferro o leite, chá, mate, café e cereais (fitatos).

E) Diminuem a assimilação do Ferro: frutose, citrato, ácido ascórbico e alguns aminoácidos da carne.

55. Analise as afirmações abaixo:

I. O reflexo cutâneo plantar de extensão, ou dos pododáctilos presente no recém-nato, persiste normalmente até os 12 meses.

II. O reflexo de Landau I surge por volta dos 2 – 3 meses de vida.

III. O reflexo tônico cervical assimétrico ou de Magnus de Kleijn desaparece normalmente após o 3-4º. mês de vida.

IV. O reflexo de olhos de boneca normalmente desaparece em vigília após os 2-3º. mês de vida.

Está(ão) correta(s) APENAS :

A) II, III e IV. B) I, III e IV. C) III e IV. D) I e II. E) I, II, III e IV.

56. Sobre a impossibilidade de aleitamento materno em

lactente com alergia à proteína do leite de vaca, é CORRETO afirmar:

A) As fórmulas com proteína extensamente

hidrolisada (ou as fórmulas de aminoácidos quando a hidrolisada não for tolerada) são indicadas no tratamento a alergia à proteína do leite de vaca não mediadas por IgE.

B) As fórmulas sem lactose são utilizadas em pacientes com alergiaàa proteína do leite de vaca, pois, como o próprio nome diz, não têm lactose na sua composição.

C) As fórmulas com proteína extensamente hidrolisada são a segunda escolha em pacientes com alergia à proteína do leite de vaca em crianças abaixo de 6 meses.

D) As fórmulas com proteína de soja podem ser utilizadas na intolerância à lactose e nas alergias mediadas por IgE e não mediadas por IgE. Uma das vantagens no seu uso é a possibilidade de ser oferecida desde o primeiro mês de vida mesmo em prematuros.

E) Na alergia alimentar as fórmulas com proteína de soja podem ser utilizadas em menores de 6 meses de vida.

57. Sobre icterícia neonatal, assinale a alternativa CORRETA:

A) A presença de icterícia antes de 24 horas de

vida deve ser investigada quando os níveis encontram-se acima de 12 mg/dl.

B) Na doença hemolítica por incompatibilidade ABO, o RN é A ou B e a mãe é O, ocorrendo o desenvolvimento da doença na segunda gestação.

C) Em prematuros com hiperbilirrubinemia e anemia, deve ser investigada a deficiência de G6PD.

D) A alta hospitalar antes de 48 horas após o parto não é fator de risco para a icterícia pela “falta do leite materno”.

E) A encefalopatia bilirrubínica caracteriza-se inicialmente por hipertonia, opistótono, choro com tonalidade aguda, progredindo em 3 a 4 dias para hipotonia, sucção débil e convulsões. Após um período de relativa melhora, pode evoluir com deficiência auditiva, paralisia cerebral espástica, distúrbios da deglutição.

58. Em relação à asfixia perinatal, é CORRETO afirmar:

A) Convulsões podem ocorrer nas primeiras horas após o nascimento e benzodiazepínico é usado como droga de escolha para tratá-las.

B) A síndrome de secreção inapropriada do hormônio antidiurético, frequentemente observada por 3 a 4 dias após o insulto hipóxico-isquêmico, manifesta-se por hiponatremia e hiperosmolaridade.

C) Nenhum recém-nascido sofre agressão asfíxica perinatal suficientemente severa para provocar lesão cerebral permanente sem que outros órgãos, como rins, coração, intestinos, pulmões e/ou fígado, sejam igualmente afetados de forma grave.

D) Minimizar o uso de bicarbonato de sódio e sobrecarga hídrica, corrigir distúrbios metabólicos e hidroeletrolíticos, principalmente hipoglicemia e hipocalcemia, promover hipotermia e garantir pressão arterial adequada são medidas que comprovadamente melhoram o prognóstico neurológico.

E) O diagnóstico é feito baseado no índice de Apgar.

59. Segundo o estadiamento puberal estabelecido por Tanner, uma menina com mamas (M) com tecido glandular apenas subaerolar e pelos pubianos (P) grossos e encaracolados, com distribuição em vulva e região suprapúbica, em pequena quantidade, deve ser classificada como:

A) M1 – P2. B) M2 – P1. C) M3 – P2.

Page 14: Pontifícia Universidade Católica do Paraná Processo ...static.medgrupo.com.br/static/concursos/editais/HUC/2013/GABARITOS... · A) Homem de 24 anos após uso inalatório de cocaína

Pontifícia Universidade Católica do Paraná – Programa de Residência Médica – Janeiro/ 2013 Pág. 13

D) M2 – P3. E) M3 – P3.

60. Qual das queixas relatadas abaixo são

características da anamnese de uma criança com asma? Sobre o tema, avalie as assertivas, identificando-as com V (verdadeira) e F (falsa).

1. Os episódios de sibilância são contínuos e muito duradores, sem fatores de melhora. 2. A tosse é principalmente produtiva e ocorre durante todo o dia. 3. Falta de ar aos exercícios está presente raramente, somente nos casos de asma classificados como grave persistente. 4. Uso de anti-inflamatórios está entre os desencadeantes mais comuns das exacerbações da asma na infância. 5. As crises de tosse ocorrem principalmente á noite e ao amanhecer, sendo este um dos principais sintomas dos períodos intercríticos.

Marque a sequência CORRETA:

A) F, F, F, F, F B) F, F, F, F, V C) V, V, F, F, F D) V, V, V, V, V E) V, F, F, V, F

61. Durante o período gestacional, na realização do pré-

natal, vários exames complementares são solicitados às gestantes, alguns deles mais de uma vez durante a gestação. A solicitação de hemograma para o diagnóstico de anemia é importante, pois, entre diferentes fatores de risco, está a restrição de crescimento intrauterino (RCIU). Nessa perspectiva, as seguintes situações podem ser consideradas medidas de prevenção, respectivamente:

I. Solicitação de hemograma para diagnóstico de

anemia no primeiro trimestre da gestação. II. Tendo em vista a prevenção do RCIU, o

tratamento de anemia materna no período gestacional, medida preventiva com relação ao feto.

A) (I) Primária de grupo de risco e (II) Secundária. B) (I) Secundária e (II) Terciária. C) (I) Primária e (II) Primária. D) (I) Secundária e (II) Secundária. E) (I) Secundária e (II) Primária.

62. A histerectomia é uma das cirurgias mais realizadas

em todo o mundo, sendo indicada para tratamento de doenças benignas e malignas. Sobre histerectomias e suas possíveis complicações, é CORRETO afirmar:

A) Durante a cirurgia a visualização e/ou palpação

do ureter no espaço pararretal diminui a possibilidade de lesões dessa estrutura.

B) Dentre as complicações intraoperatórias, a laceração de bexiga pode ser facilmente

reparada, principalmente quando há envolvimento do trígono vesical.

C) As taxas de complicações nas histerectomias são relativamente altas, mesmo nas cirurgias de processos benignos.

D) Na histerectomia subtotal, ou supracervical, o risco de lesões de órgãos vizinhos é maior que na histerectomia total, sendo, portanto, menos utilizada.

E) Pacientes com miomas assintomáticos devem ser submetidas à histerectomia devido ao risco elevado de degeneração sarcomatosa.

63. Com relação ao rastreamento do câncer de colo

uterino, é CORRETO afirmar segundo as recomendações do Ministério da Saúde:

A) A presença de epitélio metaplásico na citologia

oncótica indica possibilidade de câncer. B) É recomendação do Ministério da Saúde

indicar colposcopia se a citologia demonstrar metaplasia imatura.

C) O início da coleta do Papanicolaou deve ser aos 25 anos para as mulheres que já tiveram atividade sexual.

D) Mulheres submetidas à histerectomia total por doença benigna devem continuar coletando citologia anualmente, mesmo que tenham todos os exames anteriores normais.

E) Gestantes não devem coletar citopatológico devido ao risco de abortamento.

64. Com relação aos ligamentos da pelve óssea, é

CORRETO afirmar:

A) O ligamento de Cooper estende-se da espinha isquiática até a parte lateral do sacro.

B) A artéria uterina é encontrada entre os ligamentos sacroespinhal e sacrotuberoso.

C) O ligamento sacrotuberoso situa-se anterior ao nervo pudendo e aos vasos pudendos internos.

D) O ligamento sacrotuberoso funde-se lateralmente ao ligamento espinhal.

E) O ligamento pectíneo é usado com frequência nos procedimentos de suspensão da bexiga.

65. De acordo com a fisiologia reprodutiva, é CORRETO afirmar:

A) O núcleo arqueado do hipotálamo libera GnRH

na circulação portal de maneira pulsátil. B) Os pulsos de GnRH de menor frequência

favorecem a secreção de LH e os de maior frequência favorecem a secreção de FSH.

C) Receptores de LH são característicos das células da granulosa.

D) O folículo dominante tem a mesma quantidade de receptores de FSH que os demais folículos.

E) Inibina B, secretada pelas células da teca, estimulam a secreção de FSH.

66. Paciente que se apresenta com transtorno disfórico pré-menstrual terá como principal tratamento:

Page 15: Pontifícia Universidade Católica do Paraná Processo ...static.medgrupo.com.br/static/concursos/editais/HUC/2013/GABARITOS... · A) Homem de 24 anos após uso inalatório de cocaína

Pontifícia Universidade Católica do Paraná – Programa de Residência Médica – Janeiro/ 2013 Pág. 14

A) Restrições dietéticas. B) Progesterona isolada. C) Contraceptivos hormonais orais combinados. D) Inibidores seletivos da recaptação da

serotonina. E) Vitaminas e minerais específicos.

67. Sobre o diagnóstico e tratamento da dismenorreia é

possível afirmar que:

A) A dismenorreia primária sempre necessita de exames complementares para seu diagnóstico.

B) Somente os anti-inflamatórios inibidores seletivos da COX-2 são boas opções para o alívio da dor.

C) A dosagem de CA-125 é padrão-ouro para o diagnóstico de endometriose.

D) Mulheres que não têm boa resposta ao tratamento com AINE e/ou anticoncepcional hormonal devem ser reavaliadas quanto à possibilidade de causas secundárias de dismenorreia.

E) Miomas são causas de dismenorreia primária.

68. Na avaliação dos sangramento uterinos anormais é importante a avaliação dos diagnósticos diferenciais. Sobre eles está CORRETO:

A) Em meninas pós-menarca as principais causas

incluem sangramento anovulatório, coagulopatias, infecção e complicações da gravidez.

B) Cervicite, endometrite e salpingite não causam sangramentos anormais.

C) Alterações da função tireoidiana não causam alterações dos ciclos menstruais.

D) Contraceptivos hormonais são utilizados para tratamento e não causam sangramentos anormais.

E) Em crianças nos sangramentos genitais está descartada a possibilidade da presença de tumores.

69. Galactorreia refere-se à secreção mamária láctea

que não é fisiológica. Sobre essa condição clínica assinale a CORRETA:

A) A galactorreia é sempre bilateral. B) O aumento do nível de dopamina eleva o nível

de prolactina e leva à galactorreia. C) A produção de níveis elevados de prolactina

tem sempre origem na hipófise. D) Todas as pacientes com galactorreia entram

em amenorreia. E) Muitas drogas podem estar associadas à

galactorreia, mas nesses casos os níveis de prolactina são menores que 100 mg/ml.

70. O conhecimento das contraindicações dos métodos contraceptivos auxilia na escolha de método contraceptivo mais adequado. Considerando os anticoncepcionais hormonais só com progestagênio, qual opção representa contra-indicação absoluta ao seu uso (categoria 4 da OMS)?

A) Trombose venosa profunda/embolia pulmonar aguda.

B) Câncer de mama atual. C) Pressão arterial sistólica >160 ou diastólica >

100. D) Lúpus eritematoso sistêmico. E) Diabetes com nefropatia.

71. Paciente de 50 anos vem à consulta para solicitação

de mamografia. No exame clínico da mamãe detectado nódulo em QSL de mama esquerda. Ao retornar com o resultado da mamografia, verifica-se que a mamografia tinha o laudo BIRADS 2 com calcificações. Qual a conduta adequada para esta paciente?

A) Solicitar mamografia em 1 ano. B) Repetir mamografia em 2 anos. C) Solicitar ultrassonografia para avaliar o nódulo

que não foi verificado na mamografia. D) Repetir mamografia em 6 meses. E) Solicitar biópsia das calcificações.

72. Sobre as alterações endócrinas na gestação é

CORRETO afirmar:

A) Existe aumento da liberação de GnRH, LH e FSH.

B) O fígado sintetiza menor quantidade de globulina transportadora de T4.

C) A maior parte do hormônio lactogênico placentário passa para a circulação fetal pelo cordão umbilical.

D) O nível de cortisol fetal é maior que o nível materno.

E) Há aumento da massa da hipófise com hipertrofia dos lactótrofos.

73. Sobre as repercussões da gestação no organismo materno é CORRETO:

A) O volume sanguíneo materno aumenta até

50% comparado ao volume pré-gestacional. B) O aumento do volume sanguíneo é menor nas

gestações múltiplas. C) A viscosidade plasmática está aumentada,

aumentado o trabalho cardíaco. D) O volume eritrocitário absoluto apresenta

diminuição, havendo hemodiluição. E) Os níveis plaquetários estão discretamente

aumentados e os de leucócitos diminuídos.

74. É CORRETO afirmar no atendimento ao trabalho de parto:

A) Na cardiotocografia intraparto as

desacelerações precoces indicam sofrimento fetal e necessidade de parto imediato.

B) Durante o segundo período do trabalho de parto, deve-se fazer a ausculta dos batimentos cardíacos fetais a cada hora.

C) Manobra de Kristeller é ótima opção para auxiliar no desprendimento do polo cefálico.

Page 16: Pontifícia Universidade Católica do Paraná Processo ...static.medgrupo.com.br/static/concursos/editais/HUC/2013/GABARITOS... · A) Homem de 24 anos após uso inalatório de cocaína

Pontifícia Universidade Católica do Paraná – Programa de Residência Médica – Janeiro/ 2013 Pág. 15

D) O prolapso de cordão pode estar relacionado à amniotomia e indica a realização de cesariana imediata.

E) O desprendimento do polo cefálico deve ser o mais rápido possível.

75. Sobre o puerpério é CORRETO afirmar:

A) Após o parto recomenda-se ao máximo o repouso no leito.

B) Imediatamente após o parto a hipercoagulabilidade se desfaz.

C) Vacinação para rubéola é contraindicada enquanto houver aleitamento materno.

D) DIU de cobre não é boa opção para a mulher que amamenta.

E) No puerpério imediato ocorre redução do volume uterino com angulação das artérias uterina e ovariana, causando diminuição da perfusão uterina.

76. Considerando as formas clínicas do abortamento, assinale a CORRETA:

A) Abortamento em curso apresenta sangramento

intermitente, volume uterino menor que o esperado para a idade gestacional e ultrassonografia mostra cavidade uterina sem conteúdo.

B) Ameaça de abortamento caracteriza-se por sangramento vaginal pequeno com ou sem dor abdominal em hipogástrio, com colo uterino fechado e tamanho uterino compatível com o atraso menstrual.

C) Abortamento completo: sangramento moderado a intenso, colo uterino pérvio e ultrassonografia mostra descolamento ovular com saco gestacional baixo.

D) Abortamento incompleto: colo fechado, produto conceptual sem batimentos cardíacos.

E) Abortamento habitual: dois episódios de abortamento, não necessariamente consecutivos.

77. Assinale a alternativa CORRETA com relação à gestação ectópica:

A) Os níveis de beta HCG crescem no mesmo

ritmo que na gestação tópica. B) Pode ser intersticial, em cicatriz de cesárea,

cervical, ou em outros locais fora da cavidade corporal do útero.

C) Para tratamento conservador basta o desejo da paciente.

D) Ultrassonografia com Doppler não traz benefício ao diagnóstico.

E) A localização ovariana é a mais frequente.

78. Na doença trofoblástica gestacional, é CORRETO afirmar que:

A) A mola completa apresenta embrião. B) A mola parcial tem seus cromossomos

somente de origem paterna.

C) O tratamento de escolha é esvaziamento uterino e radioterapia.

D) A mola completa tem evolução mais benigna que a parcial.

E) O quadro clínico se apresenta com sangramento vaginal e útero maior que o esperado para a idade gestacional.

79. A prematuridade é causa importante de mortalidade e morbidade neonatal. Sobre este tema, assinale a alternativa CORRETA:

A) Interrupções prematuras da gravidez por

cesáreas eletivas decorrentes de erro na determinação da idade gestacional podem causar prematuridade iatrogênica.

B) Dentre os miomas, os subserosos são os com maior potencial de levar à prematuridade.

C) A prevenção primária tem facilidade de ser realizada na prática.

D) A tocólise estará bem indicada na placenta prévia sangrante.

E) Cesariana está indicada para todos os fetos prematuros.

80. Gestante de 22 anos com sangue A negativo e marido com sangue B positivo vem para orientação pré-natal. Assinale a conduta CORRETA:

A) Se na rotina do primeiro trimestre o coombs

indireto for reagente, administrar imediatamente imunoglobulina anti-Rh.

B) Se o recém-nato tiver sangue Rh negativo, administrar imunoglobulina na mãe após o parto.

C) Se com 28 semanas de gestação o coombs indireto for não reagente, administrar imunoglobulina anti-Rh.

D) Após a imunoglobulina anti-Rh com 28 semanas, deve-se fazer coombs indireto seriado.

E) Não há necessidade de qualquer cuidado se esta for a primeira gestação.

81. Sobre doenças sexualmente transmissíveis e

gestação, é CORRETO afirmar:

A) Paciente que se apresente com cancro duro deve apresentar VDRL positivo se estiver com sífilis.

B) Cultura em ágar Thayer-Martin é o padrão-ouro para diagnóstico de cervicite por Chlamydia trachomatis.

C) Paciente com corrimento vaginal bolhoso e fétido com colo apresentando lesão de aspecto tigroide ao teste de Schiller não necessita tratamento se assintomática.

D) Para sífilis latente tardia ou de duração indeterminada, o tratamento indicado é com penicilina benzatina 2.400.000 UI por semana por 3 semanas.

E) Na cervicite por gonococo a gestante tem como primeira opção de tratamento dose única de ciprofloxacino 500 mg por via oral.

Page 17: Pontifícia Universidade Católica do Paraná Processo ...static.medgrupo.com.br/static/concursos/editais/HUC/2013/GABARITOS... · A) Homem de 24 anos após uso inalatório de cocaína

Pontifícia Universidade Católica do Paraná – Programa de Residência Médica – Janeiro/ 2013 Pág. 16

82. Para que um Sistema de Saúde público seja eficaz e de qualidade, é fundamental que tenha profissionais capacitados e dedicados para a assistência e cuidado da população; e uma gestão comprometida em garantir toda infraestrutura e fluxos necessários para esse bom andamento. Na estrutura do SUS brasileiro, a Atenção Primária à Saúde (APS) é a orientadora da rede de atenção. Sobre a APS no Brasil, é CORRETO afirmar:

A) A longitudinalidade do cuidado pressupõe o ajuste a condutas quando necessário, evitando a perda de referências e diminuindo os riscos de iatrogenia decorrentes do desconhecimento das histórias de vida e da coordenação do cuidado.

B) A Estratégia Saúde da Família é a única forma de APS reconhecida pelo Ministério da Saúde por apresentar um território geográfico delimitado que possibilita a vigilância em saúde, a equidade e a universalidade.

C) Dentre os princípios do SUS, a integralidade não está presente na APS, pois são necessárias tecnologias duras e a cooperação de especialistas para que esse princípio seja aplicado na sua totalidade.

D) O estabelecimento de mecanismos que assegurem acessibilidade e acolhimento pressupõe uma lógica em que a unidade de saúde deva receber e ouvir todas as pessoas que procuram os seus serviços, de modo universal e no momento e da forma que o usuário desejar.

E) A APS caracteriza-se por um conjunto de ações de saúde, estritamente no âmbito coletivo, que abrange a promoção e a proteção da saúde, a prevenção de agravos, o diagnóstico, o tratamento, a reabilitação, a redução de danos e a manutenção da saúde.

83. O Sr. José, 65 anos, hipertenso, diabético, obeso e tabagista chega à Unidade de Saúde às 10h da manhã de uma quarta-feira queixando-se de fortes dores precordiais. É avaliado pela enfermeira da Unidade e em seguida pelo médico, que, após examiná-lo, decide imediatamente encaminhá-lo para o CMUM (Centro Municipal de Urgências Médicas). O paciente passa três dias em observação no CMUM e é encaminhado para internamento no setor de cardiologia do Hospital Santa Casa de Misericórdia de Curitiba. É submetido a uma revascularização do miocárdio e, após a alta, fica vinculado ao ambulatório de cardiologia da mesma instituição e volta a ter seu acompanhamento na Unidade de Saúde de referência, passando a participar das reuniões do grupo de hipertensos mensalmente na Unidade. Pergunta-se: qual princípio do SUS foi contemplado na descrição acima?

A) Universalidade. B) Intersetorialidade. C) Equidade. D) Integralidade. E) Controle social.

84. Em 19 de setembro de 1990 a Lei 8080/90 foi sancionada pelo então Presidente da República para regulamentar em todo o território nacional as ações e serviços do Sistema Único de Saúde (SUS). Assinale a alternativa INCORRETA relacionada às determinações desta lei:

A) A saúde é um direito fundamental do ser

humano, devendo o Estado prover as condições indispensáveis ao seu pleno exercício.

B) O SUS tem como campo de atuação a participação em estudos, pesquisas, avaliação e controle dos riscos e agravos potenciais a saúde existentes no processo de trabalho.

C) É preconizada a igualdade da assistência à saúde, sem preconceitos ou privilégios de qualquer espécie.

D) A iniciativa privada poderá participar do Sistema Único de Saúde - SUS - em caráter complementar.

E) As ações e serviços de saúde serão organizados de forma regionalizada e hierarquizada em níveis de complexidade decrescente, priorizando os níveis de maior complexidade.

85. Pelo conceito ampliado de saúde proposto pela OMS, não é suficiente que o médico detenha-se somente ao tratamento doenças. É necessária uma visão ampliada que garanta também a promoção, a prevenção e a reabilitação. Diante dessa necessidade, assinale a afirmativa CORRETA sobre os conceitos de prevenção de doença.

A) Em uma consulta médica, é possível realizar

as seguintes ações de prevenção: tratar angina (prevenção secundária), perguntar sobre hábito de fumo (prevenção primária), perguntar se ela realizou o exame papanicolau (prevenção primária) e ajudar a superar suas ansiedades (prevenção terciária).

B) A prevenção primária é caracterizada por garantir a remoção da causa de um problema de saúde em um indivíduo assim que ele se manifestou.

C) Quando são realizadas ações para reduzir os efeitos crônicos de um problema de saúde focando na redução de prejuízo funcional está sendo realizada a prevenção terciária.

D) Considera-se que na prevenção secundária a causa já está instalada, mas a doença não está ativa, portanto é possível realizar uma ação individual que permita que este indivíduo não tenha outra doença.

E) A prevenção quartenária é realizada exclusivamente no indivíduo, pois visa diminuir o risco de supermedicalização e proteger de uma intervenção médica invasiva; sugerindo procedimentos científica e eticamente aceitáveis.

Page 18: Pontifícia Universidade Católica do Paraná Processo ...static.medgrupo.com.br/static/concursos/editais/HUC/2013/GABARITOS... · A) Homem de 24 anos após uso inalatório de cocaína

Pontifícia Universidade Católica do Paraná – Programa de Residência Médica – Janeiro/ 2013 Pág. 17

86. A Sra Lourdes, 58 anos, faz consultas regulares na Unidade de Saúde Vila Feliz. Mora no bairro há 42 anos, desde que se casou com o Sr. Rubens. Suas queixas nas últimas consultas são sempre relacionadas a uma “dor de cabeça que a acompanha durante quase todos os dias” e uma “tristeza que não vai embora nunca”. Sempre relata estar cansada de lidar com seus problemas. Já conversou com o Dr. Roberto sobre querer mudar de vida, mas sempre diz que não tem coragem. Nos últimos 6 meses mudou diversas vezes de medicação por não se adaptar com os efeitos colaterais de alguns remédios e também por achar que “eles não resolvem seus problemas”. Na tentativa de conhecer um pouco mais a história de D. Lourdes, Dr. Roberto constrói um genograma. De acordo com o genograma apresentado a seguir, marque a alternativa CORRETA:

I. Há visivelmente a repetição de padrões entre as gerações da família e possivelmente as questões relacionadas a alcoolismo e dependência química dos membros da família influenciem diretamente o estado de saúde de Sra.. Lourdes.

II. Pensando na visão sistêmica, o planejamento familiar seria uma das principais medidas a serem abordadas pelo Dr. Roberto, principalmente relacionada à 3ª geração.

III. Uma das possíveis abordagens do Dr. Roberto seria trabalhar questões, como apoiar um papel mais central das gerações do meio e manter o funcionamento e interesses próprios do casal em face do declínio biológico.

A) Somente a afirmativa I é verdadeira. B) As afirmativas I, II e III são verdadeiras. C) Somente a afirmativa II é verdadeira. D) Somente a afirmativa III é verdadeira. E) As afirmativas I e II são verdadeiras.

87. Em 2005, foi instituída, pela Organização Mundial da Saúde (OMS), a Comissão dos Determinantes Sociais da Saúde (DSS), cujo compromisso básico foi apoiar os países e os parceiros da saúde global no enfrentamento dos fatores sociais que levam a problemas de saúde. Tendo em vista seus conhecimentos a respeito dos DSS e impacto da condição da saúde na qualidade de vida da população, considere as afirmações a seguir:

I. A Comissão dos DSS, em seu relatório final,

apontou como recomendações chave: melhorar as condições de vida diária; abordar a distribuição desigual de poder, dinheiro e recursos; e medir e compreender o problema e avaliar o impacto da ação.

II. Fatores comportamentais, sociais e ambientais têm papel secundário na determinação de doenças, quando comparados com o papel de fatores biológicos.

III. O Brasil aderiu ao chamado da OMS criando em março de 2006 a Comissão Nacional dos DSS. Dada a complexidade da tarefa, a Comissão foi integrada por 16 personalidades oriundas dos mais diversos setores da vida social, econômica, cultual e científica do país.

IV. A Comissão Nacional dos DSS analisou pontos prioritários da situação de saúde no Brasil, tais como: situação e tendências da evolução demográfica, social e econômica, a estratificação socioeconômica e a saúde, condições de vida, ambiente e trabalho, redes sociais, comunitárias e saúde, comportamentos, estilo de vida e saúde, saúde materno-infantil e saúde indígena.

Está(ão) CORRETA(S):

A) Todas as afirmações. B) Somente as afirmações I e II. C) Somente a afirmação I. D) Somente as afirmações III e IV. E) Somente as afirmações I, III e IV.

88. Considere o resumo extraído do periódico Revista

Brasileira de Clínica Médica (2012); 10(4): 291-7 a seguir: “A qualidade da assistência à saúde é prejudicada diante do longo tempo de permanência hospitalar, relacionado às características e à procedência do paciente. O objetivo deste estudo foi avaliar quais os fatores que interferem no tempo de internação em uma enfermaria de clínica médica. O estudo foi realizado em Hospital Universitário, no período de agosto de 2010 a março de 2011, por intermédio da aplicação de questionários aos pacientes de alta hospitalar da clínica médica, consulta ao prontuário médico e aos médicos responsáveis. Os dados foram analisados, dividindo-se os pacientes em Grupo 1 (G1), tempo de internação inferior a 10 dias; e Grupo 2 (G2), tempo de internação superior a 10 dias. A amostra foi composta de 48 pacientes, sendo 72% do sexo feminino, com idade variando entre 18 e 85 anos. A média do tempo de internação foi 20,9 dias. A média de idade foi maior no G2 (p = 0,004). Houve predominância de indivíduos com escolaridade mais

Page 19: Pontifícia Universidade Católica do Paraná Processo ...static.medgrupo.com.br/static/concursos/editais/HUC/2013/GABARITOS... · A) Homem de 24 anos após uso inalatório de cocaína

Pontifícia Universidade Católica do Paraná – Programa de Residência Médica – Janeiro/ 2013 Pág. 18

alta no G1 (p = 0,001). A percepção da mudança da dor pelo paciente, avaliada através da escala Likert, mostrou predominância de melhora deste sintoma no G1 (p = 0,014). Os pacientes do G1 eram mais idosos e tinham menor escolaridade em média. A maior escolaridade desse grupo no tempo de internação pode sugerir que estes indivíduos cuidam mais da saúde. Ainda, o estudo permite inferir que os pacientes recebem alta apenas quando apresentam melhora do quadro de dor, sendo este um fator que pode prolongar a internação.” O desenho desse estudo epidemiológico pode ser classificado como:

A) Estudo Transversal. B) Estudo de Caso-Controle. C) Estudo de Coortes. D) Estudo Ecológico. E) Ensaio Clínico Randomizado.

89. Cerca de 12 horas após uma ceia de

confraternização de fim de ano de uma empresa, 37 pessoas desenvolveram um quadro de intoxicação alimentar. Diversos alimentos e bebidas foram consumidos no evento. Autoridades sanitárias suspeitaram de que os episódios estavam relacionados ao consumo da bomba de creme. Identificaram que 17 dos 37 acometidos haviam consumido a sobremesa. Estes resultados foram comparados com os de outras 33 pessoas que não apresentaram intoxicação alimentar, aleatoriamente escolhidas dentre as que participaram do mesmo evento. Destas, apenas 7 haviam consumido o prato sob suspeita, enquanto 26 não o fizeram. Qual a chance de ter sido a bomba de creme a responsável pela intoxicação alimentar?

A) Os dados não permitem reconhecer a chance. B) 4,37 chances de ter sido para uma de não ter

sido. C) 2,16 chances de ter sido para uma de não ter

sido. D) 1,87 chances de ter sido para uma de não ter

sido. E) 3,16 chances de ter sido para uma de não ter

sido.

90. Foi realizado um estudo sobre a capacidade dos clínicos de diagnosticar pneumonia em 265 pacientes que visitaram a emergência de um hospital. As impressões clínicas dos médicos foram comparadas aos resultados de RX de tórax. Dos 67 pacientes que apresentaram RX de tórax positivo para pneumonia, 47 foram diagnosticados pelos médicos como tendo a doença. Entre os 198 pacientes que apresentaram o RX de tórax negativo para pneumonia, os médicos diagnosticaram que 35 deles apresentavam a doença. Dado este contexto, considere as afirmações abaixo: I. A sensibilidade da impressão clínica do médico

sobre a pneumonia neste estudo foi de 70,1%.

II. A especificidade da impressão clínica do médico sobre a faringite estreptocócica neste estudo foi de 82,3%.

III. Se o médico pensou que o paciente tinha pneumonia, ele estava certo em 89% dos pacientes.

IV. A acurácia da impressão clínica do médico sobre a pneumonia nos pacientes que procuraram a emergência foi de aproximadamente 79,2%.

Está(ão) CORRETA(S):

A) Somente as afirmações I e II. B) Somente as afirmações I, III e IV. C) Somente a afirmação IV. D) Somente as afirmações I, II e IV. E) Somente as afirmações II e III.

91. Com o objetivo de avaliar a associação entre

consumo de chocolate e doenças cardiometabólicas, Buitrago-Lopez et col (BMJ, 2011) realizaram uma revisão sistemática com meta-análise de estudos observacionais. Na figura abaixo é apresentado o diagrama dos resultados obtidos (forest plot).

Considere as seguintes afirmações sobre os resultados indicados nos subtotais (sínteses):

I. O consumo de chocolate em níveis mais altos,

quando comparado ao consumo em níveis mais baixos, está associado a uma redução de 37% no risco de doenças cardiovasculares, de 5% no risco de insuficiência cardíaca e de 29% no risco de AVC.

II. A redução do risco de doenças cardiovasculares e de AVC é estatisticamente significativa,

Estudo

Doença cardiovascular

Insuficiência cardíaca

AVC

Risco relativo

(IC 95%)

Risco relativo

(IC 95%)

A favor de consumo de

chocolate em níveis mais altos

A favor de consumo de chocolate

em níveis mais baixos

Page 20: Pontifícia Universidade Católica do Paraná Processo ...static.medgrupo.com.br/static/concursos/editais/HUC/2013/GABARITOS... · A) Homem de 24 anos após uso inalatório de cocaína

Pontifícia Universidade Católica do Paraná – Programa de Residência Médica – Janeiro/ 2013 Pág. 19

enquanto que a redução do risco de insuficiência cardíaca não é estatisticamente significativa.

III. O consumo de chocolate em níveis mais altos apresentou-se como um fator de risco para doenças cardiovasculares, insuficiência cardíaca e AVC.

Está(ão) correta(s) SOMENTE:

A) As afirmações I e III. B) A afirmação I. C) As afirmações I e II. D) A afirmação III. E) As afirmações II e III.

92. O Programa Nacional de Controle da Tuberculose,

juntamente com o Ministério da Saúde, em 2011, lançaram um guia rápido para profissionais da saúde com recomendações atuais para o controle da tuberculose. A partir dos seus conhecimentos sobre o tema considere as afirmações abaixo: I. As indicações de baciloscopia são indivíduos

sintomáticos respiratórios, suspeita clínica e/ou radiológica de tuberculose pulmonar, febre, sudorese noturna, emagrecimento, suspeita clínica de tuberculose extrapulmonar (exame em materiais biológicos diversos).

II. A baciloscopia de escarro deve ser realizada em, no mínimo, duas amostras: uma, por ocasião da primeira consulta, e outra, independente do resultado da primeira, na manhã do dia seguinte, preferencialmente ao despertar.

III. A radiografia de tórax deve ser solicitada para todo paciente com suspeita clínica de tuberculose pulmonar; ela inclusive pode substituir a baciloscopia.

IV. Os pacientes com intolerância grave a algum dos fármacos antiTB deverão ser encaminhados à Referência Secundária e os pacientes com qualquer tipo de resistência aos fármacos antiTB deverão ser encaminhados à Referência Terciária.

Está(ão) correta(s) APENAS a(s) afirmativa(s):

A) I e II. B) I, II e IV. C) III e IV. D) I, III e IV. E) I.

93. A hanseníase é uma doença infecciosa crônica de

elevada magnitude em vários países. Em 2012, o coeficiente de prevalência de hanseníase alcançou 1,56 casos por 10 mil habitantes, com importantes variações regionais. Sobre esse tema, considere as afirmações abaixo: I. O Mycobacterium leprae (bacilo de Hansen),

causador da doença infectocontagiosa, é considerado com baixa infectividade e alta patogenicidade.

II. A principal via de eliminação e infecção de indivíduo pelo bacilo são as vias aéreas superiores: mucosa nasal e orofaringe.

III. Os medicamentos em geral, aqueles utilizados na poliquimioterapia (PQT/OMS) e no tratamento dos estados reacionais, podem provocar efeitos colaterais. No entanto, os trabalhos bem controlados, publicados na literatura disponível, permitem afirmar que o tratamento PQT/OMS raramente precisa ser interrompido em virtude de efeitos colaterais.

IV. Em mulheres na idade reprodutiva, deve-se atentar ao fato que a rifampicina pode interagir com anticoncepcionais orais, diminuindo a sua ação.

Está(ão) correta(s) APENAS a(s) afirmativa(s):

A) I e II. B) III e IV. C) I, III e IV. D) II, III e IV. E) I.

94. O Sistema de Notificação Negativa faz parte das

ações de vigilância epidemiológica por meio do Sistema de Informação de Agravos de Notificação – SINAN. Sobre este tema, considere as afirmativas:

I. O formulário de Notificação Negativa deve ser

preenchido pela unidades de saúde caso seja identificada uma importante redução à incidência da doença de notificação compulsória, devido à eficácia das intervenções.

II. Essa é uma estratégia criada para demonstrar que os profissionais e o sistema de vigilância da área estão alerta para a ocorrência de tais eventos.

III. Essa é uma estratégia criada para evitar a subnotificação.

IV. A comunicação deverá ocorrer a cada dois meses.

Está(ão) correta(s) APENAS a(s) afirmativa(s):

A) II e III. B) III e IV. C) I, II e III. D) I, III e IV. E) I.

95. Em setembro de 2012, os responsáveis pela

Vigilância Epidemiológica da Secretaria Estadual de Saúde de um estado brasileiro observaram um aumento inusitado no número de casos de toxoplasmose não especificada em uma região pobre no interior do estado, com uma população de 50.000 habitantes. Em apenas 30 dias, foram registrados 1.000 casos de toxoplasmose, procedentes de diferentes bairros da zona urbana, além de diversos povoados da zona rural. Com base nos conceitos de distribuição espacial e temporal das doenças na população, a ocorrência

Page 21: Pontifícia Universidade Católica do Paraná Processo ...static.medgrupo.com.br/static/concursos/editais/HUC/2013/GABARITOS... · A) Homem de 24 anos após uso inalatório de cocaína

Pontifícia Universidade Católica do Paraná – Programa de Residência Médica – Janeiro/ 2013 Pág. 20

destes casos de toxoplasmose é melhor descrita como uma:

A) Doença emergente. B) Pandemia. C) Endemia. D) Epizootia. E) Epidemia.

96. A medida de associação que representa a

probabilidade de um evento ocorrer em uma população sob estudo e que apresenta a melhor forma de os pacientes entenderem como os fatores de risco podem afetar suas vidas é o:

A) Valor preditivo negativo. B) Risco atribuível. C) Risco relativo. D) Valor preditivo positivo. E) Risco absoluto.

97. Um grupo de dez profissionais da Atenção Primária

em Saúde incluindo duas enfermeiras, duas auxiliares de enfermagem, um educador físico, uma nutricionista, um dentista, duas técnicas em saúde bucal e um médico projetaram e desenvolveram no último ano uma ação de promoção da saúde com adolescentes entre 12 e 17 anos em uma escola do território de abrangência da sua unidade de saúde. A diretora da escola e duas professoras foram convidadas a contribuir na ação que contou com a articulação de saberes e experiências em um processo de construção compartilhada, estabelecendo vínculos de co-responsabilidade e co-getão pela melhoria da qualidade de vida da população em questão, procurando reduzir vulnerabilidades e riscos relacionados à saúde e aos seus determinantes e condicionantes. Assinale a alternativa com o princípio da promoção da saúde que melhor representa essa ação:

A) Intersetorialidade. B) Participação. C) Governança. D) Empoderamento. E) Sustentabilidade.

98. Estabelecer estratégias e procedimentos de

acompanhamento da gestão do Sistema Único de Saúde (SUS), articulando-se com os demais colegiados como os de seguridade, meio ambiente, justiça, educação, trabalho, agricultura, idosos, criança, adolescentes e outros é uma das competências de:

A) Profissionais da Atenção Primária em Saúde. B) Usuários do Sistema Único de Saúde. C) Conselhos de Saúde. D) Profissionais da Vigilância Epidemiológica. E) Gestores das unidades de saúde.

99. Considere as afirmações abaixo:

I. O método epidemiológico de coorte é o mais indicado para avaliar a eficácia de novos medicamentos.

II. O processo de randomização de amostragem, utilizado nos ensaios clínicos randomizados destaca esse tipo de estudo por reduzir o viés de seleção.

III. O indicador utilizado no planejamento da quantidade de leitos necessários para a internação por pneumonia em um sistema de saúde é o coeficiente de letalidade.

IV. Para o controle de variáveis confundidoras nos estudos caso-controle é utilizado o procedimento de pareamento. Uma dificuldade para sua realização é que esse procedimento é demorado para encontrar controles com as características necessárias.

A) Somente as afirmações I e II são verdadeiras. B) Somente as afirmações I e III são verdadeiras. C) Somente as afirmações III e IV são

verdadeiras. D) Somente as afirmações II e IV são verdadeiras.

Somente as afirmações II, III e IV são verdadeiras.

100. Com o objetivo de induzir a ampliação do acesso e a melhoria da qualidade da Atenção Básica, com garantia de um padrão de qualidade comparável nacional, regional e localmente de maneira a permitir maior transparência e efetividade das ações governamentais direcionadas à Atenção Básica em Saúde, em julho de 2011, foi instituído(a) o(a):

A) Programa Nacional de Melhoria do Acesso e

da Qualidade da Atenção Básica. B) Política Nacional de Promoção da Saúde. C) Programa Nacional de Segurança Pública com

Cidadania. D) Programa Nacional de Educação Permanente

das Equipes de Atenção Básica. E) Programa Nacional de Desenvolvimento

Gerencial no SUS.

Page 22: Pontifícia Universidade Católica do Paraná Processo ...static.medgrupo.com.br/static/concursos/editais/HUC/2013/GABARITOS... · A) Homem de 24 anos após uso inalatório de cocaína

Pontifícia Universidade Católica do Paraná Processo Seletivo de Residênc ia Médica

20 de Janeiro de 2013

Cirurgia de Mão

INFORMAÇÕES / INSTRUÇÕES:

1. Verifique se a prova está completa: questões de números 1 a 50. 2. A compreensão e a interpretação das questões constituem parte integrante da prova, razão pela qual os

fiscais não poderão interferir. 3. Preenchimento do Cartão-Resposta :

- Preencher para cada questão apenas uma resposta - Preencher totalmente o espaço correspondente, conforme o modelo: - Usar caneta esferográfica, escrita normal, tinta azul ou preta. - Para qualquer outra forma de preenchimento, a leitora anulará a questão.

O CARTÃO-RESPOSTA É PERSONALIZADO. NÃO PODE SER SUBSTITUÍDO, NEM CONTER RASURAS.

Duração total da prova: 3 horas

�- - - - - - - - - - - - - - - - - - - - - - - - - - - - - - - - - - - - - - - - - - - - - - - - - - - - - - - - - - - - - - - - - - - - - - - - Anote o seu gabarito.

1.

2.

3.

4.

5.

6.

7.

8.

9.

10.

11.

12.

13.

14.

15.

16.

17.

18.

19.

20.

21. 22. 23. 24. 25. 26. 27. 28. 29. 30.

31.

32.

33.

34.

35.

36.

37.

38.

39.

40.

41.

42.

43.

44.

45.

46.

47.

48

49.

50.

Page 23: Pontifícia Universidade Católica do Paraná Processo ...static.medgrupo.com.br/static/concursos/editais/HUC/2013/GABARITOS... · A) Homem de 24 anos após uso inalatório de cocaína

Pontifícia Universidade Católica do Paraná – Programa de Residência Médica – Janeiro / 2013 Pág. 1

Page 24: Pontifícia Universidade Católica do Paraná Processo ...static.medgrupo.com.br/static/concursos/editais/HUC/2013/GABARITOS... · A) Homem de 24 anos após uso inalatório de cocaína

Pontifícia Universidade Católica do Paraná – Programa de Residência Médica – Janeiro / 2013 Pág. 2

1. O enxerto de pele total:

A) É retirado de locais como coxa e abdome preferencialmente.

B) Permite fácil re-epitelização na área doadora. C) É retirado de locais que possam ser suturados

sem tensão excessiva. D) Preserva a parte mais profunda da derme na

área doadora. E) Pode conter componente adicional de gordura

ou cartilagem.

2. Durante o processo de integração dos enxertos de pele a fase de embebição plasmática corresponde a:

A) Fase de irrigação sanguínea que ocorre após

as primeiras 48 horas. B) Fase de formação de novos capilares que

ocorre nas primeiras 48 horas. C) Fase de formação de novos capilares que

ocorre após as primeiras 48 horas. D) Fase de nutrição inicial do enxerto que ocorre

nas primeiras 48 horas. E) Fase de proliferação de células epiteliais que

ocorre após as primeiras 48 horas.

3. Quanto à fotocarcinogênese assinale a CORRETA:

A) A radiação ultravioleta A de comprimento de onda entre 320 e 400 nm tem ação fotocarcinogênica direta e leva a carcinomas baso e espinocelulares.

B) A radiação ultravioleta B de comprimento de onda entre 290 e 320 nm tem ação fotocarcinogênica direta e leva a carcinomas baso e espinocelulares.

C) A radiação ultravioleta C de comprimento de onde entre 290 e 320 nm tem ação fotocarcinogênica direta e leva a carcinomas baso e espinocelulares.

D) A fotocarcinogênese é considerada fator inicial em 100% dos cânceres de pele.

E) A radiação ultravioleta A é absorvida pela atmosfera, via camada de ozônio.

4. Uma úlcera dolorosa sobre o tornozelo, com leito

seco e pálido e com a extremidade do pé apresentando rubor e dor na sua elevação, aponta para o diagnóstico de:

A) Úlcera resultante da doença arterial oclusiva. B) Úlcera resultante da neuropatia diabética. C) Úlcera de pressão. D) Úlcera resultante da insuficiência venosa. E) Carcinoma escamoso.

5. Em relação às sindactilias nas mãos de crianças, a

complicação mais frequente é:

A) Lesão vascular com perda do dedo. B) Infecção. C) Rotação dos dedos. D) Perda das unhas. E) Recidiva parcial.

6. Fazem parte da “fisiopatologia” do envelhecimento facial:

A) Absorção óssea e ptose tecidual. B) Alongamento muscular e flacidez cutânea. C) Hipertrofia e extrusão gordurosa. D) Manchas cutâneas e nódulos subcutâneos. E) Exoftalmia e flacidez muscular.

7. Leia as frases a seguir em relação ao entrópio

palpebral:

I. No tipo congênito, muitas vezes é acompanhado de distiquíase (mais de uma fileira de cílios).

II. O tipo adquirido pode ser causado por queimaduras na conjuntiva ou por doenças conjuntivais.

III. O tipo adquirido involucional é comum nos negros, pois eles apresentam sulco palpebral mais alto.

Está(ão) correta(s) APENAS :

A) I. B) II. C) I e II. D) III. E) I e III.

8. Quanto aos ângulos, medidas e referências ideais no

nariz, assinale a CORRETA:

A) O ângulo entre o nariz e o lábio superior varia entre 50º e 80°.

B) As asas nasais no seu ponto de maior convexidade devem estar em linha com os cantos laterais das pálpebras.

C) As narinas não devem ser menores que o tamanho do lóbulo nasal nem maiores que o dobro do seu tamanho.

D) O tamanho vertical do nariz (glabela até columela) é semelhante ao comprimento do dedo médio da mão.

E) A ponta nasal deve estar mais baixa que o dorso nasal.

9. Em relação às fissuras labiais e palatais, assinale a CORRETA:

A) A fissura labial é mais comum no sexo

masculino. B) A fissura palatal é mais comum no sexo

masculino. C) A incidência é maior na raça negra em relação

à raça amarela. D) Os fatores genéticos representam cerca de

70% da etiopatogenia da doença. E) As alterações na embriogênese ocorrem entre

a 1ª e 3ª semana de vida intrauterina. 10. O tipo de lipoaspiração que tem risco potencial de

edema agudo do pulmão é:

A) Lipoaspiração superficial. B) Lipoaspiração tumescente.

Page 25: Pontifícia Universidade Católica do Paraná Processo ...static.medgrupo.com.br/static/concursos/editais/HUC/2013/GABARITOS... · A) Homem de 24 anos após uso inalatório de cocaína

Pontifícia Universidade Católica do Paraná – Programa de Residência Médica – Janeiro / 2013 Pág. 3

C) Lipoaspiração úmida. D) Lipoaspiração seca. E) Lipoaspiração com vibração mecânica.

11. Paciente que sofreu queimadura de 2º grau nas

regiões anteriores das duas coxas é considerado:

A) Médio queimado. B) Grande queimado. C) Não é possível classificar quando a

queimadura é nas coxas. D) Paciente de área de risco. E) Pequeno queimado.

12. Relacione o retalho fasciocutâneo (RFC) ao exemplo

correspondente na sequência:

1. RFC com múltiplas artérias perfurantes fasciais 2. RFC com somente uma artéria perfurante fascial 3. RFC com várias artérias perfurantes provenientes de um vaso de maior calibre.

A) RFC da artéria radial do antebraço. B) RFC escapular. C) RFC da artéria digital.

Assinale a alternativa CORRETA:

A) “1” corresponde a “C”, “2” a “B” e “3” a “A” B) “1” corresponde a “A”, “2” a “B” e “3” a “C” C) “1” corresponde a “B”, “2” a “A” e “3” a “C” D) “1” corresponde a “C”, “2” a “A” e “3” a “B” E) “1” corresponde a “A”, “2” a “C” e “3” a “B”

13. De acordo com a Classificação de Mathes-Nahai (tipo

de vascularização muscular), o Retalho do Músculo Gracilis corresponde a retalho com:

A) Um pedículo vascular. B) Dois pedículos dominantes. C) Um pedículo dominante e vários outros

menores. D) Três pedículos vasculares. E) Pedículos vasculares segmentares.

14. O retalho randomizado (RR) rodado sobre um ponto

fixo, geralmente curvilíneo, bem aproveitado para cobrir um defeito cutâneo triangular, é chamado:

A) RR de transposição. B) RR de avanço. C) RR tubular. D) RR à distância. E) RR de rotação.

15. A vascularização de um retalho axial é proveniente

de:

A) Artéria septocutânea incluída no maior eixo do retalho.

B) Plexo dérmico incluído no maior eixo do retalho.

C) Plexo subdérmico incluído no eixo transverso do retalho.

D) Artéria musculocutânea no eixo transverso do retalho.

E) Vascularização indireta no maior eixo do retalho.

16. Considerando a biologia da cicatrização após um

trauma cutâneo, assinale a CORRETA:

A) A formação do coágulo irá manter afastadas as bordas da ferida.

B) A fase de inflamação ocorre dentro das primeiras 6 horas do trauma.

C) Os macrófagos responsáveis pelo desbridamento da lesão estão presentes na ferida nas primeiras 6 horas após o trauma.

D) Na fase inicial ocorre vasoconstrição imediata e ativação de processos que levam à formação do agregado plaquetário.

E) Na fase de cicatrização o principal elemento é a expansão da ferida.

17. Nas zetaplastias o alongamento da pele com

correção da retração cicatricial:

A) Será maior quanto menor o ângulo utilizado na confecção dos triângulos.

B) Será maior nas zetaplastias simples em relação às múltiplas.

C) Será menor na zetaplastia de Limberg. D) Depende do ângulo realizado na confecção

dos triângulos. E) Depende do maior descolamento possível da

pele para a rotação dos triângulos. 18. A derme saudável contém cerca de:

A) 80% de colágeno tipo I e 20% de colágeno tipo III.

B) 20% de colágeno tipo I e 80% de colágeno tipo III.

C) 60% de colágeno tipo I e 40% de colágeno tipo III.

D) 40% de colágeno tipo I e 60% de colágeno tipo III.

E) 50% de colágeno tipo I e 50% de colágeno tipo III.

19. O queloide:

A) É similar clinicamente à cicatriz hipertrófica. B) Histologicamente apresenta menor atividade

metabólica do que a cicatriz hipertrófica. C) O maior tensionamento da pele nas feridas faz

parte da prevenção. D) É mais comum em índios. E) Ultrapassa o limite da incisão cirúrgica.

20. Os melanócitos são encontrados em qual camada da

epiderme:

A) Camada espinhosa. B) Camada basal. C) Camada granulosa. D) Camada córnea. E) Camada queratinizada.

Page 26: Pontifícia Universidade Católica do Paraná Processo ...static.medgrupo.com.br/static/concursos/editais/HUC/2013/GABARITOS... · A) Homem de 24 anos após uso inalatório de cocaína

Pontifícia Universidade Católica do Paraná – Programa de Residência Médica – Janeiro / 2013 Pág. 4

21. Quanto às linhas de Langer, assinale a CORRETA:

A) Recebem diretamente as influências dinâmicas do sistema músculo-esquelético.

B) A tensão máxima da pele ocorre na direção paralela às linhas.

C) Descrevem as tensões cutâneas estáticas em determinado local no corpo.

D) Também chamadas de linhas de extensibilidade máxima.

E) As feridas paralelas às linhas são difíceis de cicatrizar.

22. Na lesão dos tendões extensores do indicador

ocorrida por ferimento inciso no dorso da mão, próximo ao retináculo dos extensores, haverá:

A) Incapacidade para extensão dos dedos, médio

e anular. B) Dificuldade para extensão do polegar. C) Incapacidade para realizar a extensão das

interfalângicas do indicador. D) Dificuldade para realizar a flexão da

metacarpofalângica do indicador. E) Incapacidade para realizar a extensão da

metacarpofalângica do indicador.

23. Quanto ao hematoma subungueal, assinale a CORRETA:

A) É o acúmulo de sangue entre a placa ungueal

e o leito ungueal. B) Dificilmente cursa com dor. C) A causa principal é o trauma da cutícula inicial. D) Hematomas acometendo 25% da unha devem

ser tratados com drenagem. E) Raramente é necessário radiografia devido ao

baixo risco de fratura de falange.

24. Em relação à toxicidade dos anestésicos locais (AL), assinale a CORRETA:

A) A dose necessária para causar toxicidade no

sistema nervoso central é maior do que no sistema cardiovascular.

B) As reações alérgicas são frequentes. C) As reações alérgicas são manifestações da

baixa concentração plasmática dos AL. D) A toxicidade dos AL está mais relacionada ao

aumento da concentração plasmática do que à dose total injetada.

E) Os sinais cardiovasculares são os mais importantes para o diagnóstico da intoxicação por AL.

25. Na fascite necrozante:

A) Há sinais importantes de toxicidade local no início da doença.

B) A taxa de mortalidade é de aproximadamente 10%.

C) O tipo menos comum apresenta como agente causal uma única bactéria.

D) Raramente ocorre dor. E) Doença vascular periférica e idade superior a

60 anos estão entre os fatores predisponentes.

26. O que diferencia a derme retinacular profunda da hipoderme é/são:

A) As redes vasculonervosas existentes na

hipoderme. B) A transição abrupta entre tecido fibroso

dérmico e tecido gorduroso. C) Os anexos epidérmicos existentes na derme

retinacular. D) Os adipócitos da derme retinacular. E) O tecido queratinizado da derme retinacular.

27. Em relação aos torniquetes aplicados nos membros

durante as cirurgias, assinale a CORRETA:

A) É frequente a necrose superficial da pele na margem superior do torniquete aplicado na coxa.

B) A pressão ideal do torniquete depende da idade do paciente, da pressão arterial e do tamanho da extremidade.

C) Manguitos de torniquetes estreitos são mais eficazes a baixas pressões de preenchimento do que os largos.

D) São instrumentos utilizados com frequência e sem riscos.

E) Torniquetes curvos adaptam-se melhor a membros cilíndricos.

28. O enxerto ósseo esponjoso:

A) Somente é incorporado ao osso hospedeiro se as células no enxerto ósseo permanecerem viáveis.

B) De grande tamanho pode ser obtido facilmente do côndilo femoral.

C) Tem efeito de suporte estrutural. D) Deve ser retirado de toda a região subcondral

justa articular. E) Incorpora-se ao osso hospedeiro mais

rapidamente do que o enxerto cortical.

29. Na artrodese do quadril, a posição almejada é:

A) 30° de flexão, 5° de adução e 10° de rotação externa.

B) 40° de flexão, 15° de adução e 5° de rotação externa.

C) 20° de flexão, 0° de adução e 10° de rotação interna.

D) 30° de flexão, 15° de adução e 5° de rotação interna.

E) 40° de flexão, 10° de adução e 15° de rotação externa.

30. Em relação à estabilidade patelofemoral, assinale a

CORRETA:

A) Exceto pelo vasto lateral, o quadríceps atua em linha com o eixo anatômico do fêmur.

B) O ângulo Q é medido entre o eixo anatômico do fêmur e a linha que passa na borda externa da patela.

C) No início da flexão do joelho a subluxação da patela é resistida principalmente pelo vasto medial oblíquo.

Page 27: Pontifícia Universidade Católica do Paraná Processo ...static.medgrupo.com.br/static/concursos/editais/HUC/2013/GABARITOS... · A) Homem de 24 anos após uso inalatório de cocaína

Pontifícia Universidade Católica do Paraná – Programa de Residência Médica – Janeiro / 2013 Pág. 5

D) A patela não mantém contato com a tróclea femoral apenas no final da flexão.

E) Quanto maior o ângulo Q menor a tendência de subluxação lateral da patela.

31. O organismo infectante encontrado com maior

probabilidade em bebês sadios, de 2 a 4 semanas de idade é:

A) Haemophilus influenzae. B) Coliformes Gran-negativos. C) Pseudomonas. D) Streptococcus grupo B. E) Salmonella.

32. Leia as afirmativas a seguir em relação à

tuberculose:

I. O comprometimento extrapulmonar da doença ocorre em aproximadamente 14% dos pacientes.

II. Cerca da metade dos pacientes com doença óssea apresenta envolvimento pulmonar.

III. A coluna vertebral é o sítio ósseo de maior comprometimento da doença.

Está(ão) correta(s) APENAS :

A) Todas as afirmativas. B) I e II. C) II e III. D) I e III. E) III.

33. Quanto aos tumores ósseos benignos, assinale a

CORRETA:

A) Os neurilenomas ósseos são frequentes e na radiografia apresentam-se como lesões blásticas.

B) Os lipomas intraósseos são frequentes e na radiografia apresentam-se como lesões líticas.

C) O osteoma osteoide ocorre com mais frequência em mulheres jovens.

D) O hemangioma é o tumor vascular benigno mais comum do osso e compromete o corpo vertebral com frequência.

E) Cerca de 10% dos pacientes com osteocondromas têm apenas uma lesão.

34. O nervo radial é composto por fibras originadas do plexo braquial a partir das raízes de:

A) C5, C6 e C7 e às vezes C8. B) C7 e C8. C) C7, C8 e T1 e às vezes T2. D) C8 e T1. E) C6, C7 e C8 e às vezes T1.

35. Na deformidade em DISI do carpo/punho

(instabilidade intercalar dorsal do semilunar), os ângulos escafossemilunar e capitatossemilunar são, respectivamente:

A) Maior que 60° e menor que 20° B) Menor que 60° e menor que 20° C) Maior que 60° e maior que 20°

D) Menor que 60° e maior que 20° E) Igual a 60° e igual a 20°

36. Quanto ao hálux rígido, assinale a alternativa

CORRETA:

A) Não há destruição do osso subcondral na articulação metatarsofalângica.

B) O processo pode iniciar na adolescência devido a evento traumático isolado.

C) Há postura fixa em extensão da articulação metatarsofalângica.

D) A osteocondrite dissecante da primeira cabeça metatarsal é a causa mais comum.

E) O osteófito característico ocorre na margem articular plantar da primeira cabeça metatarsal.

37. Em relação ao objetivo comum no tratamento das fraturas de calcâneo, assinale a CORRETA:

A) Realinhamento da tuberosidade na posição em

varo. B) Restauração da congruência da faceta anterior

da articulação subtalar. C) Manutenção do ângulo de Böhler abaixo de

25°. D) Aumento da largura do calcâneo. E) Redução da articulação calcaneocuboidea se

fraturada.

38. No metatarso aduto congênito classificado por Bleck como moderado, clinicamente o antepé:

A) Apresenta flexibilidade suficiente para ser

trazido para a linha média do pé. B) Pode ser abduzido para a linha média do pé ou

além desse ponto. C) Não apresenta flexibilidade para ser abduzido

até a linha média do pé. D) Não apresenta flexibilidade de modo algum. E) Não apresenta flexibilidade, porém pode-se

corrigir o hálux. 39. São consideradas complicações precoces das

fraturas supracondilares em crianças:

A) Lesão do nervo interósseo posterior e síndrome compartimental.

B) Síndrome compartimental e cúbito varo. C) Lesão do nervo ulnar e cúbito varo. D) Lesão da artéria braquial e lesão do nervo

axilar. E) Lesão do nervo interósseo anterior e lesão da

artéria braquial. 40. Leia as asertivas a seguir em relação à escoliose

idiopática (EI):

I. Em 3% dos adolescentes com EI poderá ocorrer melhora espontânea em geral nas curvas menores que 11°.

II. A progressão da EI ocorre com maior frequência no estirão de crescimento do adolescente do sexo masculino.

Page 28: Pontifícia Universidade Católica do Paraná Processo ...static.medgrupo.com.br/static/concursos/editais/HUC/2013/GABARITOS... · A) Homem de 24 anos após uso inalatório de cocaína

Pontifícia Universidade Católica do Paraná – Programa de Residência Médica – Janeiro / 2013 Pág. 6

III. Em geral, curvas lombares simples progridem mais rapidamente do que curvas torácicas simples.

Está(ão) correta(s) APENAS :

A) I e III. B) I e II. C) II e III. D) I. E) III.

41. De acordo com a classificação de tile para as lesões

do anel pélvico, assinale a CORRETA:

A) No tipo C2 as duas hemipelves são verticalmente instáveis.

B) No tipo A2 há avulsão da espinha ilíaca ou da tuberosidade isquiádica.

C) No tipo B1 os ligamentos sacroilíacos posterior e interósseo permanecem intactos.

D) O tipo B2 está relacionado a quedas no idoso E) O tipo C1 é correspondente à lesão “em alça

de balde”.

42. A pseudartrose oligotrófica:

A) É um tipo de pseudartrose cominutiva. B) É uma lesão vascular que ocorre comumente

devido à diástase dos fragmentos. C) É conhecida como pseudartrose atrófica. D) Ocorre por imobilização inadequada. E) Ocorre por prematura sustentação do peso

corporal.

43. Se a luxação da cabeça do rádio ocorrer sem luxação da articulação umeroulnar no cotovelo, a cabeça do rádio estará deslocada:

A) Posteriormente. B) Lateralmente. C) Medialmente D) Anteriormente. E) Não ocorre o deslocamento da cabeça do rádio

sem que a articulação umeroulnar esteja luxada.

44. Na fratura em duas partes de Neer, com retração do tubérculo maior que 1 cm haverá ruptura longitudinal do manguito rotador e o fragmento do úmero retraído causará bloqueio da:

A) Rotação lateral em adução. B) Rotação medial em abdução. C) Rotação lateral em abdução. D) Rotação medial em adução. E) Rotação medial em extensão.

45. Sobre a fratura do platô tibial do tipo III classificada

por Schatzker, assinale a CORRETA:

A) Existe clivagem com desvio lateral e inferior de fragmento não cominutivo.

B) Existe clivagem combinada com depressão da superfície articular.

C) Ocorre fratura do côndilo medial.

D) Ocorre fratura bicondilar. E) A superfície articular é comprimida para dentro

do platô, porém a cortical lateral permanece intacta.

46. O tipo mais comum da fratura-luxação de Monteggia em crianças é a:

A) Fratura do terço proximal da ulna com

angulação posterior e luxação anterior da cabeça do rádio.

B) Fratura do terço proximal da ulna com angulação anterior e luxação posterior da cabeça do rádio.

C) Fratura do terço proximal da ulna com angulação posterior e luxação posterior da cabeça do rádio.

D) Fratura do terço proximal da ulna com angulação lateral e luxação lateral da cabeça do rádio.

E) Fratura do terço proximal da ulna com angulação anterior e luxação anterior da cabeça do rádio.

47. Em relação às fraturas da tuberosidade da tíbia em crianças, pergunta-se: em qual tipo, classificado por Watson-Jones, o traço de fratura passa somente através do centro de ossificação secundário da tíbia proximal?

A) Tipo I. B) Tipo V. C) Tipo II. D) Tipo IV. E) Tipo III.

48. Quanto à espondilolistese traumática do axis,

assinale a CORRETA:

A) A causa mais comum são os acidentes automobilísticos onde há hiperflexão da cabeça.

B) As fraturas tipo III (Levine e Edwards) apresentam fratura do arco neural com linha de fratura predominantemente vertical.

C) As fraturas tipo I (Levine e Edwards) apresentam lesão ligamentar mínima e são estáveis.

D) As fraturas tipo II apresentam luxação e déficit neurológico associados.

E) O occipício é tensionado para baixo causando a fratura dos pedículos de C1.

49. Para o diagnóstico preciso da fratura articular da base do 5° metacarpal, além das incidências anteroposterior e lateral, deve ser tirada radiografia com:

A) 30° de supinação. B) Desvio ulnar da mão. C) Desvio radial da mão. D) 30° de pronação. E) Os dedos cerrados (em flexão máxima).

Page 29: Pontifícia Universidade Católica do Paraná Processo ...static.medgrupo.com.br/static/concursos/editais/HUC/2013/GABARITOS... · A) Homem de 24 anos após uso inalatório de cocaína

Pontifícia Universidade Católica do Paraná – Programa de Residência Médica – Janeiro / 2013 Pág. 7

50. Quanto aos compartimentos do antebraço, quais são os músculos que compõem o compartimento móvel de Henry?

A) Braquiorradial, extensor radial longo do carpo e

extensor radial curto do carpo. B) Extensor radial longo do carpo, extensor radial

curto do carpo e extensor comum dos dedos. C) Braquiorradial, abdutor longo do polegar e

extensor curto do polegar. D) Braquiorradial e extensor radial longo do carpo. E) Extensor radial curto, abdutor longo do polegar

e extensor curto do polegar.

Page 30: Pontifícia Universidade Católica do Paraná Processo ...static.medgrupo.com.br/static/concursos/editais/HUC/2013/GABARITOS... · A) Homem de 24 anos após uso inalatório de cocaína

Pontifícia Universidade Católica do Paraná Processo Seletivo de Residênc ia Médica

20 de Janeiro de 2013

Geriatria

INFORMAÇÕES / INSTRUÇÕES:

1. Verifique se a prova está completa: questões de números 1 a 50. 2. A compreensão e a interpretação das questões constituem parte integrante da prova, razão pela qual os

fiscais não poderão interferir. 3. Preenchimento do Cartão-Resposta :

- Preencher para cada questão apenas uma resposta - Preencher totalmente o espaço correspondente, conforme o modelo: - Usar caneta esferográfica, escrita normal, tinta azul ou preta. - Para qualquer outra forma de preenchimento, a leitora anulará a questão.

O CARTÃO-RESPOSTA É PERSONALIZADO. NÃO PODE SER SUBSTITUÍDO, NEM CONTER RASURAS.

Duração total da prova: 3 horas

�- - - - - - - - - - - - - - - - - - - - - - - - - - - - - - - - - - - - - - - - - - - - - - - - - - - - - - - - - - - - - - - - - - - - - - - - Anote o seu gabarito.

1. 2. 3. 4. 5. 6. 7. 8. 9. 10.

11.

12.

13.

14.

15.

16.

17.

18.

19.

20.

21.

22.

23.

24.

25.

26.

27.

28.

29.

30.

31. 32. 33. 34. 35. 36. 37. 38. 39. 40.

41.

42.

43.

44.

45.

46.

47.

48

49.

50.

Page 31: Pontifícia Universidade Católica do Paraná Processo ...static.medgrupo.com.br/static/concursos/editais/HUC/2013/GABARITOS... · A) Homem de 24 anos após uso inalatório de cocaína

Pontifícia Universidade Católica do Paraná – Programa de Residência Médica – Janeiro / 2013 Pág. 1

Page 32: Pontifícia Universidade Católica do Paraná Processo ...static.medgrupo.com.br/static/concursos/editais/HUC/2013/GABARITOS... · A) Homem de 24 anos após uso inalatório de cocaína

Pontifícia Universidade Católica do Paraná – Programa de Residência Médica – Janeiro / 2013 Pág. 2

O caso clínico a seguir servirá de base para as próximas 3 (três) questões.

Sra. Julia, 72 anos, vem ao consultório para trazer os exames solicitados na primeira consulta e receber a orientação terapêutica. Repetido o exame físico encontram-se: Peso: 48 kg, PA: 170/98 mmHg em pé e sentada; diminuição do pulso tibial posterior à direita; ITB de 0.6; presença de B4. Nos exames complementares apresentava: ECG: sobrecarga ventricular esquerda, colesterol total: 265 mg/dL; HDL: 28 mg/dL; triglicerídeos: 456 mg/dL; glicemia: 109 mg/dL e TTG: 158 mg/dL; ácido úrico: 2.8 mg/dL; creatinina: 1.3 mg/dL.

1. Considerando os dados obtidos, as melhores

evidências científicas e a recomendação da VI Diretrizes Brasileiras de Hipertensão Arterial, o risco estratificado para esta paciente é:

A) Alto risco adicional. B) Moderado risco adicional. C) Risco adicional muito alto. D) Baixo risco adicional. E) Risco basal.

2. Considerando os dados obtidos, as melhores

evidências científicas e a recomendação da VI Diretrizes Brasileiras de Hipertensão Arterial, a taxa de filtração glomerular estimada-TFGE (Cockcroft-Gault) e, uma vez que não se pode obter o valor do LDL-Col pela fórmula de Friedewald, deve-se optar pelo cálculo do “não-HDL” nestes casos. Dado este contexto, os valores obtidos para TFGE e não-HDL, são:

A) TFGE: 34,87 mL/min e não-HDL: 237 mg/dL. B) TFGE: 42.34 mL/min e não-HDL: 187 mg/dL. C) TFGE: 29,64 mL/min e não-HDL: 187 mg/dL. D) TFGE: 29,64 mL/min e não-HDL: 237 mg/dL. E) TFGE: 34,87 mL/min e não HDL: 187 mg/dL.

3. Considerando os dados obtidos, as melhores

evidências científicas e a recomendação da VI Diretrizes Brasileiras de Hipertensão Arterial, a melhor opção terapêutica inicial (Via Oral) para a paciente, seria:

A) Atenolol 100 mg/dia e hidroclorotiazida 25

mg/dia. B) Losartana 100 mg/dia e furosemida 40 mg/dia. C) Hidroclorotiazida 25 mg/dia. D) Ramipril 5 mg/dia e hidroclorotiazida 25

mg/dia. E) Losartana 100 mg/dia.

4. Sr. João Doe, 49 anos, vem à consulta com queixa

de palpitação e tontura. Feito o ECG (a seguir), qual o diagnóstico eletrocardiográfico?

A) Síndrome de Wolff–Parkinson–White. B) Síndrome de Lown–Ganong–Levine. C) Sobrecarga Ventricular Esquerda. D) Bloqueio da Divisão Póstero-Inferior do Ramo

Direito. E) Eletrocardiograma dentro dos padrões da

normalidade.

5. Sr. João Doe, 49 anos (mesmo paciente da questão anterior), após a consulta com cardiologista, que o informou de que estava com fibrilação atrial, cai desacordado ao sair do consultório. Durante o atendido de emergência o traçado do ECG obtido foi este:

Ao revisar a medicação em uso pelo paciente nos últimos seis meses, foi identificado um medicamento que poderia ser a causa deste evento. Dado este contexto, pergunta-se: qual destes medicamentos poderia, provavelmente, ter contribuído para isto?

A) Amiodarona. B) Procainamida. C) Sotalol. D) Metoprolol. E) Digoxina.

6. Sra. Maria Alice, 28 anos, procura o médico com uma

queixa de dois anos de evolução, que iniciou com crises de dor que sucedia à alteração de coloração das extremidades superiores. Em princípio era somente palidez. Passados alguns meses este evento era seguido de cianose e algumas semanas após o surgimento dessa cianose, ao final da crise, manifestava-se com a tríade: palidez, cianose e rubor. Há 3 semanas apresentou pontos de necrose em polpas digitais bastante dolorosos.

Page 33: Pontifícia Universidade Católica do Paraná Processo ...static.medgrupo.com.br/static/concursos/editais/HUC/2013/GABARITOS... · A) Homem de 24 anos após uso inalatório de cocaína

Pontifícia Universidade Católica do Paraná – Programa de Residência Médica – Janeiro / 2013 Pág. 3

Considerando estas queixas, o diagnóstico mais provável é:

A) Fenômeno de Raynaud. B) Ergotismo. C) Acrocianose. D) Livedo reticular. E) Eritromelalgia.

7. Dr. Alberto estava muito preocupado com a chegada

do Sr. Antônio ao pronto-atendimento exatamente um ano após seu infarto agudo do miocárdio. Embora se considerasse um paciente obediente não havia parado de fumar. Ao chegar apresentava forte dor abdominal difusa, de início súbito há 40 minutos, seguida de náusea e vômito. Ao exame estava pálido, com sudorese importante, PA: 60/30 mmHg. Solicitado uma radiografia simples de abdome que evidenciou: alças intestinais superpostas, imóveis e com espessamento de parede. Logo após a realização do Raio X, apresentou um episódio importante de diarreia sanguinolenta. Dado este contexto, pergunta-se: qual o diagnóstico mais provável com os dados clínicos e laboratoriais apresentados?

A) Trombose venosa mesentérica aguda. B) Mesenterite retrátil. C) Oclusão arterial mesentérica aguda. D) Torção do grande epíploo. E) Pancreatite aguda.

8. Uma professora de português aposentada vem ao

consultório, trazida por familiares, pois subitamente, ao se deparar com uma texto escrito de Carlos Drummond de Andrade, o qual gostava muito, foi capaz de reconhecer as letras, mas incapaz de reconhecer as palavras ou o significado destas. A este achado semiológico denomina-se:

A) Alexia. B) Dislexia. C) Simultagnosia. D) Agnosia visual. E) Ataxia óptica.

9. O Sr. Rodolfo está internado com forte suspeita de pancreatite aguda. Foram solicitados alguns exames laboratoriais, colhidas amostras de lugares diferentes (sangue, urina e derrame cavitário). Dado esse contexto, qual deles deveria estar alterado nas três amostras para a confirmação diagnóstica?

A) Lipase. B) Tripsina. C) Cálcio D) Amilase. E) Meta-hemalbumina.

10. Sra. Clara, 81 anos, veio à consulta no CMUM, no dia 02 de janeiro após ter abusado da alimentação no final de ano, principalmente da maionese. Apresentou vários episódios de vômito e diarreia nas últimas horas. Encontraram-se mucosas secas, pele com turgor e elasticidade diminuídos, FC: 108 bpm e PA 110/70 mmHg (deitada) e 80/40 mmHg (em pé).

Foram solicitados vários exames laboratoriais antes de medicá-la. A relação da osmolaridade urinária e plasmática (Osm U/P) esperada neste caso é:

A) = 1 B) > 1 C) < 1 D) ≥ 1 E) ≤ 1

11. Paciente de 58 anos, portador de insuficiência

cardíaca por cardiopatia hipertensiva há 3 anos em acompanhamento com cardiologista e em uso de warfarina para prevenção de fenômenos tromboembólicos por fibrilação atrial. Apresentou hoje quadro súbito de tosse seca e dispneia em repouso, sem fatores desencadeantes claros, associados a dor torácica em região axilar direita, ventilatório dependente e de forte intensidade. Ao exame físico apresenta-se em mal estado geral, taquidispneico, corado, acianótico, afebril. Dados vitais demonstram uma FC irregular média de 140 bpm, PA = 102/60 mmHg, FR – 38 ipm e T – 36,8°C. Saturação de O2 em ar ambiente – 86%. Bulhas cardíacas arrítmicas em 2 tempos, sem sopros ou estalidos. Ausculta pulmonar com discretos estertores bibasais, sem outros achados. Sem edema de MMII ou empastamento de panturrilhas. ECG – ritmo de fibrilação atrial com resposta ventricular elevada, sinais de sobrecarga ventricular esquerda. Raio X de tórax – presença de cardiomegalia, campos pleuropulmonares normais. A respeito do quadro clínico acima, é CORRETO afirmar:

A) Trata-se de uma IC descompensada por uma

fibrilação atrial de alta resposta e necessita de cardioversão elétrica pela instabilidade clínica.

B) Devido à dor torácica sugestiva, mesmo sem outros achados clínicos, o diagnóstico mais provável é pneumonia de lobo médio de pulmão direito.

C) O quadro clínico sugere um quadro de pleurite aguda de etiologia a ser definida através de métodos de imagem como tomografia ou ressonância magnética.

D) Mesmo apresentando dor torácica atípica, o diagnóstico mais provável é de síndrome coronariana aguda sem alteração do ECG, já que é a causa mais comum de dor torácica.

E) A hipótese diagnóstica de tromboembolismo pulmonar não deve ser descartada, mesmo se o paciente estiver em uso de warfarina com TAP RNI entre 2.0 e 3.0.

12. Paciente de 70 anos e 70 kg, queixa-se de dores nas coxas e ombros em repouso há 10 dias, sem piora com esforço, moderada a forte intensidade. Refere que foi ao posto de saúde e seu médico recomendou o uso de AAS 100 mg 1x ao dia, sinvastatina 40 mg 1x ao dia e enalapril 10 mg 2x dia a cerca de 40 dias. Refere ser portador de hipotireoidismo há 7 anos em uso de levotiroxina 88 mcg ao dia. Seus exames atuais demonstram hemograma normal, TSH = 2,1,

Page 34: Pontifícia Universidade Católica do Paraná Processo ...static.medgrupo.com.br/static/concursos/editais/HUC/2013/GABARITOS... · A) Homem de 24 anos após uso inalatório de cocaína

Pontifícia Universidade Católica do Paraná – Programa de Residência Médica – Janeiro / 2013 Pág. 4

creatinina = 1,5 mg/dl, VHS = 30 mm/hora, CPK = 918 UI/ml. Com relação ao caso exposto, é CORRETO afirmar:

A) Trata-se de provável miopatia por estatina, mesmo após 01 mês do início da medicação, e indicação de suspensão do tratamento para reavaliação posterior.

B) É mais provável que paciente tenha fibromialgia, já que as provas inflamatórias são normais e a CPK está elevada menos que 5 x o valor de referência.

C) O diagnóstico de polimialgia reumática é provável, já que o paciente tem dores em musculatura proximal, idade maior que 55 anos e apresenta provas de atividade inflamatória elevadas.

D) A disfunção tireoidiana não interfere nos níveis de estatina; portanto, não tem relação com o risco de desenvolver miopatia.

E) O paciente não tem risco mais elevado de desenvolver miopatia por estatina por possuir função renal normal com clearence de creatinina > 50 ml/min.

13. Senhora de 48 anos, diagnosticada como portadora

de hipertensão arterial há 5 anos em uso atualmente de hidroclorotiazida 25 mg ao dia, captopril 25 mg 3x dia e atenolol 100 mg ao dia. Paciente refere estar tomando comprimidos adequadamente. Nega comorbidades. Queixa-se que apresenta seus níveis de pressão arterial descontrolados. Bom estado geral, eupneica e corada. Níveis de pressão arterial mensurados após 5 minutos de repouso, com intervalo entre as medidas de 2 minutos. PA braço direito 1ª medida: 142/98 mmHg e 2ª medida: 140/96 mmHg. Com relação ao quadro clínico acima, é CORRETO afirmar:

A) Trata-se de hipertensão arterial resistente e

deve ser otimizado tratamento clínico com antagonistas do canal de cálcio.

B) Não existem critérios de hipertensão resistente e deve ser investigada hipertensão secundária, devido à idade < 50 anos.

C) O número de comprimidos tomados durante o dia é um problema já comprovado e influencia de forma definitiva na aderência medicamentosa, mesmo que paciente afirme que está tomando todos os medicamentos da forma correta.

D) O uso de atenolol e hidroclorotiazida é uma associação segura e sem riscos, sendo assim, o ajuste do tratamento deve ser feito aumentando o inibidor da ECA.

E) A probabilidade de efeito do jaleco branco está descartada.

14. Paciente de 40 anos, sem comorbidades, refere dor torácica em região de hemitórax direito há 3 meses desencadeada aos esforços moderados como subir escadas ou andar 1-2 km, sem piora no decorrer do tempo, às vezes associada a movimentos do tórax. Seu pai faleceu aos 45 anos de IAM; portanto, está

preocupado com a possibilidade de desenvolver quadro semelhante. ECG em repouso normal. Seria CORRETO afirmar:

A) Por se tratar de angina estável com ECG

normal, não existe necessidade de investigação adicional e somente deve-se manter tratamento clínico.

B) Por se tratar de paciente de alto risco e angina estável, não está errado pedir o cateterismo cardíaco como 1ª opção.

C) Trata-se de dor provavelmente anginosa; portanto, uma cintilografia miocárdica é uma opção melhor que o teste de esforço para determinar doença coronariana.

D) A dor torácica não é anginosa; portanto, o teste de esforço serviria somente para avaliar a capacidade funcional do paciente.

E) Trata-se de dor provavelmente não anginosa e está indicado o teste ergométrico com alto valor preditivo negativo para diagnóstico de doença coronariana.

15. Considere as afirmativas sobre as parasitoses:

I. A síndrome de Löefler é uma das complicações da estrongiloidíase.

II. A amebíase pode ser tratada com secnidazol 2 g/dia, inclusive para formas extraintestinais.

III. As formas extraintestinais da giardíase podem ser mais comuns que a forma intestinal e devem ser tratadas com metronidazol EV.

É correto o que se afirma APENAS em:

A) I. B) I e II. C) III. D) I e III. E) II e III.

16. Considere as afirmativas abaixo sobre a infecção

pelo Mycobacterium leprae:

I. Do ponto vista operacional, a definição do tratamento da hanseníase pelo número de lesões cutâneas na baciloscopia é o parâmetro principal.

II. A dapsona está indicada somente nos casos multibacilares.

III. O eritema nodoso hansênico necessita de tratamento dentro das primeiras 24 horas, sendo a droga de eleição a talidomida.

É correto o que se afirma APENAS em:

A) I. B) I e II. C) II e III. D) I e III. E) III.

Page 35: Pontifícia Universidade Católica do Paraná Processo ...static.medgrupo.com.br/static/concursos/editais/HUC/2013/GABARITOS... · A) Homem de 24 anos após uso inalatório de cocaína

Pontifícia Universidade Católica do Paraná – Programa de Residência Médica – Janeiro / 2013 Pág. 5

17. Sobre a insuficiência cardíaca aguda, é CORRETO afirmar que:

A) Nos indivíduos idosos e obesos, os níveis

séricos de BNP estão elevados. B) A nitroglicerina apresenta efeito vasodilatador

arterial e venoso semelhantes. C) Os betabloqueadores devem ser sempre

suspensos pelo risco de piorar o perfil hemodinâmico ou de congestão pulmonar.

D) A dobutamina tem menor efeito inotrópico em pacientes que fazem uso prévio de betabloqueadores.

E) A maioria dos pacientes (> 50%) se apresenta na forma de edema agudo de pulmão.

18. A Doença de Chagas é uma pandemia sul-

americana. Estimativas recentes demonstram estar presente em cerca de 2-3 milhões de pessoas somente no Brasil. Estamos ainda longe da erradicação dos triatomíneos (“bicho barbeiro”) e, pior, estima-se que a infecção está tornando-se problema de saúde pública também nos EUA, onde a imigração latina pode ter levado cerca de 200-300.000 portadores desta enfermidade nos dias atuais. Com relação esta doença, é CORRETO afirmar:

A) A degeneração do sistema condutor cardíaco

pode levar às manifestações mais precoces da doença e detectadas pelo ECG, antes mesmo do aparecimento de sintomas.

B) A presença de miocardiopatia dilatada com padrão restritivo é a manifestação cardíaca mais grave da doença.

C) Uma pequena minoria de pacientes permanece na fase indeterminada da doença, ou seja, presença de IgG positivo para Chagas, sem manifestação clínica cardíaca ou gastrointestinal.

D) O tratamento com benznidazol está indicado para a forma aguda e a indeterminada da doença, sendo ineficiente na fase crônica, onde já existe fibrose tecidual.

E) As únicas formas de transmissão são a vetorial (triatomíneo), acidental, transfusional e por transplante de tecidos.

19. Paciente de 26 anos, na fase de puerpério há 1mês, fazendo uso de metildopa desde que iniciou com hipertensão arterial nas últimas 2 semanas de gravidez. Refere que há 1 semana se sente cansada e com a pele amarelada. Ao exame físico parece pálida, mas também ictérica. Seus exames demonstram Hb – 8,8 / VCM – 85,5 / HCM – 31,2 / série branca e plaquetas normais. Bilirrubinas totais elevadas (2,5X valor de referência) com urobilinogênio positivo. Teste de Coombs positivo. Neste caso é CORRETO afirmar:

A) Este quadro pode ser causado pela gravidez;

portanto, deve-se manter observação do caso. B) As bilirrubinas elevadas com urobilinogênio

positivo sugerem doença hepática aguda causada por Síndrome Helpp ou hepatite autoimune.

C) A presença de anemia normocítica sugere deficiência medular na produção de hemoglobina causada pela metildopa.

D) A contagem de reticulócitos não iria interferir na diferenciação diagnóstica da anemia aplásica em relação à anemia hemolítica.

E) A presença de teste de Coombs positivo com sinais de hemólise sugere fortemente anemia autoimune por metildopa.

20. Com relação ao tratamento das síndromes coronarianas agudas, é CORRETO afirmar que:

A) O uso de betabloqueadores é contraindicado

em pacientes portadores de DPOC nos estágios III e IV.

B) A enoxaparina é superior à heparina não fracionada no tratamento inicial do IAM sem supra ST e na angina instável de alto risco.

C) Está contraindicado o uso de inibidores da enzima conversora de angiotensina em pacientes portadores de disfunção renal e clearence de creatinina < 40 ml/min.

D) O uso de dupla antiagregação plaquetária com AAS e clopidogrel só deve ser considerado se o paciente for realizar angioplastia coronariana com stents.

E) O uso de trombolíticos endovenosos é a melhor opção terapêutica que angioplastia primária em pacientes com IAM com supra ST em menos de 6 horas de evolução.

21. Em abril de 2009 foi detectado no México um novo vírus da influenza A, o H1N1. Em poucos meses esse vírus iniciou uma pandemia global que chegou ao Brasil e ao estado do Paraná. O nosso estado foi o que mais detectou casos de SRAG (Síndrome Respiratória Aguda Grave) naquele ano, com uma incidência desta complicação de 109 casos/100.000 habitantes. Neste ano de 2012 houve um surto de menores proporções, porém, provocou as mesmas preocupações na sociedade. Com relação a esta infecção viral, é CORRETO afirmar:

A) A técnica preferencial é o swab nasal, já que o

aspirado de nasofaringe concentra menor número de células contaminadas

B) Os contatos domiciliares do paciente devem utilizar máscara protetora até o 7º dia de aparecimento dos sintomas

C) As coletas de secreção da nasofaringe para determinação do H1N1 em pacientes suspeitos devem ser realizadas de preferência até 3º dia, podendo ser realizadas até o 7° dia do aparecimento dos sintoma

D) O oseltamivir tem sua eficácia comprovada até o 4º dia do início dos sintomas e após este período gradativamente perde seu efeito

E) As grávidas que tiveram contato com familiares com infecção pelo vírus H1N1 devem obrigatoriamente fazer uso profilático do oseltamivir.

Page 36: Pontifícia Universidade Católica do Paraná Processo ...static.medgrupo.com.br/static/concursos/editais/HUC/2013/GABARITOS... · A) Homem de 24 anos após uso inalatório de cocaína

Pontifícia Universidade Católica do Paraná – Programa de Residência Médica – Janeiro / 2013 Pág. 6

22. Homem de 38 anos, pedreiro, iniciou tratamento para tuberculose pulmonar há 10 dias com terapia quádrupla. Iniciou com alteração da cor da urina e sudorese frequente. Dado este caso, pergunta-se: qual das medicações abaixo é responsável por esta reação?

A) Estreptomicina. B) Pirazinamida. C) Etambutol. D) Isoniazida. E) Rifampicina.

23. Uma senhora de 51 anos irá iniciar um tratamento

quimioterápico para uma avançada neoplasia de câncer de mama. São esperadas alterações na produção de células sanguíneas como reação à terapia instituída. Considere as assertivas abaixo:

I. Por terem meia vida média de 6 a 8 horas, a

primeira linhagem celular que apresenta deficiência de produção medular são os glóbulos brancos.

II. Por ter sua meia vida média mais longa, em média de 100 dias, a linhagem de mais lenta recuperação é a vermelha (hemácias).

III. As plaquetas são as células de mais lenta recuperação já que sua maturação e proliferação são mais lentas que a vermelha e a branca.

É correto o que se afirma APENAS em:

A) I e III. B) III. C) I e II. D) I. E) II e III.

24. Com relação à hipertensão arterial secundária, pode-

se considerar CORRETO que:

A) Pacientes com relação aldosterona sérica/atividade de renina ≤ 30 têm alta probabilidade de ser portadores de hiperaldosteronismo primário;

B) Para o tratamento inicial dos feocromocitomas, deve-se começar pelo uso de β-bloqueadores, por causa do risco de taquicardia reflexa dos α-bloqueadores.

C) O tratamento de escolha para estenose de artéria renal por doença fibrodisplásica é a angioplastia com stent.

D) O método com melhor sensibilidade e especificidade para detecção de estenoses de artérias renais > 50% é a angioressonância.

E) A coarctação de aorta é mais comum em homens, e o diagnóstico é feito na maioria das vezes por diferença da pressão arterial entre os braços > 10 mmHg.

25. Homem de 20 anos procura atendimento médico devido à dispneia de início súbito. Nega comorbidades e refere apenas tabagismo de 6 maços/ano. Nega trauma, viagens ou alterações em membros inferiores. Em seu exame físico mostrava-se taquidispneico e com murmúrio vesicular abolido

em hemitórax direito. Sua radiografia de tórax comprovava a presença de pneumotórax direito. Considerando pneumotórax espontâneo primário (PEP) e pneumotórax espontâneo secundário (PES) leia as assertivas abaixo e assinale a CORRETA.

I. O PEP ocorre caracteristicamente em pacientes

jovens e parece possuir relação com o tabagismo.

II. O PES pode estar associado à doença pulmonar obstrutiva crônica e até mesmo à infecção pulmonar por Pneumocystis jiroveci.

III. A principal causa etiológica associada ao PEP é a presença de pequenas bolhas subpleurais, as quais podem estar presentes em 50 a 80% dos pacientes que desenvolvem a doença.

Está(ão) correta(s) APENAS :

A) II. B) I e II. C) I e III. D) II e III. E) I, II e III.

26. Mulher de 68 anos, hipertensa, diabética e

depressiva, procurou atendimento médico devido à tosse produtiva, dispneia aos médios esforços e febre de 37,9oC iniciados há 3 dias. Negava tabagismo. Seus dados vitais de chegada eram:

• PA: 110/70mmHg • Pulso: 100bpm • FR: 24ipm • Taxilar: 38,0oC • SpO2: 93% (ar ambiente)

Em seu exame físico identificavam-se crepitantes pulmonares em base esquerda. Durante toda a consulta mostrava-se lúcida e cooperativa. Considerando o diagnóstico desta paciente, leia as assertivas a seguir:

I. Se a dosagem de ureia sérica desta paciente for

60 mg/dL, ela preencherá 2 critérios do escore CURB-65.

II. O seu tratamento pode ser feito com uma fluorquinolona respiratória, como a azitromicina.

III. Streptococcus pneumoniae, Mycoplasma pneumoniae e Haemophilus influenzae são possíveis agentes etiológicos para esta condição.

Está(ão) correta(s) APENAS :

A) III. B) I e III. C) I e II. D) I. E) II e III.

27. Sobre as infecções fúngicas pulmonares, leia as

assertivas abaixo e assinale V para as verdadeiras, F para as falsas. Em seguida marque a alternativa que possui a ordem CORRETA.

Page 37: Pontifícia Universidade Católica do Paraná Processo ...static.medgrupo.com.br/static/concursos/editais/HUC/2013/GABARITOS... · A) Homem de 24 anos após uso inalatório de cocaína

Pontifícia Universidade Católica do Paraná – Programa de Residência Médica – Janeiro / 2013 Pág. 7

( ) A paracoccidioidomicose pode apresentar achado radiográfico de lesões pulmonares bilaterais, em “asa de borboleta”. O Itraconazol pode ser considerado como forma de tratamento. ( ) O aspergiloma ou micetoma pode ser um achado incidental em radiografias de tórax de pacientes que estão em investigação de hemoptise e geralmente acometem os lobos superiores. ( ) Criptoccose é a infecção pulmonar de origem fúngica mais comum e manifesta-se como nódulos ou massas nos imunodeprimidos e com padrão intersticial nos imunocompetentes.

A) F, V, F B) V, V, F C) V, F, F D) V, V, V E) F, F, V

28. Sobre a avaliação da doença pulmonar obstrutiva

crônica, leia as assertivas abaixo e assinale V para as verdadeiras, F para as falsas. Em seguida marque a alternativa que possui a ordem CORRETA.

( ) A Espirometria deve ser realizada preferencialmente na fase estável da doença, sendo que a relação VEF1/CVF menor que 70% após broncodilatador confirma a presença de limitação ao fluxo aéreo. ( ) VEF1 após broncodilatador, quando analisado isoladamente, é útil na determinação do estágio da doença. ( ) A Escala do Medical Research Council (MRC) pode ser utilizada como forma de classificação da dispneia e possui relação com o prognóstico da doença.

A) V, V, F B) V, F, F C) F, V, F D) F, F, V E) V, V, V

29. Qual dos pacientes abaixo apresentaria miose como

manifestação de intoxicação exógena aguda?

A) Mulher de 60 anos após intoxicação exógena intencional por morfina.

B) Homem de 24 anos após uso inalatório de cocaína.

C) Homem de 40 anos após intoxicação exógena intencional por bupropiona.

D) Mulher de 32 anos após intoxicação exógena intencional por mirtazapina.

E) Mulher de 18 anos após intoxicação exógena intencional por amitriptilina.

30. Paciente de 70 anos, portador de hipertensão, dislipidemia e insuficiência renal crônica não dialítica (creatinina basal: 2,4 mg/dL), iniciou há 4 dias com diarreia líquida, várias vezes ao dia, sem sangue, muco ou pus. Admitia vômitos alimentares. Negava febre. Em seu exame físico apresentava-se

taquicárdico e desidratado. Em seus exames laboratoriais destacam-se:

• Creatinina: 3,8 mg/dL • Ureia: 200 mg/dL • Potássio: 6,8 mEq/L • Sódio: 150 mEq/L

Dado este quadro clínico, leia as assertivas abaixo:

I. Deve-se realizar um eletrocardiograma. II. O tratamento de sua hipercalcemia deve ser feito

com furosemida e resina de troca intestinal. III. Hidratação e uso de solução polarizante (10UI

insulina regular em 50 g de glicose) podem ser considerados no seu tratamento.

Está(ão) correta(s) APENAS :

A) III. B) II e III. C) I e III. D) I e II. E) I.

31. Paciente de 50 anos internado na enfermaria de

clínica médica evoluiu com parada cardiorrespiratória presenciada em fibrilação ventricular. O paciente recebeu atendimento preconizado pelas condutas do Suporte Avançado de Vida em Cardiologia (American Heart Association) e retornou a ritmo sinusal, com pulso central palpável e pressão arterial efetiva. Durante seu atendimento, além dos choques monofásicos e da massagem torácica, recebeu uma dose de 1mg de adrenalina e uma dose de 300 mg de amiodarona. Dado esse contexto, pergunta-se: qual a assertiva abaixo possui as doses de manutenção de amiodarona preconizadas para as próximas 24 horas?

A) 2 mg de amiodarona por minuto nas primeiras

6 horas seguidas por 1 mg de amiodarona por minutos nas próximas 18 horas.

B) 0,5 mg de amiodarona por minuto nas primeiras 6 horas seguidas por 0,25 mg de amiodarona por minutos nas próximas 18 horas.

C) 10 mg de amiodarona por minuto nas primeiras 6 horas seguidas por 5 mg de amiodarona por minutos nas próximas 18 horas.

D) 1 mg de amiodarona por minuto nas primeiras 6 horas seguidas por 0,5 mg de amiodarona por minuto nas próximas 18 horas.

E) 1 mg de amiodarona por minuto nas primeiras 24 horas.

32. Homem de 60 anos, portador de cirrose hepática por hepatite C crônica, foi trazido para atendimento devido à hematêmese e a melena iniciadas há 8 horas. Os familiares negam síncope, mas admitem que o paciente encontra-se mais sonolento que seu habitual. Traziam endoscopia digestiva alta realizada há 4 meses revelando 2 varizes de esôfago de grosso calibre. O paciente não fazia uso de medicações. Seus dados vitais na chegada eram:

Page 38: Pontifícia Universidade Católica do Paraná Processo ...static.medgrupo.com.br/static/concursos/editais/HUC/2013/GABARITOS... · A) Homem de 24 anos após uso inalatório de cocaína

Pontifícia Universidade Católica do Paraná – Programa de Residência Médica – Janeiro / 2013 Pág. 8

• PA: 100/60 mmHg • Pulso: 112 bpm • FR: 20 ipm • Taxilar: 36,8oC • SpO2: 95% (ar ambiente)

Em seu exame físico encontravam-se ginecomastia e ascite moderada. Dado este quadro clínico, afirma-se:

I. A profilaxia para peritonite bacteriana espontânea

deve ser iniciada nos pacientes que se apresentem com hemorragia digestiva alta varicosa.

II. Nos casos de hemorragia digestiva alta varicosa, o uso de terlipressina é indicado.

III. Deve iniciar imediatamente tratamento com metoprolol 50 mg de 12/12 horas e omeprazol 8 mg/hora em bomba infusora.

Está(ão) correta(s) APENAS :

A) I e II. B) I e III. C) III. D) II e III. E) II.

33. Homem de 63 anos, tabagista, hiperuricêmico e

dislipidêmico, realizou investigação de quadro dispéptico e diagnosticou uma úlcera bulbar ativa com presença de Helicobacter pylori pelo método da urease. O paciente passou por tratamento antibiótico para erradicação desta bactéria e vem ao consultório para reavaliação. Qual das assertivas abaixo possui o teste preconizado como padrão-ouro para documentar a erradicação do Helicobacter pylori após este tratamento?

A) Sorologia para Helicobacter pylori. B) Teste da urease. C) Histologia gástrica. D) Teste respiratório da ureia. E) Determinação de antígenos fecais de

Helicobacter pylori.

34. Mulher de 46 anos procura atendimento devido à paralisia facial periférica esquerda (Paralisia de Bell) iniciada há 24 horas. Nega febre, trauma ou alterações auditivas. Em seu exame físico comprovava-se apenas a presença da paralisia facial periférica esquerda. Dado este quadro clínico, afirma-se:

I. A lesão periférica do nervo facial leva ao

comprometimento do movimento da hemiface homolateral a esta.

II. O aciclovir possui superioridade à prednisona no tratamento da Paralisia de Bell e deve ser iniciado em todos os casos.

III. Todo paciente com paralisia facial periférica deve ser submetido a exame de imagem do crânio,

como tomografia computadorizada e/ou ressonância nuclear magnética.

Está(ão) correta(s) APENAS :

A) I e III. B) I e II. C) I, II e III. D) II. E) I.

35. Homem de 49 anos, portador de cirrose hepática

alcóolica (MELD 22 pontos), procurou pronto-socorro, pois há 8 meses percebeu aumento progressivo do volume abdominal. Negava dor abdominal ou febre. Referia apenas certa dispneia aos médios esforços e discreto edema de membros inferiores contínuo. Possuía os seguintes dados vitais:

• PA: 100/60 mmHg • FR: 24 ipm • Pulso: 100 bpm • Taxilar: 37,0oC • SpO2: 96% em ar ambiente

Em seu exame físico encontravam-se aranhas vasculares em tórax anterior, rarefação de pelos e ginecomastia. O médico que o atendeu realizou paracentese e os exames abaixo:

• Albumina (ascite): 1,5 mg/dL • Albumina (soro): 3,0 mg/dL • Creatinina: 1,0 mg/dL • Sódio: 140 mEq/L • Radiografia de tórax: normal

Frente ao quadro, leia as assertivas abaixo:

I. O gradiente de albumina soro-ascítico (GASA)

deste paciente é 2 e, desta forma, sua ascite provavelmente é causada por doença que promova hipertensão portal.

II. O paciente deve iniciar restrição hídrica como forma de tratamento de sua ascite.

III. No tratamento da ascite devido à cirrose hepática, a dosagem da concentração de sódio na urina de 24 horas auxilia o clínico a decidir sobre a necessidade de aumento na dose de diuréticos e na avaliação da adesão do paciente à dieta hipossódica.

Está(ão) correta(s) APENAS :

A) II e III. B) II. C) III. D) I e II. E) I e III.

36. Sobre os tipos de tratamento usados nos quadros de

constipação, associe a primeira coluna com a segunda e marque a alternativa que possui a ordem CORRETA.

Page 39: Pontifícia Universidade Católica do Paraná Processo ...static.medgrupo.com.br/static/concursos/editais/HUC/2013/GABARITOS... · A) Homem de 24 anos após uso inalatório de cocaína

Pontifícia Universidade Católica do Paraná – Programa de Residência Médica – Janeiro / 2013 Pág. 9

1. Psílio. 2. Lactulose. 3. Policarbofila cálcica. 4. Tegaserode. ( ) Fibra sintética ( ) Agonista de receptores de serotonina ( ) Laxativo osmótico ( ) Fibra natural

A) 1, 4, 2, 3 B) 3, 4, 2, 1 C) 3, 2, 4, 1 D) 1, 2, 4, 3 E) 4, 3, 2, 1

37. Mulher de 59 anos procura atendimento devido ao

aparecimento de vesículas e crostas, pruriginosas, dolorosas, dispostas em linha, sobre um de seus gradios costais à direita. Negava comorbidades. O médico que a atendeu diagnosticou a presença de herpes zoster. Qual a assertiva abaixo possui o tratamento recomendado para este caso?

A) Tenofovir 300 mg, via oral, 2 vezes ao dia. B) Aciclovir 200 mg, via oral, 5 vezes ao dia. C) Entecavir 1 mg, via oral, 1 vez ao dia. D) Aciclovir 400 mg, via oral, 5 vezes ao dia. E) Aciclovir 800 mg, via oral, 5 vezes ao dia.

38. Em seu ambulatório de clínica médica você recebeu

três irmãos encaminhados do banco de sangue para acompanhamento.

Os três repetiram os exames com intervalo de pelo menos 6 meses e as sorologias mantiveram os mesmos resultados. Dados estes resultados, leia as assertivas abaixo:

I. Osvaldo foi vacinado para hepatite B. II. Socrates possui hepatite B crônica. III. Tarsila teve contato com a hepatite B, mas a

curou.

Está(ão) correta(s) APENAS :

A) I. B) I e II. C) I, II e III. D) I e III. E) II e III.

39. Sobre o metabolismo do cálcio, leia as assertivas a

seguir:

I. O paratormônio estimula a reabsorção óssea de cálcio, aumenta a reabsorção de cálcio pelos rins e estimula a produção de calcitriol.

II. O calcitriol estimula a absorção de cálcio pelo trato gastrintestinal.

III. A calcitonina é secretada pelas células tireoidianas C (células parafoliculares) e sua secreção é estimulada pela hipercalcemia.

Está(ão) correta(s) APENAS :

A) I e II. B) I. C) I e III. D) II e III. E) I, II e III.

40. Sobre os efeitos colaterais das medicações

antipsicóticas, leia as assertivas abaixo:

I. A clorpromazina pode apresentar efeitos anticolinérgicos e prolongamento do intervalo QT.

II. A risperidona pode promover hipotensão postural. III. Quetiapina e olanzapina induzem hipertensão e

perda de peso.

Está(ão) correta(s) APENAS :

A) I, II e III. B) I. C) I e III. D) I e II. E) II e III.

41. Homem de 62 anos procurou atendimento médico

devido a aparecimento de fezes com sangue há 1 dia. O paciente nega hematêmese ou uso de qualquer remédio. Mostra-se preocupado, pois a quantidade de sangue nas evacuações aumentou desde o início do quadro e diz sentir-se tonto. Ao exame físico estava hipotenso e taquicárdico e seu toque retal resultou em sangue vivo na luva. O paciente foi internado e sua colonoscopia revelou um sangramento diverticular. Dado este quadro, é CORRETO afirmar.

I. O preparo adequado do cólon para o exame de

colonoscopia facilita a identificação do sítio sangrante e aumenta a segurança do procedimento por diminuir a chance de perfuração.

II. O sangramento diverticular é de origem arterial. III. Apesar da grande maioria dos sangramentos

diverticulares terem sangramento autolimitado, divertículos com sangramento ativo ou coágulo aderido possuem maior chance de ressangrar e podem ser tratados durante o exame de colonoscopia.

A) Somente as assertivas I e II são verdadeiras. B) Somente a assertiva I é verdadeira. C) Somente as assertivas I, II e III são

verdadeiras. D) Somente as assertivas I e III são verdadeiras. E) Somente as assertivas II e III são verdadeiras.

HBsAg Anti-HBs Anti-HBc IgG

Anti-HBc IgM

Tarcila Reagente Não reagente

Reagente Não reagente

Osvaldo Não reagente

Reagente Não reagente

Não reagente

Socrates Não reagente

Reagente Reagente Não reagente

Page 40: Pontifícia Universidade Católica do Paraná Processo ...static.medgrupo.com.br/static/concursos/editais/HUC/2013/GABARITOS... · A) Homem de 24 anos após uso inalatório de cocaína

Pontifícia Universidade Católica do Paraná – Programa de Residência Médica – Janeiro / 2013 Pág. 10

42. Mulher de 34 anos, internada para tratamento de pneumonia comunitária, evoluiu com náuseas e vômitos há 2 dias. Negava alteração do funcionamento intestinal ou gravidez. Ao exame físico mostrava-se desidratada. Em sua avaliação laboratorial detectou-se hipernatremia (sódio: 155 mEq/L). Considerando que a paciente pese 50 Kg e que o sódio ideal seja 145 mEq/L, quanto é o déficit de água aproximado, em litros, que esta paciente possui?

A) 1 L B) 1,8 L C) 3 L D) 0,5 L E) 4 L

43. Homem de 60 anos procura atendimento na

Emergência devido à taquicardia iniciada há 2 horas. Admite certa dispneia desde o início do quadro. Nega síncope ou alteração do nível de consciência, dor torácica ou febre. Seus dados vitais eram:

• PA: 70/40 mmHg • FC: 180 bpm • FR: 20 ipm • SpO2: 94% em ar ambiente • Taxilar: 36,2°C

O restante de seu exame físico era normal. Seu eletrocardiograma revelava o traçado abaixo:

Dado esse contexto, pergunta-se: qual arritmia este paciente apresenta e qual a conduta a ser tomada?

A) Fibrilação ventricular. Desfibrilação elétrica. B) Taquicardia ventricular. Amiodarona 150 mg

endovenoso. C) Ritmo de Marcapasso. Ajuste do marcapasso. D) Taquicardia ventricular. Cardioversão elétrica. E) Taquicardia supraventricular. Massagem do

seio carotídeo.

44. Enquanto aguardava na fila do refeitório, você observa que uma das cozinheiras não passa bem. Em um instante a mulher cai ao chão e não esboça nenhuma resposta aos chamados. Ao atendê-la você primeiramente pede ajuda e solicita que busquem um DEA. Sobre este dispositivo, leia as assertivas:

I. A presença de um marcapasso é uma

contraindicação ao uso do DEA. II. O DEA pode ser usado dentro da água, pois é um

dispositivo resistente.

III. Se o DEA recomendar o choque, haverá a orientação para que você cheque se todos estão afastados do paciente.

Está(ão) CORRETA(S):

A) Apenas I e III. B) Apenas I. C) Apenas III. D) Apenas II e III. E) Apenas I e II.

45. Em alguns pacientes críticos atendidos na

emergência não se consegue obter acesso venoso. Nestes casos, vias alternativas a administração dos medicamentos são essenciais. Sobre este assunto, leia as assertivas abaixo:

I. Atropina, lidocaína e vasopressina podem ser

administradas por via endotraqueal. II. O acesso intraósseo pode ser obtido em

pacientes de qualquer faixa etária. III. A via de administração endotraqueal é preferível

à via intraóssea.

Está(ão) correta(s) APENAS :

A) III. B) I e III. C) I. D) II e III. E) I e II.

46. Feminina, 52 anos, previamente hígida. Iniciou há

cerca de 10 dias com cervicalgia associada à dor em topografia de tireoide. Apresentou também febre associada e aumento dos linfonodos cervicais à ecografia. Sobre o tema representado acima, assinale a alternativa CORRETA dentre as abaixo:

A) Nas tireoidites agudas a função tireoidiana

habitualmente está alterada. B) Nas tireoidites agudas habitualmente o VHS

encontra-se com valor normal. C) Nas tireoidites agudas a contagem de

leucócitos é normal. D) Nas tireoidites agudas a biópsia por punção

por agulha fina (PAAF) mostra infiltração por linfócitos (monomorfonucleares).

E) Amiloidose ou tireoidite induzida por amiodarona são diagnósticos diferenciais da dor na tireoide.

47. João Batista, 84 anos, é admitido em hospital terciário para tratamento clínico e cirúrgico de apendicite supurada. Comorbidades observadas, à admissão: dislipidemia, hipertensão, doença arterial coronariana, doença pulmonar obstrutiva tabágica e insuficiência renal crônica EC IV. No quarto dia de internamento desenvolveu dor súbita em joelho direito, com sinais flogísticos sobre o local. Punção articular mostrou líquido sinovial inflamatório, rico em cristais com birrefringência negativa. Sobre o tema do quadro acima, assinale a alternativa CORRETA:

Page 41: Pontifícia Universidade Católica do Paraná Processo ...static.medgrupo.com.br/static/concursos/editais/HUC/2013/GABARITOS... · A) Homem de 24 anos após uso inalatório de cocaína

Pontifícia Universidade Católica do Paraná – Programa de Residência Médica – Janeiro / 2013 Pág. 11

A) Nos ataques que acometem uma ou duas

articulações, as injeções articulares de corticoide podem ser preferíveis e efetivas.

B) O esteio do tratamento agudo da gota é a administração de alopurinol na dose de 300 mg ao dia, por via oral.

C) Diclofenaco de potássio na dose de 50 mg a cada 6h por via oral deve ser utilizado por 5-7 dias na fase de doença aguda.

D) A aplicação de compressas mornas e a movimentação das articulações podem ajudar a aliviar a dor durante a crise aguda.

E) A colchicina pode ser utilizada somente após a resolução do quadro agudo de gota.

48. É uma causa de hiponatremia euvolêmica:

A) Síndrome de Cushing. B) Cirrose hepática. C) Secreção de arginina vasopressina induzida

por dor ou náuseas. D) Hiponatremia secundária a queimaduras. E) Utilização de diuréticos de alça.

49. É uma causa de hipercalcemia, EXCETO:

A) Intoxicação por vitamina D. B) Sarcoidose. C) Intoxicação por vitamina A. D) Utilização de diuréticos de alça. E) Hipertireoidismo.

50. Sobre o tema “anemia ferropriva”, assinale

Verdadeiro (V) ou Falso (F), nas sentenças descritas abaixo:

( ) A deficiência de Ferro é uma das formas mais comuns de má nutrição. ( ) A primeiro estágio da deficiência de ferro consiste em um balanço de Ferro negativo. ( ) A ocorrência de uma perda de sangue maior que 10-20 ml de eritrócitos ao dia é superior à quantidade de Ferro absorvível pelo intestino a partir de uma dieta normal. ( ) Por definição, as reservas de Ferro estão ausentes quando o nível sérico de ferritina é menor que 15 mcg por litro. ( ) Na anemia grave a hipocromia e a microcitose tornam-se mais proeminentes, aparecem células em alvo e eritrócitos deformados (pecilócitos).

Assinale a sequência CORRETA das respostas:

A) VVVVV B) VFVFV C) VVFVV D) VFFVV E) FVVFV

Page 42: Pontifícia Universidade Católica do Paraná Processo ...static.medgrupo.com.br/static/concursos/editais/HUC/2013/GABARITOS... · A) Homem de 24 anos após uso inalatório de cocaína

Pontifícia Universidade Católica do Paraná Processo Seletivo de Residênc ia Médica

20 de Janeiro de 2013

Cirurgia do Trauma Cirurgia Videolaparoscopia

Cirurgia Plástica CirurgiaTorácica Cirurgia Vascular Coloproctologia

INFORMAÇÕES / INSTRUÇÕES:

1. Verifique se a prova está completa: questões de números 1 a 50. 2. A compreensão e a interpretação das questões constituem parte integrante da prova, razão pela qual os

fiscais não poderão interferir. 3. Preenchimento do Cartão-Resposta :

- Preencher para cada questão apenas uma resposta - Preencher totalmente o espaço correspondente, conforme o modelo: - Usar caneta esferográfica, escrita normal, tinta azul ou preta. - Para qualquer outra forma de preenchimento, a leitora anulará a questão.

O CARTÃO-RESPOSTA É PERSONALIZADO. NÃO PODE SER SUBSTITUÍDO, NEM CONTER RASURAS.

Duração total da prova: 3 horas

�- - - - - - - - - - - - - - - - - - - - - - - - - - - - - - - - - - - - - - - - - - - - - - - - - - - - - - - - - - - - - - - - - - - - - - - - Anote o seu gabarito.

1. 2. 3. 4. 5. 6. 7. 8. 9. 10.

11.

12.

13.

14.

15.

16.

17.

18.

19.

20.

21. 22. 23. 24. 25. 26. 27. 28. 29. 30.

31. 32. 33. 34. 35. 36. 37. 38. 39. 40.

41.

42.

43.

44.

45.

46.

47.

48

49.

50.

Page 43: Pontifícia Universidade Católica do Paraná Processo ...static.medgrupo.com.br/static/concursos/editais/HUC/2013/GABARITOS... · A) Homem de 24 anos após uso inalatório de cocaína

Pontifícia Universidade Católica do Paraná – Programa de Residência Médica – Janeiro / 2013 Pág. 1

Page 44: Pontifícia Universidade Católica do Paraná Processo ...static.medgrupo.com.br/static/concursos/editais/HUC/2013/GABARITOS... · A) Homem de 24 anos após uso inalatório de cocaína

Pontifícia Universidade Católica do Paraná – Programa de Residência Médica – Janeiro / 2013 Pág. 2

1. A cicatrização é um processo dinâmico que envolve

uma sequência de fases mediadas por processos celulares coordenados e fatores hormonais. A primeira fase, conhecida como fase inflamatória, ocorre do início da lesão até 4 a 6 dias. As células envolvidas são os neutrófilos (pico em 24 horas) e macrófagos (48 a 96 horas). As citocinas são os fatores moduladores. Nesta primeira fase (inflamatória), as principais citocinas envolvidas são:

A) VEGF, FGF, TNF B) PDGF,EGF e TNF, IL-1 C) IL-1, TNF-α, TFG-β D) FGF, TNF-α, IL-6 E) TNF-α. IL-6, IL-10

2. Paciente com 23 anos apresenta tumoração redutível

em região inguinal D, após esforço físico. Indicada correção cirúrgica. No transoperatório o cirurgião confirma uma hérnia direta ou anteromedial ou ainda oblíqua interna. De acordo com a Classificação de Nyhus, pode-se afirmar.

A) Hérnia tipo I exteriorizada medialmente aos

vasos epigástricos. B) Hérnia tipo II exteriorizada medialmente aos

vasos epigástricos. C) Hérnia tipo III (B) exteriorizada medialmente

aos vasos epigástricos D) Hérnia tipo III (A) exteriorizada medialmente

aos vasos epigástricos. E) Hérnia tipo IV exteriorizada medialmente aos

vasos epigástricos. 3. A combinação de nível elevado de PTH-sérico,

hipercalcemia na ausência de hipocalciuria e hipofosfatemia é quase sempre patognomônica de hiperparatireoidismo.

A) A causa mais comum de hiperparatireoidismo

primário é o carcinoma seguido de hiperplasia. B) O hiperparatireoidismo pode ocorrer na

neoplasia endócrina múltipla tipo I (Síndrome de Werner) e na neoplasia endócrina múltipla tipo IIa (Síndrome de Sipple).

C) As glândulas paratireoides apresentam função intimamente relacionada com o metabolismo de cálcio, através do PTH, que diminui a calcemia pelo bloqueio da mobilização óssea do calcio e aumenta a fosfatemia.

D) A hipocalcemia aguda produz sudorese fria, rubor facial, vômitos em jato e diarreia.

E) Pacientes com diagnóstico de hiperparatireoidismo primário dificilmente têm indicação cirúrgica.

4. J.T.S., pedreiro, foi submetido a um procedimento

cirúrgico convencional de apendicectomia. Após 22 dias deu entrada no serviço de emergência, apresentando febre, inapetência, cefaleia, sinais de desidratação. No local da cirurgia você observa um importante edema com rubor local e flutuação. Encaminhado ao centro cirúrgico para drenagem do abscesso, a infecção envolve músculo oblíquo externo e músculo transverso.

Dado estecontexto, assinale a alternativa CORRETA:

A) Com estes dados pode-se afirmar que existe infecção do sítio cirúrgico tipo incisional profunda, com provável infecção por Enterococcus sp e E.coli.

B) Não se pode afirmar que existe infecção do sítio cirúrgico, pois infecção ocorre apenas até o 15° dia do pós-operatório.

C) Com estes dados pode-se afirmar que existe infecção do sítio cirúrgico tipo incisional superficial, provável infecção por MRSA.

D) Com estes dados pode-se afirmar que existe infecção do sítio cirúrgico tipo visceral, com provável infecção por Enterococcus sp e E.coli.

E) Não se pode afirmar que existe infecção do sítio cirúrgico, pois estas ocorrem apenas até o 15° dia do pós-operatório. Porém existe uma infecção de padrão superficial.

5. Paciente com nódulo de tireoide em investigação. Os

achados ultrassonográficos que denotam maior risco de câncer em um nódulo são:

I. Microcalcificações. II. Margens irregulares. III. Hipervascularização. IV. Nódulos hipoecoicos. V. A PAAF (punção aspirativa com agulha fina) é

obrigatória nos nódulos suspeitos.

Marque a alternativa CORRETA:

A) Apenas I e III são verdadeiras e V complementa.

B) Apenas IV é falsa C) Apenas I, II e III são verdadeiras e V não

complementa. D) Apenas III é falsa. E) Apenas I, II, III, IV são verdadeiras e V

complementa. 6. O Sr. P.T.S., com 38 anos hígido previamente,

apresenta uma volumosa hérnia inguinal esquerda. O cirurgião opta pela técnica cirúrgica de Lichtenstein, para corrigir o defeito herniário. Você é designado(a) para realizar a avaliação pré-operatória. Após um exame clínico normal, deverá solicitar:

A) Nenhum exame pré-operatório é necessário

para liberar o ato cirúrgico, pois paciente é hígido com menos de 40 anos.

B) ECG (eletrocardiograma), glicemia, creatinina, coagulograma e hemograma completo.

C) Coagulograma e glicemia. D) Glicemia, creatinina, coagulograma e

hemograma completo. E) ECG (eletrocardiograma), glicemia,

coagulograma e hemograma completo. 7. Desde 2006, no Brasil, a escala de gravidade de

doença hepática proposta por Child-Turcotte-Pugh (CTP), para seleção de pacientes com indicação de transplante hepático, foi substituída pelo chamado “ MELD score” . A mudança dos critérios tem em vista

Page 45: Pontifícia Universidade Católica do Paraná Processo ...static.medgrupo.com.br/static/concursos/editais/HUC/2013/GABARITOS... · A) Homem de 24 anos após uso inalatório de cocaína

Pontifícia Universidade Católica do Paraná – Programa de Residência Médica – Janeiro / 2013 Pág. 3

uma alocação de órgãos mais justa e baseada em critérios de severidade da doença.

A) A escala de MELD baseia-se em 5 valores

laboratoriais objetivos: creatinina sérica, bilirrubina total e RNI, proteínas total e gama gt.

B) A escala de MELD baseia-se em 3 valores laboratoriais objetivos: creatinina sérica, bilirrubina total e RNI e 2 critérios clínicos hematêmese previa e ascite.

C) A escala de MELD baseia-se em 3 valores laboratoriais objetivos: creatinina sérica, bilirrubina total e RNI.

D) A escala de MELD baseia-se em 5 valores laboratoriais objetivos: creatinina sérica, bilirrubina total e RNI, proteínas total e gama gt e 2 criterios clínicos: ascite e icterícia.

E) A escala de MELD baseia-se em 4 critérios clínicos: ascite, icterícia, hematêmese prévia e ascite.

8. Paciente, 43 anos, trabalhador da construção civil,

veio ao serviço ambulatorial, com queixa de dispepsia. Durante a investigação na história clínica, paciente refere piora desta dispepsia há 3 meses, quando também apresentou discreta diminuição de peso e inapetência. Você solicita exames complementares: endoscopia digestiva alta, ultrassonografia endoscópica e tomografia abdominal. Recebe, assim, um laudo de anatomia patológica informando Classificação Bormann tipo II ou ulcerado em pequena curvatura, com células em anel de sinete. Dado este quadro, sua conduta será:

A) Confirmação de neoplasia maligna, com

indicação precoce de gastrectomia. A ultrassonografia endoscópica não é necessária neste caso.

B) Desconsideração da Classificação de Bormann, pois ela não se refere ao estadiamento das lesões gástricas.

C) Confirmação de neoplasia maligna, com indicação precoce de gastrectomia, porém deve aguardar antes a ultrassonografia endoscópica.

D) Confirmação de neoplasia benigna, iniciar imediatamente bloqueador H2 ou inibidor de bomba de prótons.

E) Confirmação de neoplasia maligna, câncer gástrico precoce; a ultrassonografia endoscópica não é necessária. Indicar mucosectomia.

9. A dor abdominal no abdômen agudo deve ser

caracterizada pelo tipo, evolução, localização e irradiação e pode ser dividida em 3 tipos. De acordo com as classificações, afirma-se:

I. Dor visceral pura ou verdadeira é a que ocorre no

início do quadro, quando existe distensão do órgão sem a presença de inflamação. Dor de localização mal definida, geralmente ocorre na linha média e em cólica.

II. Peritoneal ou peritônio-pariental é aquela decorrente do comprometimento do peritônio-parietal pela inflamação visceral, traduzida pela defesa muscular ou sinais de irritação peritoneal ao exame físico.

III. As vísceras ocas intraperitoneais apresentam uma inervação sensitiva, sendo os impulsos transmitidos pelo sistema nervoso autônomo. O estímulo que desperta a dor geralmente é a distensão ou contração.

IV. A característica da dor do abdômen perfurativo é a dor tipo cólica, com intervalos variáveis.

V. A dor do abdômen hemorrágica é súbita, fraca, difusa, com intervalos curtos.

Marque a alternativa CORRETA:

A) Apenas a afirmativa III é falsa. B) Apenas as afirmativas IV e V são verdadeiras. C) Apenas as afirmativas I, II, III e IV são

verdadeiras. D) Apenas a afirmativa IV é falsa. E) Todas as afirmativas são verdadeiras.

10. Considera-se pólipo intestinal todo o tumor

circunscrito que apresente protrusão desde a parede ate o lúmen intestinal. Os pólipos podem ser únicos ou múltiplos. Quando o número de pólipos é grande são classificados em polipose intestinal. As poliposes podem apresentar características únicas. Polipose adenomatosa familiar é caracterizada pela tríade: pólipos gastrointestinais, tumores de tecidos moles (cistos epidermoides) e osteomas, acompanhada de hipertrofia do epitélio pigmentário da retina. Pacientes com pólipo intestinal são portadores da:

A) Síndrome de Turcot. B) Síndrome de Gardner. C) Síndrome de Peutz-Jeghers. D) Síndrome de Cowden. E) Síndrome de Cronkhite-Canada.

11. Em relação ao trauma abdominal segundo o ATLS

Advanced Trauma Life Suport for Doctors, assinale a alternativa CORRETA:

I. A lavagem peritoneal diagnóstica LPD é um processo diagnóstico invasivo, de rápida execução, que alcança uma sensibilidade de 50% para a detecção de hemorragia intraperitoneal.

II. A lavagem peritoneal diagnóstica pode ser indicada em um doente vítima de traumatismo multissistêmico e hemodinamicamente instável com traumatismo cranioencefálico associado.

III. A tomografia computadorizada de abdome exige a transferência do doente para o setor de radiologia e está indicado apenas nos doentes hemodinamicamente estáveis.

IV. Trauma abdominal contuso com parada cardiorrespiratória na sala de emergência é uma indicação de toracotomia de reanimação na sala de emergência.

V. O lavado peritoneal diagnóstico é considerado positivo quando se detectam bactérias pela coloração pelo Gram, quando há 100.000 ou

Page 46: Pontifícia Universidade Católica do Paraná Processo ...static.medgrupo.com.br/static/concursos/editais/HUC/2013/GABARITOS... · A) Homem de 24 anos após uso inalatório de cocaína

Pontifícia Universidade Católica do Paraná – Programa de Residência Médica – Janeiro / 2013 Pág. 4

mais glóbulos vermelhos ou 500 ou mais glóbulos brancos por mm3

.

Está(ão) correta(s) APENAS a(s) afirmativa(s):

A) I e II. B) II, III e IV. C) II. D) III. E) II, III e V.

12. Paciente do sexo feminino, 59 anos, apresenta

quadro de dor abdominal tipo aperto e vômitos há 12 horas. Os sintomas iniciaram após a refeição da noite anterior. Não tem história de uso de álcool ou de medicamentos. Ao exame físico, tem frequência cardíaca de 110 bpm, com dor de moderada intensidade a palpação epigástrica, sem sinais de irritação abdominal. Após exame clínico, laboratorial e de imagem fez-se o diagnóstico de pancreatite aguda. Sobre pancreatite aguda, assinale a alternativa CORRETA:

A) Segundo o Grupo de Estudos para a Classificação da Pancreatite, a pancreatite aguda grave caracteriza-se por apresentar três ou mais critérios do escore de Ranson, oito ou mais pontos na classificação de APACHE II, complicações pancreáticas ou a presença de falência orgânica.

B) Todos os pacientes com diagnóstico de pancreatite aguda devem ser submetidos à tomografia computadorizada de abdome sem contraste.

C) Na suspeita de necrose pancreática, deve-se solicitar como primeira opção de exame de imagem uma cintilografia.

D) Critérios de Ranson devem ser avaliados na admissão e após 48h. TGO – tansaminase oxalacética e LDH desigrogenase láctica fazem parte dos critérios de 48 horas.

E) A nutrição parenteral total NPT deve ser indicada em pacientes com pancreatite grave, não existindo outra opção de dieta para estes pacientes.

13. Paciente feminina, 50 anos, previamente hígida, deu

entrada, no pronto-atendimento, às 21 horas, com história de dor em hipocôndrio direito de forte intensidade, de início pela manhã do mesmo dia, associado à náusea e a vômitos biliosos. Nega episódios semelhantes anteriores. Nega comorbidades associadas. Ao exame físico, a paciente mostra-se estável hemodinamicamente, anictérica, com dor à palpação abdominal em quadrante superior direito e sinal de Murphy positivo. O restante do exame físico apresenta-se sem alterações. Referente ao provável diagnóstico do enunciado acima, assinale a alternativa CORRETA:

A) A paciente referida apresenta quadro clínico compatível com cólica biliar – dor biliar, podendo ser liberada sem maiores

investigações. Deverá apenas ser submetida à analgesia intra-hospitalar e receber alta com orientações dietéticas.

B) A prioridade do tratamento da colecistite aguda é o uso de antibióticos parenterais, sendo o tratamento cirúrgico uma segunda opção.

C) A cintilografia das vias biliares é um excelente método para o diagnóstico do quadro clínico descrito acima.

D) A tomografia de abdome sem contraste é o exame diagnóstico de escolha no caso de colecistite aguda e sempre deverá ser realizado na investigação desse quadro clínico.

E) O achado ecográfico mais comum que sugere inflamação aguda da vesícula biliar é uma vesícula biliar com paredes lisa e sem espessamento.

14. Um paciente masculino de 67 anos foi submetido à

correção de hérnia inguinal bilateral pela técnica de Lichtenstein. Do lado direito identificou-se um defeito herniário indireto, com anel inguinal interno sem dilatação. Do lado esquerdo observou-se hérnia recidivada direta. Assinale a alternativa que apresenta a descrição cirúrgica deste caso de acordo com a classificação de Nyhus.

A) Lado esquerdo: Nyhus I - Lado direito: Nyhus

IVc. B) Lado direito: Nyhus IIIb - Lado esquerdo:

Nyhus I. C) Lado direito: Nyhus IIIa - Lado esquerdo:

Nyhus IVa. D) Lado direito: Nyhus I - Lado esquerdo: Nyhus

IIIc. E) Lado direito: Nyhus II - Lado esquerdo: Nyhus

IVa. 15. Em relação à síndrome compartimental abdominal,

afirma-se:

I. A definição atual de síndrome compartimental abdominal (SCA) é uma pressão intraabdominal PIA maior ou igual 60 mmHg independente da presença ou ausência de falência orgânica associada.

II. Hipertensão intraabdominal é definida como uma elevação persistente ou repetida da pressão intraabdominal maior ou igual a 12 mmHg.

III. A síndrome compartimental abdominal (SCA) é consequência de um aumento agudo da pressão intra-abdominal (PIA), que promove alterações fisiológicas adversas, devido ao acometimento de vários sistemas, podendo levar à falência orgânica e à morte, caso não ocorra uma descompressão imediata da cavidade abdominal.

IV. A mensuração da PIA é considerada como dado adicional de suporte diagnóstico, podendo ser realizada de forma indireta com a aferição da pressão vesical. Atualmente a pressão vesical tornou-se o sistema padrão de mensuração da PIA, pois a bexiga é capaz de transmitir a PIA sem contribuir com qualquer pressão adicional de sua própria musculatura.

Page 47: Pontifícia Universidade Católica do Paraná Processo ...static.medgrupo.com.br/static/concursos/editais/HUC/2013/GABARITOS... · A) Homem de 24 anos após uso inalatório de cocaína

Pontifícia Universidade Católica do Paraná – Programa de Residência Médica – Janeiro / 2013 Pág. 5

V. A SCA deve ser pensada em todo paciente vítima de trauma abdominal grave, com sangramento intraperitoneal significativo e que foi politransfundido.

Está(ão) correta(s) APENAS a(s) afirmativa(s):

A) II, III, IV e V. B) III. C) IV e V. D) I. E) II e III e IV.

16. Paciente de 42 anos, com diagnóstico prévio de

colecistite crônica litiásica dá entrada no pronto-atendimento com icterícia com 2 dias de evolução. Submetido à colangiorresonância pré-operatória, que evidenciou colédoco de 3,1 cm com 8 cálculos no seu interior. Do ponto de vista cirúrgico, além da colecistectomia, qual a melhor conduta para o tratamento da coledocolitíase associada?

A) Coledocotomia e coledocorrafia sobre tubo em

“T”, saindo por contraincisão. B) Duodenotomia e papilotomia para retirada dos

cálculos seguida de duodenorrafia. C) Apenas colecistectomia sem manipulação da

via biliar principal. D) Anastomose biliodigestiva coledocojejunal em

Y de Roux. E) Litrotripsia ou uso de medicamentos como por

exemplo o ácido ursodesoxicólico - ursacol associados à papilotomia.

17. Paciente 74 anos, mulher, dá entrada no

pronto-atendimento com história de dor abdominal intensa, difusa, progressiva, sem fatores de melhora, com 1 dia de evolução associada à náusea e vômitos. Nega história prévia de cirurgia abdominal. Refere tratamento inadequado para fibrilação atrial (diagnóstico desta enfermidade feito há 4 anos). Ao exame físico paciente apresenta abdome flácido, distensão leve, com dor a palpação superficial e profunda, sem sinais de irritação peritoneal. Exames laboratoriais: hemograma com 15.000 leucócitos, 11 % de bastões, LDH 953 U/dl, Cálcio sérico de 6. Rotina de abdome agudo com sinais de edema de parede de alça intestinal. Em relação ao quadro acima, assinale o diagnóstico mais provável:

A) Volvo de sigmoide. B) Pancreatite necro-hemorrágica. C) Obstrução intestinal. D) Isquemia mesentérica. E) Úlcera péptica perfurada.

18. Em relação aos tumores de tireoide, afirma-se:

I. O adenocarcinoma papilar responde por 85% dos cânceres da glândula tireoide e se apresenta normalmente como um nódulo solitário no início da vida adulta para então se disseminar principalmente por via hematogênica

II. O adenocarcinoma folicular responde por aproximadamente 10% dos tumores malignos da

tireoide. O carcinoma de células de Hurthle é uma variante deste tipo de neoplasia.

III. O carcinoma medular contém amiloide; é um tumor sólido, duro e nodular, que não capta radioiodo e secreta calcitonina.

IV. A linfadenopatia cervical e metástases pulmonares são comuns no carcinoma indiferencido de tireoide.

V. Os depósitos metastáticos de carcinoma folicular e papilar devem ser tratados com 131 I, após tireoidectomia total ou ablação da tireoide com iodo radioativo.

A) Somente II, III, IV e V são verdadeiras. B) Somente I, III e IV é verdadeira. C) Somente II e IV são verdadeiras. D) Somente V é verdadeira. E) Somente II é verdadeira.

19. Nos casos de tratamento cirúrgico das afecções do

baço, é CORRETO afirmar:

A) A maior parte dos abscessos piogênicos do baço se formam por contiguidade das lesões no próprio baço e apresentam baixa taxa de mortalidade, em torno de 5%.

B) A presença de esplenomegalia no exame físico praticamente confirma o diagnóstico de púrpura trombocitopênica idiopática.

C) O tratamento de escolha para os cistos parasitários equinocócicos do baço é clínico, sendo a esplenectomia raramente realizada nestes pacientes.

D) Púrpura trombocitopênica idiopática e esferocitose são contraindicações absolutas para realização de esplenectomia videolaparoscópica.

E) Streptococcus pneumoniae, Haemophilus influenzae e meningococos são os micro-organismo mais frequentemente envolvidos nos casos de infecção fulminante pós-esplenectomia.

20. Em relação à terapia nutricional, assinale a

alternativa CORRETA:

A) Nutrição parenteral deve ser preferida sempre, mesmo naqueles pacientes com o trato gastrointestinal íntegro por apresentar menores taxas de complicações metabólicas

B) A bioimpedância tem sido utilizada na determinação da composição corporal e baseia-se na aplicação de corrente elétrica pelo corpo conseguindo determinar o percentual de massa magra e massa gorda.

C) Pacientes desnutridos apresentam as mesmas incidências de complicações infecciosas que pacientes nutridos.

D) O uso da infusão da dieta enteral em bolus previne o risco de broncoaspiração quando comparada à infusão contínua.

E) Para terapia nutricional parenteral por veia periférica a dieta deve ser mais hiperosmolar para evitar o risco de flebite.

Page 48: Pontifícia Universidade Católica do Paraná Processo ...static.medgrupo.com.br/static/concursos/editais/HUC/2013/GABARITOS... · A) Homem de 24 anos após uso inalatório de cocaína

Pontifícia Universidade Católica do Paraná – Programa de Residência Médica – Janeiro / 2013 Pág. 6

21. Durante sua prática clínica, independente da especialidade ou área de atuação, é comum encontrar pacientes que foram ou serão submetidos a um ato operatório. Compreender como o organismo responde a um procedimento cirúrgico é fundamental para que é médico possa orientar o seu paciente, o tratamento a ser seguido e como esse procedimento influencia em outras patologias. Por esses motivos o histórico cirúrgico faz parte da história mórbida pregressa do doente durante a anamnese. Sobre a resposta metabólica ao trauma operatório considere as seguintes afirmações:

I. O simples contato do sangue com superfícies

sem endotélio após o ato operatório pode desencadear resposta inflamatória.

II. A manutenção da nutrição por via enteral, quando possível, é importante para a melhor resposta do doente. Entretanto, ela não é capaz de alterar o consumo de proteína muscular endógena após o ato operatório.

III. A administração de somatotrofina ao doente reduz as perdas ponderal e de nitrogênio no pós-operatório, pois ela estimula a síntese protéica e a lipólise.

IV. Antes mesmo do ato operatório propriamente dito a resposta biológica do doente pode ser iniciada, pois a ansiedade e o medo que o paciente possa sentir antes do procedimento já promovem alterações psíquicas e orgânicas que levam a alterações endócrinas e vasomotoras.

É correto o que se afirma em APENAS:

A) II e III. B) II e IV. C) I, III e IV. D) I, II, III e IV. E) I e IV.

22. Um assalto a uma farmácia aconteceu em frente à

Unidade de Saúde 24horas em que você trabalha, na noite do seu plantão. Após o evento, populares assustados carregam até essa unidade de saúde um senhor de 72 anos que fora agredido. São 4h da manhã e você é chamado para avaliar o doente. O paciente apresenta-se gemente, pálido, frio e diaforético. Ele está agitado, movimentando os quatro membros e apresenta múltiplas contusões por todo o abdômen. O abdômen encontra-se distendido e doloroso difusamente à palpação. Na avaliação inicial ele apresenta FC (frequência cardíaca) de 80 bpm, PA (pressão arterial) de 60x40 mmHg e FR (frequência respiratória) de 36 irpm. Sobre este caso e com base na avaliação de choque preconizada pelo American College of Surgeons e utilizada no ATLS (Suporte de Vida Avançado no Trauma), considere as seguintes afirmações:

I. A causa mais provável de choque para esse

doente é o choque hipovolêmico. II. O uso de medicamentos como betabloqueador ou

o uso de marcapasso podem justificar a frequência cardíaca desse idoso.

III. Como a frequência cardíaca do doente não está elevada, pode-se concluir que a perda volêmica do doente foi inferior a 15%.

IV. No atendimento ao paciente idoso a frequência cardíaca é o melhor parâmetro para avaliação do estado de choque.

V. Com esta história e dados vitais a causa mais provável de choque é um choque neurogênico.

É correto o que se afirma em APENAS:

A) I, II, III e IV. B) II, III e V. C) III, IV e V. D) I e III. E) I e II.

23. Um rapaz de 26 anos sofreu uma queda de moto e é

levado a um pronto-socorro pela equipe de resgate pré-hospitalar. Você está de plantão na sala de emergência e recebe o paciente. O doente chega pálido, frio e diaforético. Na avaliação inicial ele apresenta-se obnubilado, FC (frequência cardíaca) de 144 bpm, PA (pressão arterial) de 85x40 mmHg e FR (frequência respiratória) de 38 irpm. Considerando que o peso do paciente é de 70 kg e com base na Classificação de Choque do American College of Surgeons utilizada no ATLS (Suporte de Vida Avançado no Trauma), assinale a alternativa CORRETA:

A) Esse paciente perdeu mais de 2000 mL de

sangue, um choque grau IV. B) A estimativa de perda volêmica desse paciente

é de 30 a 40%, um choque grau III. C) Esse paciente perdeu de 1500 a 2000 mL de

sangue e apresenta um choque grau II. D) Com esse quadro clínico, provavelmente o

débito urinário desse paciente ainda apresentaria valores normais.

E) Esse paciente provavelmente perdeu 15 a 30% da sua volemia.

24. O processo de reparação tecidual é fundamental para manter a integridade orgânica e homeostase do homem. Como não consegue regenerar órgãos complexos, o ser humano possui apenas a capacidade de restaurar a continuidade física entre os tecidos, processo esse conhecido por cicatrização. Sobre cicatrização, analise as afirmações a seguir:

I. O cuidado durante a sutura para que os pontos

não sejam muito apertados se deve à tentativa de evitar a hipóxia tecidual que leva à disfunção de fibroblastos e morte de células endoteliais.

II. Qualquer grau de anemia em doentes normovolêmicos e hiperglicemia afeta o processo cicatricial e deve ser sempre corrigidos.

III. Radiação e quimioterapia alteram a resposta cicatricial. O tecido irradiado apresenta, de forma geral, lesão residual das células endoteliais com atrofia e reparo tecidual não satisfatório. Evitam-se os agentes quimioterápicos até 5 a 7 dias após a operação.

IV. A deficiência de vitamina C afeta a cicatrização da ferida. Apesar de o número de fibroblastos

Page 49: Pontifícia Universidade Católica do Paraná Processo ...static.medgrupo.com.br/static/concursos/editais/HUC/2013/GABARITOS... · A) Homem de 24 anos após uso inalatório de cocaína

Pontifícia Universidade Católica do Paraná – Programa de Residência Médica – Janeiro / 2013 Pág. 7

adequado ser encontrado nas feridas, esses fibroblastos produzem colágeno em quantidade insuficientes, e a vitamina C é necessária para a hidroxilação de resíduos de lisina e pralina.

É correto o que se afirma em APENAS:

A) I e III. B) III e IV. C) II e III. D) I, III e IV. E) I, II e III.

25. Compreender as alterações nutricionais é

fundamental para o sucesso de um procedimento operatório. Todo médico deve saber que a desnutrição é um fator independente no aumento da morbi-mortalidade do doente cirúrgico para poder orientar de forma adequada o seu doente. Sobre nutrição em cirurgia analise as afirmações a seguir:

I. A complicação mais frequente da nutrição enteral

é a diarreia. II. Entre os fatores de risco para a desnutrição do

doente cirúrgico, pode-se citar a depressão, a falta de recursos financeiros dos doentes para adquirir alimentos, a falta de conhecimento de pacientes sobre a importância de uma alimentação adequada e a falta de conhecimento por profissionais da saúde sobre os problemas nutricionais.

III. Um doente que perdeu em até 6 meses mais do que 10% do seu peso sem fazer dietas deve ser considerado um portador de desnutrição.

IV. Pacientes desnutridos graves submetidos à nutrição parenteral devem ser monitorados quanto aos valores séricos de potássio, magnésio e fósforo pelo risco da nutrição parenteral deflagar a síndrome do roubo celular.

É correto o que se afirma em APENAS:

A) II e III. B) I, II e IV. C) II, III e IV. D) I e IV. E) I, II, III e IV.

26. O Sr. André Fernandes, de 25 anos, chega em um

sábado ao pronto-socorro, às 20h, acompanhado da mãe por quadro de dor abdominal. A mãe conta que pela manhã ele disse que estava sentindo a “barriga estranha”. Ela disse que estranhou muito o fato de o filho “mal ter tocado no prato” na hora do almoço, pois a feijoada aos sábados é tradição na família e o “Andrezinho sempre repetia várias vezes sua feijoada”. O André nesse momento relata que a dor piorou ao longo do dia. Ao ser perguntado sobre onde era a dor, ele coloca a mão em movimentos circulares sobre o abdômen inferior à direita. Questionado quanto à emese, ele refere um episódio de vômito alimentar. Durante sua palpação abdominal do quadrante inferior esquerdo, ele se queixou de dor no quadrante inferior direito. Apresentou também descompressão dolorosa presente à palpação da fossa ilíaca direita. Baseado

nesse contato inicial entre o paciente e o médico, o quadro clínico mais provável e melhor conduta ser tomada inicialmente são, respectivamente:

A) Apendicite aguda e anamnese e exame físico

são insuficientes para diagnosticar esse doente. É necessário solicitar tomografia computadorizada de abdômen total com contraste para elucidar o diagnóstico.

B) Apendicite aguda e apenas com anamnese e exame físico pode-se indicar a laparotomia ou videolaparoscopia para tratamento definitivo.

C) Úlcera perfurada e apenas com anamnese e exame físico pode-se indicar a laparotomia ou videolaparoscopia para tratamento definitivo.

D) Úlcera perfurada e anamnese e exame físico são insuficientes para diagnosticar esse doente. É necessário solicitar radiografia PA de cúpulas diafragmáticas para elucidação diagnóstica.

E) Gastroenterocolite aguda deve-se tranquilizar a mãe, recomendar a hidratação do doente e liberá-lo com orientações.

27. O médico deve estar preparado para o envelhecimento da população. Cada vez mais idosos são internados e submetidos a diferentes procedimentos cirúrgicos que já foram considerados proibidos nessa população anteriormente. Sobre o doente cirúrgico idoso, considere as afirmações a seguir:

I. Os principais elementos na avaliação de risco

perioperatório do idoso são: história clínica, antecedentes pessoais de patologia, medicamentos e cirurgias anteriores.

II. O risco cardíaco relacionado à história de edema pulmonar prévio é o mesmo independente do intervalo de tempo em que ele ocorreu antes da cirurgia.

III. A escala de risco da ASA (American Society of Anesthesiology) utiliza a idade de 65 anos como um de seus parâmetros (indivíduos saudáveis, abaixo de 65 anos – ASA I; doença sistêmica leve: sem limitação funcional ou idade maior que 65 anos – ASA II).

IV. Um paciente de 61 anos submetido a uma cirurgia com mais de 60 minutos de duração apresenta um risco alto de desenvolver de TVP (trombose venosa profunda) e TEP (tromboembolismo pulmonar).

É correto o que se afirma em APENAS:

A) I, II e III. B) I e IV. C) I, III e IV. D) I, II e IV. E) II e IV.

28. O Sra. Maria Televisiva Pontocom, 38 anos, chega

em uma segunda-feira pela manhã no seu consultório médico com a seguinte queixa principal: “Preciso de uma endoscopia urgente”. Ela conta que havia assistido a um programa no domingo à noite na televisão, que explicou que os sintomas que ela

Page 50: Pontifícia Universidade Católica do Paraná Processo ...static.medgrupo.com.br/static/concursos/editais/HUC/2013/GABARITOS... · A) Homem de 24 anos após uso inalatório de cocaína

Pontifícia Universidade Católica do Paraná – Programa de Residência Médica – Janeiro / 2013 Pág. 8

apresenta podem “dar câncer no esôfago” [sic]. Ela se queixa que mal conseguiu dormir à noite e virou a madrugada fazendo buscas na internet... chegou à conclusão de que precisa tirar o esôfago. Ela relata que apresenta há muitos anos uma sensação de azia e que às vezes parece que o alimento volta do “estômago até a boca” [sic] e que esse sintoma piora quando resolve deitar após uma refeição. Ela está muito preocupada, pois perdeu 3 kg com facilidade depois de iniciar caminhadas associadas a uma dieta que sua amiga lhe passou. A amiga dela não emagreceu com a dieta. Sobre endoscopia digestiva alta e a doença do refluxo gastroesofágico (DRGE), considere as seguintes afirmações:

I. O paciente portador de DRGE pode apresentar-

se com os chamados sintomas típicos caracterizados por pirose e sensação de queimação ascendente, que pioram após uma refeição copiosa ou com o hábito de deitar após a refeição.

II. A presença de epitélio escamoso na biópsia esofágica indica a presença de Esôfago de Barret.

III. Os achados endoscópicos característicos da DRGE são os diferentes graus de solução de continuidade da mucosa, traduzidos por erosões lineares localizadas a partir da junção escamocolunar.

IV. Endoscopia normal não exclui o diagnóstico de DRGE.

É correto o que se afirma em APENAS:

A) I, II e III. B) I e IV. C) I, II e IV. D) II, III e IV. E) I, III e IV.

29. Você está de plantão no último dia de disputa do

Brasileirão e nessa rodada final dois times da capital se enfrentam. Esse jogo é decisivo, pois uma das equipes foge do rebaixamento e a outra tenta vaga para a Copa Sul-Americana. Ao final do jogo acontece um grande tumulto e 50 pessoas são feridas. Considere as seguintes afirmações sobre a avaliação da gravidade das vítimas:

I. Vítimas que não respiram, porém passam a

respirar com o adequado posicionamento das vias aéreas, são vítimas que devem receber tratamento imediato (vermelho).

II. Em vítimas que não respiram, mesmo com você mantendo suas vias aéreas permeáveis com manobras simples de suporte básico de vida, inicia-se a reanimação cardiopulmonar no local. São vítimas que devem receber tratamento imediato (vermelho).

III. Vítimas que respiram espontaneamente em frequência superior a 32irpm. São vítimas que podem aguardar em cena (amarelo).

IV. Vítimas que estão caminhando no local são vítimas triadas inicialmente como leves (verde).

É correto o que se afirma em APENAS:

A) I e IV. B) I, II e IV. C) II e IV. D) I, III e V. E) II, III e IV.

30. A Sra. Clara Neves, de 73 anos, sofre há anos com

quadro de obstipação. Diz que quando faz muita força para evacuar às vezes chega a pingar sangue no vaso. Ela é frequentadora assídua da panificadora próxima de sua casa, dieta que a levou a ganhar 5 kg nos últimos 2 meses. Ela não gosta de verduras e se recusa a comer o “arroz escuro e sujo” [sic] que o médico de família lhe indicou. Diz que toma bastante líquido: “Mais ou menos 2 copos cheios de água por dia” [sic]. Ela chega no seu plantão às 15h para uma avaliação por dor abdominal. A Sra. Clara Neves refere dor intensa em quadrante inferior esquerdo do abdômen. Ao exame, ela apresenta temperatura de 38,3oC. Refere ainda que havia evacuado a última vez na tarde que antecedeu à consulta do plantão sem a presença de sangue, muco ou pus nas fezes. Disse que não teve vontade de comer e inclusive apresentou um episódio de vômito. Com base nesse quadro clínico a patologia mais provável que a Sra. Clara Neves apresenta nesse momento seria:

A) Apendicite aguda. B) Tumor de cólon perfurado. C) Diverticulite aguda. D) Hemorragia diverticular. E) Cistite.

31. O carcinoma gástrico é o quinto tumor mais comum

no Brasil. A taxa de mortalidade chega a 90%. No mundo todo há uma redução em sua incidência, mas em países como Brasil, Japão e Chille ela é ainda alta. Sobre câncer gástrico considere as afirmações a seguir:

I. Reconhecer sinais como hepatomegalia, ascite,

icterícia, nódulo de Virchow e tumor de Krukemberg são essenciais para realizar diagnóstico precoce do adenocarcinoma gástrico.

II. A presença de Helicobacter pilori é considerada um fator de risco de câncer gástrico.

III. O envolvimento de gânglios linfáticos não possui relação com a sobrevida do doente.

IV. Dentre as neoplasias malignas do estômago, o adenocarcinoma gástrico é a mais comum (95% das neoplasias malignas do estômago).

É correto o que se afirma em APENAS:

A) I e II. B) I, II e IV. C) I, II e III. D) II e IV. E) II e III.

Page 51: Pontifícia Universidade Católica do Paraná Processo ...static.medgrupo.com.br/static/concursos/editais/HUC/2013/GABARITOS... · A) Homem de 24 anos após uso inalatório de cocaína

Pontifícia Universidade Católica do Paraná – Programa de Residência Médica – Janeiro / 2013 Pág. 9

32. O Sr. Raposo Tavares, 54 anos, chega ao seu

consultório com queixa “sangue nas fezes” [sic]. Relata que tem obstipação intestinal e que o sangramento ocorre durante o esforço evacuatório em que chega a “pingar sangue no vaso”. Você inicia a anamnese já pensando nos seus conhecimentos sobre hemorragia digestiva baixa (HDB). Sobre HDB analise as afirmações a seguir:

I. Hemorragia digestiva baixa (HDB) representa

24% dos casos de hemorragia digestiva com mortalidade associada à intensidade do sangramento, à idade e a doenças associadas.

II. A cintilografia pode detectar sangramento digestivo em ritmo de 0,1 mL/min. A arteriografia pode detectar sangramento digestivo em ritmo de 1mL/min e pode precisar a fonte de sangramento, bem como possibilita o tratamento.

III. O procedimento de escolha para o diagnóstico da hemorragia digestiva baixa é a colonoscopia e está indicada principalmente em pacientes com sangramentos graves.

IV. A enterorragia sem hematêmese é, por definição, uma hemorragia digestiva baixa.

É correto o que se afirma em APENAS:

A) I. B) III e IV. C) I e II. D) I, II e III. E) I e IV.

33. O estudante de medicina Felipe Estudio, 22 anos,

chega ao seu consultório acompanhando o avô Sr. Marco Estudio, 71 anos. Durante suas últimas férias ele refere que se assustou ao reencontrar após alguns anos avô que mora no interior tão pálido e emagrecido. Ele conta que desconfiou também da reposição de ferro que Sr. Marco faz há mais de um ano para uma anemia crônica. Sr. Marco conta que apresenta evacuações como piche. Você ouve com atenção a história do estudante e do avô e inicia exames para rastreamento de câncer colorretal. Com base nesse caso clínico apresentado, julgue as afirmações a seguir:

I. O rastreamento para câncer colorretal na

população inicia-se por pesquisa de sangue oculto nas fezes. Mas, no caso do Sr. Marco Estudio, o exame inicial deveria ser a colonoscopia.

II. O melhor exame a ser solicitado para esse caso é a tomografia computadorizada abdominal com contraste por ter uma maior especificidade diagnóstica e ser menos invasiva que a colonoscopia.

III. Os tumores malignos de cólon mais comuns possuem origem no tecido epitelial.

IV. O toque retal é importante, pois detecta 50% dos tumores situados no reto e canal anal além de mostrar o grau de infiltração e mobilidade do tumor.

É correto o que se afirma em APENAS:

A) I e IV. B) II e IV. C) I, II e IV. D) I, III e IV. E) I e III.

34. O Sr. Carlos de Sol, 52 anos, trabalhador da

construção civil, chega ao seu consultório para mostrar uma lesão avermelhada próximo ao nariz de mais ou menos 1,5 cm que descama periodicamente e sangra com facilidade. Refere constante exposição ao sol, não utiliza qualquer tipo de proteção (protetor solar ou chapéu). Sobre esse tema analise as afirmações:

I. O carcinoma basocelular é o tumor maligno de

pele mais frequente do organismo. II. O carcinoma espinocelular é mais agressivo que

o carcinoma basocelular e produz metástase em até 5% dos casos.

III. Em lesões pigmentadas em que se suspeite de melanoma, deve ser realizada biópsia excisional, sempre que possível, e somente após o resultado do anatomopatológico o tratamento definitivo é planejado.

IV. A medida da profundidade do tumor é o fator mais importante para a determinação do prognóstico do melanoma.

É correto o que se afirma em APENAS:

A) I e IV. B) II, III e IV. C) I, II e III. D) III e IV. E) I, II, III e IV.

35. O Sr. João Caminho, 55 anos, etilista, chegou ao

pronto-socorro conduzido por amigos após ter vomitado grande quantidade de sangue. Você como médico plantonista, o encontra pálido (pele e mucosas), pulso 135 bpm, PA sistólica 85 mmHg, confuso e agitado. Durante seu exame o paciente apresenta novo episódio de vômito com sangue. Sobre este caso, analise as afirmações a seguir e assinale a única alternativa CORRETA:

I. Sua primeira conduta deve ser o tratamento do

choque hipovolêmico. II. A etiologia mais provável da hemorragia nesse

caso é de rotura de varizes de esôfago por hepatopatia crônica relacionada ao etilismo.

III. A EDA (endoscopia digestiva alta) está contraindicada de imediato nesse paciente devido ao novo episódio de hematêmese. A EDA possui caráter diagnóstico (de lesão sangrante e de achados preditivos de ressangramento), com baixa eficácia terapêutica.

IV. O tratamento cirúrgico é realizado em 60% dos casos de hemorragia digestiva alta. São situações que incluem, por exemplo, hemorragia maciça.

Page 52: Pontifícia Universidade Católica do Paraná Processo ...static.medgrupo.com.br/static/concursos/editais/HUC/2013/GABARITOS... · A) Homem de 24 anos após uso inalatório de cocaína

Pontifícia Universidade Católica do Paraná – Programa de Residência Médica – Janeiro / 2013 Pág. 10

É correto o que se afirma em APENAS:

A) I, II e III. B) I, II, III e IV. C) I e II. D) I e IV. E) I, III e IV.

36. O Sr. Derby Malboro, 68 anos de idade, tabagista 40

maços/anos vem ao seu consultório após radiografia de tórax realizada no exame periódico de sua empresa. Seu exame apresentou um nódulo pulmonar solitário de 2,5 cm, periférico. Sobre nódulo pulmonar solitário (NPS) analise as afirmações abaixo:

I. A eficácia da broncoscopia no diagnóstico de

lesões pulmonares está relacionada ao tamanho e à localização da lesão. Em lesões periféricas, obtém-se o diagnóstico em cerca de 70% das vezes.

II. A sensibilidade da broncoscopia para nódulos menores que 1,5 cm é de apenas 10%. A ausência de malignidade não exclui um resultado falso-negativo.

III. A punção transtorácica também pode ser útil na investigação de NPS, principalmente para lesões periféricas em contato com a pleura parietal. O pneumotórax após a punção transtorácica é a complicação mais comum, ocorrendo em até 30% dos casos.

IV. A biopsia cirúrgica é o exame padrão-ouro na investigação dos nódulos pulmonares. A videotoracoscopia é indicada, principalmente nos casos de lesões periféricas, por ser menos agressiva do que a toracotomia convencional.

É correto o que se afirma em APENAS:

A) I, II e III. B) II, III e IV. C) I, II e IV. D) III e IV. E) I, III e IV.

37. Há 10,5 milhões de adultos obesos no Brasil,

segundo o IBGE (Instituto Brasileiro de Geografia e Estatística) e a prevalência é maior entre as mulheres. Sobre as indicações de cirurgia bariátrica (NIH, 1991), analise as afirmações a seguir:

I. A indicação cirúrgica no tratamento da obesidade

mórbida está formalmente indicada para pacientes com Índice de Massa Corpórea maior igual a 35 associada a comorbidades.

II. O paciente não precisa obrigatoriamente compreender os riscos a que será submetido e as alterações em seus hábitos de vida que seguirão o procedimento antes do ato operatório. Um acompanhamento psicológico depois da cirurgia é o suficiente nesses casos.

III. Doentes dependentes químicos não são candidatos a cirurgia de obesidade mórbida.

IV. É necessário excluir os distúrbios endócrinos como causa da obesidade antes de indicar a cirurgia.

São necessárias para indicar a cirurgia bariátrica as afirmações contidas em APENAS :

A) I e II. B) I, II e III. C) I, III e IV. D) I, II e IV. E) I, II, III e IV.

38. Cerca de 30% das infecções hospitalares evitáveis.

Para reduzir e controlar as taxas de infecção hospitalar a adoção de medidas preventivas e educacionais são fundamentais. Sobre infecção em cirurgia, analise as afirmações a seguir:

I. A tricotomia deve ser limitada à área a ser

operada se o cabelo ou pelo interferir no procedimento. (categoria IA).

II. Controlar a glicemia em todos os paciente diabéticos, evitando em particular e hiperglicemia peroperatória. (categoria IB).

III. Não utilizar pro-pés com o intuito de prevenir infecção de sítio cirúrgico. (categoria IB).

IV. Antes de cirurgias colorretais, realizar o preparo mecânico dos cólons. Administrar antimicrobianos orais, não absorvíveis, em doses divididas no dia anterior à cirurgia. (categoria IA).

Fazem parte do consenso em prevenção de infecção do sítio cirúrgico do CDC (Center for Disease Control) as afirmações contidas em APENAS :

A) I, II, III e IV. B) I, III e IV. C) I e IV. D) III e IV. E) II e III.

39. Você recebe um trabalhador de um silo que sofreu

uma queimadura após a explosão de grãos. Ele apresenta-se muito agitado, reclamando de dor no corpo todo e principalmente no antebraço esquerdo, em que ele também apresenta formigamento. Ele não usava os EPI’s (equipamentos de proteção individual) no momento da explosão. Você observa queimaduras extensas em face, troncos e membros superiores com queimadura circular região proximal de antebraço esquerdo. Os compartimentos do antebraço estão visivelmente tensos. Sua primeira conduta nesse caso deve ser:

A) Realizar escarotomia do antebraço. B) Garantir acesso venoso central e infusão

rápida de líquidos. C) Realizar analgesia intramuscular com opioides. D) Realizar curativo das lesões. E) Avaliar permeabilidade das vias aéreas.

40. Um pólipo é qualquer estrutura tecidual em forma de

proeminência circunscrita do lúmen intestinal. Sua incidência chega a 25% em indivíduos com mais de 50 anos em países ocidentais. Por isso, é

Page 53: Pontifícia Universidade Católica do Paraná Processo ...static.medgrupo.com.br/static/concursos/editais/HUC/2013/GABARITOS... · A) Homem de 24 anos após uso inalatório de cocaína

Pontifícia Universidade Católica do Paraná – Programa de Residência Médica – Janeiro / 2013 Pág. 11

fundamental que independente da especialidade clínica ou cirúrgica, o médico saiba suas características e tratamento. Sobre os pólipos intestinais, analise as afirmações abaixo:

I. O único tratamento efetivo na prevenção do

câncer colorretal na polipose adenomatosa familiar é a colectomia total.

II. As manifestações clínicas dos pólipos intestinais são pouco expressivas sendo a eliminação de sangue pelo ânus a manifestação mais comum.

III. Dos pólipos colônicos identificados durante o exame endoscópico devem ser ressecados apenas aqueles com aparência carcinomatosa.

IV. A polipose adenomatosa familiar é uma condição autossômica dominante.

É correto o que se afirma em APENAS :

A) I, II, III e IV. B) II, III e IV. C) II e III. D) I, II e IV. E) I e II.

41. A primeira tentativa de um transplante hepático

ocorreu em 1963. Ele é uma opção para as doenças hepáticas agudas ou crônicas irresponsivas a alternativas terapêuticas. Mas nem todo o paciente pode receber o transplante de fígado. Sobre as indicações e contraindicações do transplante de fígado analise as afirmações a seguir e assinale a única alternativa CORRETA:

I. O alcoolismo persistente é uma contraindicação

absoluta ao transplante de fígado. II. Trombose de veia porta é uma contraindicação

relativa. III. A presença de infecção ou neoplasia extra-

hepática é considerada uma contraindicação absoluta ao transplante hepático.

IV. O hepatocarcinoma maior que 5 cm é uma contraindicação absoluta ao transplante.

É correto o que se afirma em APENAS:

A) I, II e IV. B) I e III. C) I, II e III. D) III e IV. E) I, III e IV.

42. Na saída de um bar em Curitiba, um rapaz foi

abordado por um motociclista armado. Ao reagir à tentativa de assalto o rapaz foi atingido por um projétil de calibre 38 com entrada em 4o EICD (espaço intercostal direito) na LAM (linha axilar média), sem orifício de saída. Um taxista que passava pelo local recolheu o rapaz e levou-o ao hospital mais próximo e encontrou você como médico plantonista. Você recebe um paciente pálido, frio, diaforético. Ele apresenta jugulares ingurgitadas, murmúrio vesicular abolido à direita com hipertimpanismo à percussão também à direita, PA 70x40 mmHg e FC 143 bpm. Sobre esse caso

assinale qual a hipótese diagnóstica mais provável e conduta inicial mais adequada, respectivamente:

A) Pneumotórax Hipertensivo / Drenagem de

tórax em selo d’água. B) Pneumotórax Hipertensivo / Toracocentese de

alívio. C) Pneumotórax Aberto / Curativo de três pontas. D) Pneumotórax Aberto / Drenagem de tórax em

selo d’água. E) Hemotórax Maciço / Drenagem de tórax em

selo d’água.

43. Durante um dos dias de agenda aberta para atendimento na Unidade de Saúde de um médico da ESF (Estratégia Saúde da Família), acontece uma briga entre dois vizinhos daquela área: Cleyton e Cledyr. Cleyton possuía um revólver calibre 44 ilegal em sua residência. No momento da raiva acabou usando a arma e atirou contra o tórax de Cledyr, mas se arrependeu em seguida. O próprio Cleyton carregou Cledyr até a unidade de saúde onde ele foi atendido inicialmente pelo médico da ESF, que corretamente diagnosticou e tratou o problema que colocava Cledyr em risco iminente de morte. Cledyr foi encaminhado pelo SAMU posteriormente a um Hospital Referência em Trauma da cidade e em menos de 2 semanas já estava de volta ao lar. Sobre o atendimento ao traumatismo torácico analise os itens a seguir:

I. Pneumotórax Hipertensivo. II. Pneumotóra Simples. III. Pneumotórax Aberto. IV. Tamponamento Cardíaco. V. Hemotórax Maciço. VI. Lesão de Aorta. VII. Rotura de Esôfago. VIII. Trauma Cardíaco Contuso. IX. Obstrução de Vias Aéreas. X. Lesão Transfixante de Mediastino. XI. Tórax Instável.

É correto o que se afirma em APENAS:

A) I, III, IV, V, VI, IX, X e XI. B) I, IV, VI e X. C) I, III, IV, V, X e XI. D) I, III, IV, V, IX e XI. E) I, III, IV, V, VI, VIII, X e XI.

44. O médico na sala de emergência deve saber ouvir a

história contada pela equipe pré-hospitalar que atendeu, transportou e entregou o paciente politraumatizado no pronto-socorro. Detalhes desta história podem levar o médico plantonista a suspeitar de possíveis lesões internas. Sobre a cinemática do trauma, analise as relações de mecanismo de trauma e lesão mais provável:

I. Se durante uma colisão frontal o condutor sem

cinto de segurança for arremessado para frente e para cima, haverá o sinal de teia de aranha no para-brisa e o paciente provavelmente apresentará lesão cerebral traumática.

Page 54: Pontifícia Universidade Católica do Paraná Processo ...static.medgrupo.com.br/static/concursos/editais/HUC/2013/GABARITOS... · A) Homem de 24 anos após uso inalatório de cocaína

Pontifícia Universidade Católica do Paraná – Programa de Residência Médica – Janeiro / 2013 Pág. 12

II. Se durante um colisão frontal, o condutor sem cinto de segurança for arremessado para frente e para baixo, haverá fratura de painel e contusão sobre o joelho e impacto sobre a tíbia com possível lesão da artéria poplítea.

III. Durante a colisão traseira o condutor pode sofrer o mecanismo de chicote com lesão medular.

IV. Durante uma colisão frontal com desaceleração brusca do tronco do condutor pode ocorrer rotura da aorta entre o arco aórtico e a aorta descendente.

É correto o que se afirma em APENAS:

A) I, II e III. B) I e II. C) I, II, III e IV. D) I e III. E) II, III e IV.

45. Fístula digestiva é uma comunicação anormal entre

dois órgãos internos ou entre um órgão interno e a superfície corporal. Sobre fístulas digestivas considere as afirmativas abaixo:

I. Desordens nutricionais são comuns (55 a 90%)

em portadores de fístulas digestivas. II. O fechamento espontâneo da fístula pode ocorrer

em 60% dos casos com suporte nutricional adequado e sepse controlada.

III. Pacientes com peritonite generalizada e sepse sistêmica por fístula digestiva apresentam 70% de mortalidade.

IV. O octreotide reduz as perdas hidroeletrolíticas e de nutrientes pela fístula.

É correto o que se afirma em APENAS:

A) I, III e IV. B) II e IV. C) I, II e III. D) II e III. E) I, II, III e IV.

46. Um rapaz de 27 anos sofreu uma queda de moto em

alta velocidade. O capacete, que não estava adequadamente preso à cabeça do doente, soltou durante a queda. O paciente ficou imediatamente arresponsivo no local. Ao exame inicial ele abria o olho apenas à dor, falava sons incompreensíveis e fazia movimento de decorticação do lado esquerdo e descerebração do lado direito. A tomografia de crânio mostrava um hematoma em forma de lente côncavo-convexa à esquerda. Sobre esse caso analise as afirmações abaixo:

I. Esse paciente possui indicação de entubação

orotraqueal. II. A Escala de Coma de Glasgow desse doente é 7,

um trauma cranioencefálico grave. III. A Escala de Coma de Glasgow desse doente é 6,

um trauma cranioencefálico grave. IV. Esse paciente apresenta um hematoma epidural. V. Esse paciente apresenta um hematoma subdural.

É correto o que se afirma em APENAS :

A) I, II e IV. B) I, III e V. C) I, III e IV. D) I e IV. E) I, II e V.

47. O Sr. Cláudio Coelho, 81 anos, chega ao hospital

vítima de trauma. Ele foi atingido por um projétil de arma de fogo no tórax durante um assalto que sofreu ao sair da casa nos netos. Foi encontrado agitado, se queixando-se de dificuldade para respirar com hipertimpanismo e murmúrio vesicular abolido em hemitórax esquerdo, distensão de jugulares, abdômen flácido, pelve estável sem outras lesões. Apresenta FC de 135 bpm, PA de 80x60 mmHg, FR de 32 irpm, SaO2 de 85%. Assinale a alternativa que indica a primeira conduta a ser tomada:

A) Avaliar a permeabilidade da via aérea. B) Entubação endotraqueal. C) Toracocentese de alívio. D) Acesso venoso com reposição com solução de

cristaloide E) Drenagem de tórax em selo d’água.

48. O Sr. João Marca de 35 anos está em avaliação por

um quadro de dor abdominal, dispepsia e esteatorreia. Sobre a pancreatite crônica analise as afirmações a seguir:

I. A ingesta crônica de álcool leva à secreção de

suco pancreático rico em proteínas e baixo em volume e bicarbonato. A precipitação proteica nos ductos leva à formação de rolhas proteicas que obstruem os ductos e levam à fibrose.

II. A indicação mais comum para o tratamento cirúrgico da pancreatite crônica é a dor abdominal.

III. O melhor exame para diferenciar a pancreatite crônica do câncer de pâncreas é a tomografia computadorizada.

É correto o que se afirma em APENAS:

A) II. B) I, II e III. C) I e II. D) I e III. E) II e III.

49. Coleções purulentas na cavidade abdominal são

conhecidas como abscessos abdominais. Dois terços deles ocorrem adjacentes a um processo inflamatório visceral, sem peritonite concomitante. Sobre abscessos abdominais considere as seguintes afirmações:

I. A persistência de um processo infeccioso após a

resolução de uma peritonite difusa pode dar origem a um abscesso.

II. A maioria dos abscessos intracavitários possuem é causado por gram-negativos, anaeróbios e enterococos.

Page 55: Pontifícia Universidade Católica do Paraná Processo ...static.medgrupo.com.br/static/concursos/editais/HUC/2013/GABARITOS... · A) Homem de 24 anos após uso inalatório de cocaína

Pontifícia Universidade Católica do Paraná – Programa de Residência Médica – Janeiro / 2013 Pág. 13

III. O abscesso intra-abdominal cursa com leucocitose com desvio à esquerda.

IV. O melhor exame para localização de abscessos intra-abdominais é a ultrassonografia.

É correto o que se afirma em APENAS:

A) I e II. B) II e IV. C) II, III e IV. D) I, II e III. E) I e III.

50. A hérnia abdominal é uma protrusão de órgão ou

víscera por um defeito ou orifício na parede abdominal. Sobre hérnias abdominais considere as afirmativas abaixo e assinale a alternativa CORRETA:

I. Hérnia de Spiegel ocorre entre a linha semilunar

lateralmente e a borda lateral do músculo reto abdominal.

II. Hérnia encarcerada é a hérnia com protrusão crônica e comprometimento vascular.

III. Hérnia de Littre é aquela que contém uma parede do intestino delgado.

É correto o que se afirma em APENAS:

A) I. B) I e III. C) III. D) I e II. E) II e III.